Está en la página 1de 108

MANUAL ENARM

NEUMOLOGÍA Y CIRUGÍA TORÁCICA


(edición 2022)

ISBN
978-84-18767-14-2

DEPÓSITO LEGAL
M-26699-MMXIX

ACADEMIA AMIR MÉXICO S. DE R. L. DE C. V.


www.amirmexico.com
info@amirmexico.com

DISEÑO Y MAQUETACIÓN
Iceberg Visual Diseño, S.L.N.E.

La protección de los derechos de autor se extiende tanto al contenido redac-


cional de la publicación como al diseño, ilustraciones y fotografías de la misma,
por lo que queda prohibida su reproducción total o parcial sin el permiso del
propietario de los derechos de autor.

Este manual ha sido impreso con papel ecológico, sos-


tenible y libre de cloro, y ha sido certificado según los
estándares del FSC (Forest Stewardship Council) y del PEFC
(Programme for the Endorsement of Forest Certification).
AUTORES

DIRECCIÓN LUIS GUILLERMO MORENO MADRIGAL EDUARDO FRANCO DÍEZ


EDITORIAL Dirección Académica AMIR México. H. U. Ramón y Cajal, Madrid.
JAIME CAMPOS PAVÓN AIDA SUÁREZ BARRIENTOS
H. U. 12 de Octubre, Madrid. Clínica Universidad de Navarra, Madrid.
BORJA RUIZ MATEOS
H. Infanta Cristina. Parla, Madrid y
H. Central de la Cruz Roja. Madrid.

AUTORES ANTÓN SANTOS, JUAN MIGUEL (9) ALONSO GARCÍA-POZUELO, JAVIER (52)
FRANCO DÍEZ, EDUARDO (7) DORANTES VALDÉS, BENJAMÍN (65)
GALLO SANTACRUZ, SARA (24) LÓPEZ GONZÁLEZ, BERENICE (75)
GREDILLA-ZUBIRÍA, ÍÑIGO (27) MORENO MADRIGAL, LUIS GUILLERMO (77)
LÓPEZ-SERRANO, ALBERTO (30)
PASCUAL GUARDIA, SERGI (37)
RUIZ MATEOS, BORJA (42)
SESMA ROMERO, JULIO (43)
TEIGELL MUÑOZ, FRANCISCO JAVIER (9)

ILUSTRACIONES CARLA AVILÉS HUICOCHEA


Hospital General Regional N.º 1 Cuernavaca. IMSS. CDMX.
BIANCA SHECCID NÁJERA PALOMARES
Hospital General Regional N.º 1 Cuernavaca. IMSS. CDMX.

5
RELACIÓN GENERAL DE AUTORES
ESPAÑA
ADEVA ALFONSO, JORGE (1) GALLO SANTACRUZ, SARA (24) ORTIZ SALVADOR, JOSÉ MARÍA (15)
ALEDO-SERRANO, ÁNGEL (2) GANDÍA GONZÁLEZ, MARÍA LUISA (4) OTAOLA ARCA, HUGO (36)
ALONSO PEREIRO, ELENA (3) GARCÍA CARRERAS, ALEJANDRO (1) PADULLÉS CASTELLÓ, BERNAT (10)
ALONSO SANZ, JAVIER (4) GARCÍA SEBASTIÁN, CRISTINA (7) PANADÉS-DE OLIVEIRA, LUISA (13)
ÁLVAREZ ANDRÉS, EVA (5) GARCÍA-ESCRIBANO MARTÍN, FLORENCIO (13) PARRA DÍAZ, PAULA CAROLINA
AMMARI SÁNCHEZ-VILLANUEVA, FADI (6) GARROTE GARROTE, MARÍA (21) PASCUAL GUARDIA, SERGI (37)
AMORES LUQUE, MIGUEL CAYETANO (7) GIMÉNEZ VALLEJO, CARLOS (25) PASCUAL MARTÍNEZ, ADRIANA (38)
ANTÓN MARTIN, MARÍA DEL PILAR (8) GÓMEZ ROMERO, MARÍA (26) PÉREZ SÁNCHEZ, EZEQUIEL JESÚS (39)
ANTÓN SANTOS, JUAN MIGUEL (9) GÓMEZ SERRANO, MANUEL (13) PÉREZ TRIGO, SILVIA (12)
ARREO DEL VAL, VIVIANA (4) GÓMEZ-MAYORDOMO, VÍCTOR (13) PINILLA SANTOS, BERTA (40)
BALBACID DOMINGO, ENRIQUE J. (4) GÓMEZ-PORRO SÁNCHEZ, PABLO (22) PINTOS PASCUAL, ILDUARA (17)
BATALLER TORRALBA, ÁLEX (10) GONZÁLEZ ROCAFORT, ÁLVARO (4) PIRIS BORREGAS, SALVADOR (12)
BENAVENT NÚÑEZ, DIEGO (4) GREDILLA-ZUBIRÍA, ÍÑIGO (27) PLASENCIA RODRÍGUEZ, CHAMAIDA (4)
BENÍTEZ, LETICIA GUIJARRO VALTUEÑA, AINHOA (22) RAMIRO MILLÁN, PATRICIA (41)
BERNAL BELLO, DAVID (11) HERNÁNDEZ ONTORIA, MARÍA (12) RAMOS JIMÉNEZ, JAVIER (7)
BUZÓN MARTÍN, LUIS (1) HONRUBIA LÓPEZ, RAÚL (28) RODRÍGUEZ DOMÍNGUEZ, VÍCTOR (4)
CABRERA MARANTE, ÓSCAR (12) IBÁÑEZ-SANZ, GEMMA (29) RODRÍGUEZ-BATLLORI ARÁN, BEATRIZ (9)
CAMPOS PAVÓN, JAIME (12) LALUEZA BLANCO, ANTONIO (12) RODRÍGUEZ-MONSALVE, MARÍA (22)
CANO-VALDERRAMA, ÓSCAR (13) LÓPEZ-SERRANO, ALBERTO (30) RUIZ MATEOS, BORJA (42)
CARDOSO LÓPEZ, ISABEL (14) LOUREIRO AMIGO, JOSÉ (49) SÁNCHEZ VADILLO, IRENE (4)
CERVERA YGUAL, GUILLERMO (15) LOZANO GRANERO, CRISTINA (7) SESMA ROMERO, JULIO (43)
CÍVICO ORTEGA, JESÚS ANTONIO (16) LUENGO ALONSO, GONZALO (12) SEVILLA-RIBOTA, SERGIO (9)
COBREROS PÉREZ, ÁLVARO MAEZTU, MIKEL (31) SOUTO SOTO, AURA DANIELA (22)
CORRALES BENÍTEZ, CARLOS (17) MANJÓN RUBIO, HÉCTOR (7) SUÁREZ BARRIENTOS, AIDA (44)
CUENCA RAMÍREZ, MARÍA DESAMPARADOS (18) MARCO ALACID, CRISTIAN (32) TABEAYO ÁLVAREZ, ELOY (4)
CUESTA HERNÁNDEZ, MARTÍN (13) MARTÍN RUBIO, INÉS (22) TAJIMA POZO, KAZUHIRO (20)
CUÑO ROLDÁN, JOSÉ LUIS (11) MARTÍNEZ DÍEZ, JAVIER (33) TARAMINO PINTADO, NOELIA (12)
DÁVILA GONZÁLEZ, PABLO (19) MARTÍNEZ DÍEZ, JOSÉ MANUEL (4) TEIGELL MUÑOZ, FRANCISCO JAVIER (9)
DE MIGUEL-CAMPO, BORJA (12) MARTÍNEZ-FIDALGO VÁZQUEZ, CONCEPCIÓN (9) TORRES FERNÁNDEZ, DAVID (12)
DELGADO LAGUNA, ANA (20) MARTOS GISBERT, NATALIA (5) TOUZA FERNÁNDEZ, ALBERTO (45)
DELGADO MÁRQUEZ, ANA MARÍA (48) MASANA FLORES, ELENA (34) UDONDO GONZÁLEZ DEL TÁNAGO, MARÍA (31)
ESTEBAN-SÁNCHEZ, JONATHAN (21) MOGAS VIÑALS, EDUARD (35) VALTUEÑA SANTAMARÍA, JARA (46)
FERRE-ARACIL, CARLOS (22) MONJO HENRY, IRENE (4) VÁZQUEZ GÓMEZ, FELISA (47)
FORTUNY FRAU, ELENA (23) MUERTE MORENO, IVÁN (13) VÁZQUEZ GÓMEZ, JULIO ALBERTO (47)
FRANCO DÍEZ, EDUARDO (7) NAVARRO ÁVILA, RAFAEL JOSÉ (12) VILLANUEVA MARTÍNEZ, JAVIER (9)

(1) H. G. U. Gregorio Marañón. Madrid. (19) H. de Manacor. Mallorca. (35) H. U. Vall d’Hebron. Barcelona.
(2) H. Ruber Internacional. Madrid. (20) H. U. Fundación Alcorcón. Madrid. (36) Clínica Alemana. Santiago de Chile, Chile.
(3) H. U. del Sureste. Arganda del Rey, Madrid. (21) H. U. de Getafe. Madrid. (37) Parc de Salut Mar. Barcelona.
(4) H. U. La Paz. Madrid. (22) H. U. Puerta de Hierro. Madrid. (38) H. U. Infanta Elena. Madrid.
(5) H. U. Severo Ochoa. Madrid. (23) H. U. Son Espases. Palma de Mallorca. (39) Instituto de Neuropsiquiatría y Adicciones,
(6) H. U. Virgen del Rocío. Sevilla. (24) H. Can Misses. Ibiza. PSMAR. Barcelona.

(7) H. U. Ramón y Cajal. Madrid. (25) Centre d’Ophtalmologie Sainte Odile. (40) Psiquiatra en ámbito privado. Madrid.

(8) Phoenix Children´s Hospital. Phoenix, EE.UU. Alsacia, Francia. (41) H. C. U. Lozano Blesa. Zaragoza.

(9) H. Infanta Cristina. Parla, Madrid. (26) H. U. Joan XIII. Tarragona. (42) H. Infanta Cristina. Parla, Madrid y
(27) H. Quironsalud A Coruña. La Coruña. H. Central de la Cruz Roja. Madrid.
(10) H. Clinic. Barcelona.
(28) H. U. Infanta Sofía. Madrid. (43) H. G. U. de Alicante. Alicante.
(11) H. U. de Fuenlabrada. Madrid.
(29) H. U. de Bellvitge. L’Hospitalet de Llobregat, (44) Clínica U. de Navarra. Madrid.
(12) H. U. 12 de Octubre. Madrid.
Barcelona. (45) H. U. de Torrejón. Torrejón, Madrid y
(13) H. C. San Carlos. Madrid. H. HM Puerta del Sur. Móstoles, Madrid.
(30) H. U. San Juan de Alicante. Alicante.
(14) H. Vithas Ntra. Sra. de América. Madrid. (46) H. C. U. de Valladolid. Valladolid.
(31) H. U. de Basurto. Bilbao.
(15) H. Central U. de Valencia. Valencia. (47) H. Infantil U. Niño Jesús. Madrid.
(32) H. Virgen de los Lirios. Alcoy, Alicante.
(16) H. U. Virgen de la Victoria. Málaga. (48) H. U. Rey Juan Carlos. Móstoles, Madrid.
(33) H. U. Central de Asturias. Oviedo.
(17) H. U. Fundación Jiménez Díaz. Madrid. (49) H. Moisès Broggi. Sant Joan Despí, Barcelona.
(34) H. U. Virgen de las Nieves. Granada.
(18) H. U. Doctor Peset, Valencia.

6
MÉXICO
AGUILAR NÁJERA, OCTAVIO (50) GONZÁLEZ CHÁVEZ, ALBERTO MANUEL (67) PAZ GUZMÁN, JOSÉ DANIEL (78)
ALEMÁN GARAY, EDUARDO MOISÉS (51) GONZÁLEZ LAUREANI, JESÚS (63) PÉREZ GUZMÁN, MIREYA CITLALI (79)
ALONSO GARCÍA-POZUELO, JAVIER (52) GONZÁLEZ PÉREZ, ITZEL (68) PÉREZ PÉREZ, JOSÉ RENAN (80)
AMARO GUTIÉRREZ, NÉSTOR (51) GUAKIL SAKRUKA, LINDA (69) RAMÍREZ HINOJOSA, JUAN PABLO (63)
AQUINO RAMOS, ALEJANDRA LUCÍA (53) GUERRA BLANCO, PALOMA (70) RANGEL SELVERA, OMAR ALEJANDRO (81)
ARCEO OLAIZ, RICARDO ANTONIO (54) HERNÁNDEZ ESQUIVEL, CARLOS ARTURO (58) REBULL ISUSI, JUAN MANUEL (82)
BAHENA LÓPEZ, JOSÉ EDUARDO (55) HERNÁNDEZ VEGA, BRENDA (71) RODRÍGUEZ GUTIÉRREZ, GEORGINA (74)
BENARDETE HARARI, DENISE NIZA (56) HINOJOSA AMAYA, JOSÉ MIGUEL (72) ROMERO BARBA, ROSA NALLELY (70)
CAMACHO ROSAS, LAURA HAYDEE (57) INTRIAGO ALOR, MARIELLE (73) RUIZ MATA, JUAN MANUEL (83)
CANTORAL FARFÁN, EMILIA (58) ISLAS ESCOTO, SANTIAGO (51) SANTOS GRAPAIN, SANTIAGO (51)
CASTILLO BÁRCENA, EDGAR DAVID (59) LARA MARTÍNEZ, ALDO ENRIQUE (74) TORRES VÁZQUEZ, JUAN AGUSTÍN (84)
CHÁVEZ DELGADO, NALLELY EDUVIGES (60) LÓPEZ GONZÁLEZ, BERENICE (75) TOVAR UGALDE, MARTHA SARAI (85)
CHIAPAS GASCA, KARLA (61) MÁRQUEZ GONZÁLEZ, STEPHANY MICHELLE (61) VALDÉS FERRER, SERGIO IVÁN (50)
CRUZ GÓMEZ, CLAUDIA NAYELI (62) MARTÍN MARTÍN, MARÍA ASUNCIÓN (76) VELÁZQUEZ GUTIÉRREZ, CARLOS NORMAN (79)
DELANO ALONSO, ROBERTO (63) MARTÍNEZ CASTAÑEDA, ERIKA ADRIANA (74) VIDAL ROJO, PAOLA (86)
DÍAZ ESPINOZA, JORGE LUIS (64) MERAZ ÁVILA, DIEGO (56) VILLANUEVA RODRÍGUEZ, LUISA GERALDINE (56)
DORANTES VALDÉS, BENJAMÍN (65) MORENO MADRIGAL, LUIS GUILLERMO (77) ZARAGOZA CARRILLO, RICARDO EMMANUEL (87)
DURÁN PIÑA, ELIZABETH ANDREA (51) MUÑOZ CASTAÑEDA, WALLACE RAFAEL A (74) ZENTENO RUIZ, JUAN CARLOS (88)
GARCÍA ALBISUA, ANA MERCEDES (66) PALACIOS GARCÍA, ALBERTO AGUSTÍN (50) ZÚÑIGA DOMÍNGUEZ, LIZBETH ADRIANA (51)

(50) Inst. Nacional de Ciencias Médicas y Nutrición (63) H. Gral. Dr. Manuel Gea González. CDMX. (76) Neonatología, Unidad Médica de Alta
Salvador Zubirán. CDMX. (64) Facultad de Medicina UNAM. CDMX. Especialidad IMSS. Mérida, Yucatán.
(51) H. General de México. CDMX. (65) Depto. de Farmacología. Facultad de Medicina (77) H. Gral. Regional 1 Dr. Carlos Mac Gregor
(52) Dirección académica AMIR Rep. Dominicana. UNAM. CDMX. Sánchez Navarro, IMSS. CDMX.

(53) CMN 20 de Noviembre del ISSSTE. CDMX. (66) Asociación Para Evitar la Ceguera en México. (78) H. Gral. de Zona 33 IMSS. Monterrey, Nuevo León.

(54) H. del Niño y del Adolescente Morelense. CDMX. (79) Centro Médico ABC. CDMX.
Estado de Morelos. (67) H. Español de México. CDMX. (80) Inst. Mexicano del Seguro Social /
(55) Inst. Nacional de Cardiología Ignacio Chávez. (68) ISSEMYM Tlalnepantla. CDMX. H. General Regional 200. CDMX.
CDMX. (69) Fund. H. Ntra. Sra. de La Luz I.A.P. CDMX. (81) Inst. de Seg. y Serv. Soc. de los Trabajadores
(56) H. Ángeles Lomas. CDMX. del Estado de Nuevo León “ISSSTELEON”.
(70) Centro Médico Nacional de Occidente.
(57) Centro Dermatológico Dr. Ladislao de la Pascua. Guadalajara, Jalisco. (82) H. Ángeles Acoxpa. CDMX.
CDMX. (71) Depto. de Psiquiatría y Salud Mental. (83) H. Juárez de México. CDMX.
(58) H. Gral. de Zona 32, IMSS. CDMX. Facultad de Medicina UNAM. CDMX. (84) Centro Universitario de la Costa,
(59) H. San Ángel Inn Universidad. CDMX. (72) H. Universitario Dr. José E. González. U. de Guadalajara. Guadalajara, Jalisco.

(60) H. de Especialidades Centro Médico Siglo XXI Monterrey, Nuevo León. (85) H. Ángeles Metropolitano. CDMX.
Bernardo Sepúlveda. CDMX. (73) Centro Médico Nacional Siglo XXI. CDMX. (86) H. Infantil Privado, Star Médica. CDMX.
(61) H. Regional Lic. Adolf López Mateos del ISSSTE. (74) H. Médica Sur. CDMX. (87) H. Civil de Guadalajara Fray Antonio Alcalde.
CDMX. (75) Inst. Nacional de Enfermedades Respiratorias. Guadalajara, Jalisco.
(62) Servicios de Atención Psiquiátrica de la CDMX. (88) Fundación de Asistencia Privada.
Secretaria de Salud. CDMX. Conde de Valencia. CDMX.

7
ORIENTACIÓN ENARM
Importancia ENARM
Rendimiento por asignatura
Número medio de preguntas (importancia de la asignatura
(preguntas por página)
en el ENARM)

1,3 7 1,6%

9
ÍNDICE

TEMA 1 FISIOLOGÍA Y FISIOPATOLOGÍA ......................................................................................................15


1.1. Mecánica ventilatoria ............................................................................................................................. 15
1.2. Valoración del intercambio gaseoso ....................................................................................................... 20
1.3. Hipoxemia ............................................................................................................................................. 22
TEMA 2 NEOPLASIAS PULMONARES ............................................................................................................25
2.1. Nódulo pulmonar solitario ..................................................................................................................... 25
2.2. Neoplasias malignas de pulmón ............................................................................................................. 26
2.3. Neoplasias pulmonares benignas ........................................................................................................... 34
2.4. Neoplasias de pared torácica ................................................................................................................. 34
TEMA 3 ENFERMEDADES DE LA PLEURA .....................................................................................................36
3.1. Derrame pleural ..................................................................................................................................... 36
3.2. Neumotórax........................................................................................................................................... 39
3.3. Tumores pleurales .................................................................................................................................. 40
TEMA 4 ENFERMEDAD PULMONAR OBSTRUCTIVA CRÓNICA (EPOC) ..........................................................42
4.1. Concepto y anatomía patológica ........................................................................................................... 42
4.2. Características diferenciales de enfisema y bronquitis crónica ................................................................ 44
4.3. Diagnóstico............................................................................................................................................ 44
4.4. Estadificación GOLD .............................................................................................................................. 45
4.5. Tratamiento ........................................................................................................................................... 46
4.6. Comorbilidades en el paciente EPOC ..................................................................................................... 49
4.7. Pronóstico.............................................................................................................................................. 49
TEMA 5 ASMA ..............................................................................................................................................50
5.1. Tratamiento ........................................................................................................................................... 52
TEMA 6 TROMBOEMBOLISMO PULMONAR ..................................................................................................56
TEMA 7 TRASTORNOS DE LA VENTILACIÓN .................................................................................................62
7.1. Hipoventilación alveolar ......................................................................................................................... 62
7.2. Hiperventilación alveolar ........................................................................................................................ 62
7.3. Síndrome de apnea del sueño................................................................................................................ 63
TEMA 8 SÍNDROME DE DIFICULTAD RESPIRATORIA AGUDA .......................................................................65
TEMA 9 ENFERMEDADES INTERSTICIALES 1: NEUMONITIS INTERSTICIALES IDIOPÁTICAS.........................67
9.1. Fibrosis pulmonar idiopática................................................................................................................... 68
9.2. Otras neumonitis intersticiales................................................................................................................ 69
TEMA 10 ENFERMEDADES INTERSTICIALES 2: NEUMONITIS DE CAUSA CONOCIDA.....................................71
10.1. Neumonitis intersticiales por inhalación de polvos inorgánicos: neumoconiosis ...................................... 71
10.2. Neumonitis intersticiales por inhalación de polvos orgánicos .................................................................. 72
10.3. Neumonitis intersticiales asociadas a enfermedades autoinmunes sistémicas.......................................... 73
10.4. Neumonitis intersticiales por fármacos ................................................................................................... 74
TEMA 11 ENFERMEDADES INTERSTICIALES 3: NEUMONITIS ASOCIADAS
A PROCESOS NO BIEN CONOCIDOS ................................................................................................75
11.1. Sarcoidosis ............................................................................................................................................. 75
11.2. Linfangioleiomiomatosis e histocitosis X ................................................................................................. 78
11.3. Eosinofilias pulmonares .......................................................................................................................... 80
11.4. Proteinosis alveolar ................................................................................................................................ 81
TEMA 12 ENFERMEDADES DEL MEDIASTINO ................................................................................................82
12.1. Masas mediastínicas .............................................................................................................................. 82
12.2. Mediastinitis .......................................................................................................................................... 82
TEMA 13 HIPERTENSIÓN PULMONAR (HTP) ...................................................................................................84
13.1. Hipertensión arterial pulmonar idiopática ............................................................................................... 84
13.2. Tratamiento general de la HTP ............................................................................................................... 85
13.3. Enfermedad tromboembólica crónica..................................................................................................... 86
TEMA 14 BRONQUIECTASIAS .........................................................................................................................87
TEMA 15 BRONQUIOLITIS...............................................................................................................................89
TEMA 16 SÍNDROME DE HEMORRAGIA ALVEOLAR DIFUSA ..........................................................................90
16.1. Hemosiderosis pulmonar idiopática (HPI) ................................................................................................ 90
16.2. Enfermedad de Goodpasture ................................................................................................................. 90
TEMA 17 ENFERMEDAD DEL DIAFRAGMA .....................................................................................................91
17.1. Parálisis del diafragma ........................................................................................................................... 91
17.2. Hernias diafragmáticas ........................................................................................................................... 91

11
TEMA 18 TRASPLANTE PULMONAR ...............................................................................................................92
18.1. Indicaciones del trasplante de pulmón ................................................................................................... 92
18.2. Complicaciones...................................................................................................................................... 92
TEMA 19 MALFORMACIONES .........................................................................................................................93
19.1. Deformidades de la pared torácica......................................................................................................... 93
19.2. Anomalías traqueobronquiales ............................................................................................................... 93
TEMA 20 ANATOMÍA ......................................................................................................................................94
20.1. Histología .............................................................................................................................................. 94
20.2. Radiografía de tórax .............................................................................................................................. 94
TEMA 21 VENTILACIÓN MECÁNICA ...............................................................................................................96
21.1. Fundamentos ......................................................................................................................................... 96
21.2. Indicaciones generales ........................................................................................................................... 96
21.3. Indicación de las distintas modalidades .................................................................................................. 96
21.4. Modalidades de soporte ventilatorio invasivo ......................................................................................... 96
TEMA 22 SEMIOLOGÍA RESPIRATORIA...........................................................................................................98

VALORES NORMALES EN NEUMOLOGÍA Y CIRUGÍA TORÁCICA ...........................................................................99

BIBLIOGRAFÍA .......................................................................................................................................................100

RESPUESTAS ENARM ............................................................................................................................................101

12
CURIOSIDAD

El asbesto se ha utilizado durante miles de años por


sus excelentes propiedades aislantes e ignífugas. Por
ejemplo, Carlomagno tenía un mantel de fibras de
asbesto que limpiaba tras los banquetes echándolo al
fuego, y que luego volvía a sacar intacto (con lo que
asombraba a sus invitados). Sin embargo, sus riesgos
también son conocidos desde hace siglos. Ya en el
siglo I, Plinio el Viejo, describió una enfermedad de los
pulmones en los esclavos que tejían ropa de asbesto. Los
primeros trabajos científicos que relacionaron el asbesto
con el cáncer datan de 1.935, pero su uso no se prohibió
hasta la década de los ‘80.

13
Tema 1
Fisiología y fisiopatología

La principal función del aparato respiratorio es la de permitir Distinguimos cuatro volúmenes estáticos:
una adecuada oxigenación de la sangre y eliminar el dióxido de - Volumen Corriente (VC) o “Tidal Volume” (VT).
carbono (CO2) producido. Esto se logra gracias a cuatro proce- Volumen de aire que entra en los pulmones durante una res-
sos: ventilación, difusión, perfusión y distribución. El resultado piración normal (500 ml aproximadamente).
es el intercambio gaseoso. Cualquier trastorno en estos proce- - Volumen Residual (VR).
sos puede provocar hipoxemia, siendo la causa más frecuente Volumen de gas que permanece en los pulmones despues de
de la misma el trastorno en la relación ventilación-perfusión. una espiración máxima (1.200 ml aproximadamente).
- Volumen de Reserva Espiratorio (VRE).
Volumen de gas expulsado durante una maniobra espiratoria
1.1. Mecánica ventilatoria máxima después de haber eliminado el volumen corriente, o
el volumen que se puede espirar tras una espiración normal
(1.100 ml aproximadamente).
La mecánica ventilatoria es el conjunto de fuerzas capaces de
- Volumen de Reserva Inspiratorio (VRI).
vencer la resistencia que ofrecen la caja torácica y el parénquima
Volumen de gas inhalado durante una maniobra inspiratoria
pulmonar, para llevar a cabo la ventilación. Un ciclo respiratorio
máxima tras una inspiración normal (3.000 ml aproximada-
normal consta de una fase inspiratoria y una fase espiratoria. La
mente).
inspiración es un proceso activo que consume energía, porque
requiere la utilización de los músculos inspiratorios para vencer
las resistencias que se oponen a la entrada de aire en los pul- La suma de dos o más de los volúmenes anteriores se define
mones. Estos músculos son los intercostales externos, paraester- como capacidad, tenemos 4:
nales, esternocleidomastoideo, escaleno y, el más importante, el - Capacidad Pulmonar Total (CPT o TxLC).
diafragma. La espiración normal es pasiva. Cantidad de aire que contienen los pulmones cuando se ha-
Se denomina ventilación al movimiento de gas desde la llan totalmente distendidos. Es la suma de todos los volúme-
atmósfera hasta el interior de los pulmones durante la respira- nes (aproximadamente 5.800 ml).
ción. Intervienen los siguientes elementos: - Capacidad Funcional Residual (CFR).
Es el volumen de gas que permanece en los pulmones después
de una espiración normal. Es la posición de reposo del aparato
Sistema conductor: árbol traqueobronquial respiratorio (2.300 ml aproximadamente) = VR + VRE.
Existe una zona de conducción que no participa en el inter- - Capacidad Vital (CV).
cambio gaseoso, llamada espacio muerto anatómico (desde Volumen de gas expulsado durante una espiración máxima a
nariz hasta bronquiolos terminales, que consta de unos 150 partir de una inspiración máxima (4.600 ml aproximadamente)
cc). Debemos diferenciarla del espacio muerto alveolar, que = VT+VRE+VRI = CPT-VR.
se corresponde con el aire contenido en alvéolos no perfun- - Capacidad Inspiratoria (CI).
didos y que aumenta en determinadas patologías, como las Volumen de gas inspirado durante una maniobra forzada
enfermedades intersticiales, enfermedades vasculares pulmo- máxima a partir de la CFR (3.500 ml aproximadamente) = VRI
nares (TEP), etc. Se denomina espacio muerto fisiológico a + VT.
la suma del espacio muerto anatómico y del espacio muerto
alveolar. Para calcular el VR, y por tanto la CPT y la CFR, es necesario
La resistencia al paso de aire por la vía aérea depende sobre realizar la técnica de dilución de helio o la pletismografía cor-
todo de la sección transversal del conducto, por lo que la vía poral. El resto de volúmenes estáticos se pueden calcular con
aérea superior (nariz, boca, faringe, laringe y tráquea) presenta la espirometría.
la mayor resistencia, mientras la pequeña vía aérea contribuye
sólo al 10-20% de la resistencia total de la vía aérea. (Ver figura 1 en la página siguiente)

Volúmenes pulmonares Volúmenes dinámicos


Volúmenes estáticos Miden cantidad de gas en condiciones de movi-
Determinan la cantidad de aire que hay en los miento del mismo, fundamentalmente durante
pulmones, teniendo en cuenta las distintas posi- una espiración forzada. Al introducir el factor
ciones que adopta la caja torácica. Para calcular- tiempo estamos refiriéndonos a flujos (volumen/
los se utiliza la espirometría y la pletismografía. tiempo). Para calcularlos utilizamos el espirómetro.

15
Manual ENARM · Neumología y Cirugía Torácica

Recuerda que...
CPT = 5.800 Todos los volúmenes pulmonares (tanto estáticos como dinámi-
CV = 4.600
CI = 3.500 cos) dependen de la raza, edad, sexo, talla y peso del individuo y
CFR = 2.300 se considera como normal valores comprendidos entre el 80% y
CI 120% del esperado, excepto el FEF 25%-75%, que se considera
VRI = 3.000 como normal a partir del 60% del teórico.
CV
VT = 500 ml.
CPT Volumen de cierre
A volúmenes inferiores a la CFR, la presión pleural inspiratoria
puede hacerse positiva en las bases y originar un cierre distal
VRE = 1.100 de las vías aéreas, probablemente en los bronquiolos respira-
CFR torios. Este volumen aumenta con la edad, pudiendo superar
a la CFR en ancianos aparentemente sanos. En otras situacio-
nes en las que la retracción elástica del pulmón está reducida
VR = 1.200 (EPOC) puede producirse esta misma situación, de forma que
las porciones basales de los pulmones sólo son ventiladas
intermitentemente.
Figura 1. Volúmenes pulmonares estáticos.

(Ver figura 3 en la página siguiente)


- VEF1 o FEV1 (ver figura 2).
Volumen de gas espirado en el primer segundo de una espi-
ración forzada. Podemos distinguir dos conceptos teóricos: la ventilación total
- Capacidad Vital Forzada (CVF). o volumen/minuto y la ventilación alveolar. Sabiendo que la
Volumen total que el paciente espira mediante una espiración frecuencia respiratoria (FR) es 12-16 respiraciones minuto:
forzada máxima.
- Flujo mesoespiratorio. Ventilación total = volumen corriente × FR
Flujo espiratorio forzado de aire en la parte media de la espi-
ración (FEF 25%-75% o VMFM, velocidad máxima del flujo Así, la ventilación total es el volumen total de aire movilizado
mesoespiratorio). Es la medida más sensible para valorar la en un minuto.
obstrucción precoz de la pequeña vía aérea (vías de diámetro Sin embargo, la ventilación alveolar o volumen real de aire
menor a 2 mm) y suele ser la primera alteración detectada en sigue la fórmula siguiente:
fumadores. Se mide en litros/segundo.
- Relación VEF1/CVF.
Ventilación Alveolar (VA) =
Su valor normal es de 0.8. Si es menor de 0.7 es diagnóstico
(volumen corriente – espacio muerto fisiológico) × FR
de obstrucción al flujo aéreo.

De esta forma, la VA representa el volumen real de aire que


participa en el intercambio gaseoso en un minuto.
El parámetro fundamental que determina la ventilación
en un persona sana es la presión parcial de CO2 en san-
gre arterial (PaCO2).

Volumen

75%

CVF

50%
FEV1

25%

VR
Tiempo
0 1 2 3 4 5 6

Figura 2. Curva de flujo tiempo. Representa el volumen espirado en el primer segundo (FEV1).

16
Tema 1 · Fisiología y fisiopatología

Volumen

75%

CVF

50%
FEV1

25%

VR
Tiempo
0 1 2 3 4 5 6

Figura 3. Espirometría dinámica. En verde, curva normal; en rojo discontinuo, patrón restrictivo.

Curva normal Patrón Patrón Estenosis fija Obstrucción Obstrucción


obstructivo restrictivo de vía aérea variable variable
Flujo (L/s) superior extratorácica intratorácica
8 8
PEF
FEF25 6 6
Espiración FEF50
4 4
FEF75 3 3 3
Volumen 2 2
Capacidad residual
pulmonar total 0 0 0 0 0
Volumen (L)
Inspiración -2 -2
PIF -3 -3 -3
FIF50 -4 -4
-6 -6
CVF
-8 -8

Figura 4. Curvas flujo-volumen.

Elastancia la CPT y la CV. El patrón restrictivo se define por una CPT


Resistencia a la expansión que ofrecen las estructuras elásticas <80% y una relación VEF1/CVF normal (>0,8). Para su diagnós-
de pulmón y caja torácica o, dicho de otro modo, la tendencia tico es imprescindible el cálculo del volumen residual mediante
a volver a la configuración inicial cuando la fuerza inspiratoria una pletismografía o por métodos de dilución de gases.
cesa. Depende de las fibras elásticas y de la tensión superficial A su vez, dentro de las alteraciones restrictivas y en función del
del surfactante pulmonar. Está disminuida en el enfisema, por VR, podemos distinguir:
destrucción de tejido elástico. - Enfermedades con patrón restrictivo con aumento del VR.
Se trata de alteraciones extraparenquimatosas en las que,
Complianza tanto la inspiración como la espiración están afectadas. Por
lo tanto, al pulmón le cuesta tanto llenarse como vaciarse
Es la inversa de la elastancia; refleja la facilidad para la insu- de aire y por ello el VR suele aumentar. Son ejemplos típicos
flación pulmonar. Mide los cambios de volumen en relación algunas alteraciones de la caja torácica, como la espondilitis
con los cambios de presión. Está disminuida en patologías que anquilosante y enfermedades neuromusculares, como la mias-
ofrecen una resistencia a la entrada de aire, como el edema tenia gravis, Guillain-Barré, distrofias musculares, etc.
pulmonar, las intersticiales y las alteraciones esqueléticas - Enfermedades con patrón restrictivo con disminución del VR.
torácicas. Está aumentada en el enfisema, por destrucción del Se incluyen enfermedades en las que predomina la dificul-
tejido elástico. tad para llenar de aire los pulmones y el VR suele disminuir.
Dentro de éstas, encontramos las enfermedades restrictivas pa-
(Ver figura 4) renquimatosas, como la sarcoidosis, la neumoconiosis y algunas
enfermedades intersticiales, como la fibrosis pulmonar idiopá-
tica. También se incluyen alteraciones extraparenquimatosas
Patrones de función anormal con disfunción principalmente inspiratoria, como la obesidad
Restrictivo (enfermedad restrictiva más frecuente), la cifoescoliosis y en-
Existe una dificultad para llenar de aire el pulmón, por lo que fermedades neuromusculares, como la parálisis diafragmática,
todos los volúmenes pulmonares están disminuidos, sobre todo pues el diafragma es un músculo exclusivamente inspiratorio.

17
Manual ENARM · Neumología y Cirugía Torácica

Otros parámetros que tienen interés en la patología restrictiva nosticar la obstrucción). De hecho, en las obstrucciones leves
son la presión espiratoria (PEM) e inspiratoria máximas (PIM), la FEV1 es normal.
que valoran la fuerza muscular que se tiene que desarrollar Durante el ejercicio, aparece un fenómeno denominado hipe-
en una espiración o inspiración forzada ante una vía aérea rinsuflación dinámica. La taquipnea, al acortar el tiempo de
ocluida. espiración, impide el vaciado alveolar completo, con el consi-
En la enfermedad restrictiva extraparenquimatosa inspiratoria guiente aumento de VR y CFR, y descenso concomitante de la
y espiratoria, tanto el PIM como el PEM pueden ser normales CVF (aunque sin empeoramiento de la obstrucción: la FEV1 no
o estar disminuidos, mientras que en la extraparenquimatosa decae, a menos que el tiempo espiratorio se haga inferior a un
inspiratoria el PEM será normal, pero el PIM puede estar dis- segundo). La gravedad de la obstrucción se establece según la
minuido (parálisis diafragmática). Es por este motivo que los clasificación SEPAR, salvo en el caso concreto de la EPOC que
pacientes con parálisis diafragmática empeoran en decúbito, tiene su propia clasificación (GOLD).
puesto que al tener dificultad para inspirar, el aumento de la
presión abdominal sobre la caja torácica en el decúbito dificul-
ta todavía más la inspiración. NIVEL DE GRAVEDAD FEV1 (% VALOR DE REFERENCIA)
Tanto en la patología obstructiva como en la restrictiva paren-
Leve >70
quimatosa, el PIM y el PEM son normales.
Moderada 60-69

ENF. EXTRAPARENQUIMATOSA Moderadamente grave 50-59


ENF. PAREN-
QUIMATOSA INSPIRATORIA
INSPIRATORIA Grave 35-49
ESPIRATORIA
CPT ↓ Muy grave <35

VR ↓ o normal ↑ Tabla 3. Grados de alteración obstructiva según la SEPAR.

VEF1/CVF Normal o aumentado Variable


- Etiologías del patrón obstructivo:
PIM Y PEM Normales PIM puede ↓ Normales o ↓ EPOC, asma, bronquiectasias (incluyendo la fibrosis quística),
y bronquiolitis (infecciosa, inflamatoria, obliterante). Además,
algunas enfermedades intersticiales pueden presentar en
Tabla 1. Patrón restrictivo.
ocasiones patrón obstructivo: histiocitosis X, linfangioleiomio-
matosis, neumonía eosinófila crónica, neumonitis por hiper-
- Sarcoidosis sensibilidad, Churg Strauss, ABPA...
PARENQUI- - Neumoconiosis
MATOSAS - Fibrosis pulmonar idiopática Recuerda que...
(VR ↓, VEF1/CVF 0,8)
- Otras enfermedades intersticiales
FEV1 VEF1/CVF VR CPT CV
EXTRA- INSPIRATORIAS INSPIRATORIAS-
PARENQUI- (VEF1/CVF= 0,8; ESPIRATORIAS OBSTRUCTIVAS ↓↓ ↓ ↑ ↑ ↓
MATOSAS VR ↓ O N) (VEF1/CVF VARIABLE, VR ↑)
RESTRICTIVAS ↓ Nó↑ Variable ↓ ↓
ENFERMEDADES Parálisis
Miastenia gravis
NEURO- Guillain-Barré 1. El VR no se calcula con la espirometría, sino mediante
diafragmática pletismografía o la técnica de dilución del helio.
MUSCULARES Distrofias musculares
2. Toda enfermedad con ↑ VR es una enfermedad obstructiva
ALTERACIONES (con la excepción de las enfermedades restrictivas extra-
Obesidad Espondilitis
DE LA parenquimatosas con disfunción inspiratoria y espiratoria).
Cifoescoliosis anquilosante
CAJA TORÁCICA 3. Toda enfermedad con VEF1/CVF <0,7 es una enfermedad obs-
tructiva (sin excepciones).
4. El FEV1 no sirve para el diagnóstico de nada; sólo sirve para el
Tabla 2. Principales enfermedades respiratorias con patrón restrictivo. pronóstico.

Obstructivo
Patrón característico de asma, EPOC y bronquiectasias. Se Curvas flujo-volumen
caracteriza por la dificultad para el vaciamiento de los pul- Se obtienen a partir de la espirometría forzada y muestran el
mones. Por tanto, la CPT estará normal o aumentada y el VR flujo de aire (L/seg) en función del volumen pulmonar; tanto
tenderá a aumentar. A medida que aumenta el VR, la CVF durante la inspiración (desde el VR hasta la CPT) como durante
(que inicialmente es normal) tenderá a disminuir. La primera la espiración forzada (desde la CPT hasta el VR). El estudio de
alteración funcional que suele aparecer es la disminución del la morfología de estas curvas permite caracterizar los patrones
PEF25-75%. El criterio diagnóstico que define la obstrucción es obstructivo y restrictivo; pero es también muy útil para el estu-
una relación FEV1/CVF muy disminuida (<70%), que refleja la dio de las estenosis de vías aéreas superiores. La altura de las
dificultad para el vaciado rápido del pulmón. La FEV1 tenderá a curvas se relaciona con la magnitud de los flujos, y la anchura
ir disminuyendo según aumente la gravedad de la obstrucción con los volúmenes pulmonares (a menor CVF, menor anchura
(por lo que sirve para estimar la gravedad, pero no para diag- de la curva).

18
Tema 1 · Fisiología y fisiopatología

- Curva normal. sión en el parénquima pulmonar, aumentando la frecuencia


Representa la espiración por encima del eje horizontal, con un respiratoria (reflejo de Hering-Breuer; ej. edema pulmonar,
pico de flujo espiratorio (PEF) muy precoz y mayor que el pico tromboembolismo...).
de flujo inspiratorio (PIF), que se representa por debajo del eje
horizontal y ocurre hacia la mitad de la inspiración. En resumen, el patrón ventilatorio puede verse modificado: por
- Patrón obstructivo. causas centrales (voluntad, depresores o estimulantes del SNC,
Se afectan sobre todo los flujos espiratorios (que pierden enfermedades neurológicas); por cambios en el pH arterial, la
mucha altura), con PEF<PIF, concavidad hacia arriba en la PaCO2 o la PaCO2; o por estimulación de los receptores intra-
curva espiratoria y pérdida de anchura de la curva (por au- pulmonares (embolia pulmonar, neumonía, asma).
mento del VR).
- Patrón restrictivo. Perfusión pulmonar
La altura de las curvas es prácticamente normal (pues los flujos
están preservados), pero están notablemente estrechadas (lo El pulmón presenta un doble sistema de vascularización,
que refleja la disminución del volumen pulmonar: disminuye por lo que la isquemia pulmonar es rara:
la distancia entre CPT y VR). - Circulación bronquial.
- Estenosis fija de vía superior (p. ej., estenosis traqueal, En torno al 1% del gasto cardiaco. Sale del corazón izquierdo
bocio). (aorta → arterias intercostales → arterias bronquiales) pero no
Las curvas están aplanadas en meseta tanto en inspiración tiene retorno venoso sistémico, sino que vuelve al corazón iz-
como en espiración (formando una morfología en “rectán- quierdo por las venas pulmonares (formando parte del shunt
gulo”) con igualamiento de PEF = PIF, lo que refleja el máximo fisiológico). Su función es nutrir las paredes bronquiales. Ex-
flujo que puede atravesar la estenosis, tanto en inspiración cepto en algunas enfermedades (bronquiectasias, insuficiencia
como en espiración. ventricular izquierda), su trascendencia funcional es escasa. De
- Obstrucciones variables (sólo afectan a una parte del ciclo hecho, la función pulmonar se mantiene intacta incluso en au-
respiratorio). sencia completa de circulación bronquial (trasplante pulmonar).
• Extratorácicas (p. ej., parálisis de cuerda vocal). - Circulación pulmonar.
La obstrucción aparecerá exclusivamente con la inspiración, Conectada en serie con la circulación sistémica, interviene de
por lo que la curva espiratoria es normal y en la inspiratoria manera directa en el intercambio de gases. Es la más impor-
surgirá una meseta de flujo. tante cuanti- y cualitativamente. Sale del corazón derecho
• Intratorácicas (p. ej., traqueomalacia). portando sangre venosa y retorna al corazón izquierdo sangre
Aparecerá obstrucción sólo con la espiración, por lo que la oxigenada. Tiene características propias:
curva espiratoria tendrá morfología obstructiva pero conser- • Circuito de alto volumen pero de baja presión.
vando una anchura normal (VR conservado). La curva inspi- La circulación pulmonar mantiene presiones muy bajas (unos
ratoria es normal. 25 mmHg en la sístole y 8 mmHg en la diástole, con una
PAP media de 14 mmHg), para lograr equilibrarse con las
presiones de la vía aérea. Esto se logra gracias a unas resis-
Control de la ventilación
tencias vasculares mínimas, que apenas se modifican frente
1. Control voluntario. a aumentos notables del gasto cardiaco. Ello se debe a la
Las conexiones neuronales entre el centro respiratorio y la cor- capacidad de distensión capilar, y al reclutamiento de nuevos
teza cerebral permiten el control voluntario de la ventilación. territorios capilares que normalmente no están perfundidos.
2. Centros respiratorios involuntarios troncoencefálicos. De esta forma, se reduce el trabajo del ventrículo derecho
Núcleos inspiratorio y espiratorio (en la sustancia reticular necesario para mantener una perfusión capilar constante.
del bulbo) y centros neumotáxico y apnéustico (puente). El • Respuesta vasoconstrictora a la hipoxia.
centro neumotáxico regula la duración de la inspiración (su Otra particularidad pulmonar es que ante la hipoxia se pro-
estimulación causa taquipnea, y su lesión la llamada respira- duce vasoconstricción, para impedir que zonas mal ventila-
ción apnéustica, con inspiración prolongada). das sean perfundidas. Así, se redistribuye el flujo sanguíneo
El estímulo ventilatorio directo a nivel bulbar es la acidez del hacia zonas bien oxigenadas. Este mecanismo es eficaz para
LCR, que a su vez es inducida por la hipercapnia; por ello, conservar la relación ventilación/perfusión (V/Q) ante una
la PaCO2 se considera el principal estímulo ventilatorio. Sin lesión pulmonar aguda, pero si se perpetúa en el tiempo
embargo, en los pacientes con retención crónica de CO2 (p. ej. (como ocurre en las neumopatías crónicas), se desarrollan
EPOC), el pH del LCR tiende a tamponarse con bicarbonato; lo cambios proliferativos que conducen a una hipertensión pul-
que reduce la sensibilidad a los incrementos de CO2 arterial. monar irreversible.
3. Quimiorreceptores arteriales (aórticos y en bifurcación
carotídea).
Se definen tres zonas pulmonares según el flujo sanguíneo:
Se localizan principalmente en el seno carotídeo (conducción
aferente de estímulos a través del nervio glosofaríngeo) y - Zona 3:
en el cuerpo aórtico (conducción a través del nervio vago), Perfusión continua (presión diastólica > presión alveolar).
muy sensibles al descenso de PaO2 (su mayor estímulo es la - Zona 2:
hipoxemia). Este sistema pasa a ser principal en los retene- Perfusión intermitente (presión sistólica > presión alveolar >
dores crónicos de CO2 (con escasa sensibilidad a la PaCO2), presión diastólica).
por lo que en estos pacientes se debe evitar el empleo de - Zona 1 (patológica):
altos flujos de oxígeno suplementario para no inhibir el estí- Flujo nulo (presión alveolar > presión sistólica).
mulo ventilatorio que supone la hipoxemia (de lo contrario
se corre el riesgo de agravar la hipercapnia e incluso inducir En bipedestación, lo normal es que los pulmones sólo tengan
acidosis y coma). zonas 2 y 3 de flujo: zona 2 en los vértices y 3 en las bases. En
4. Mecanorreceptores pulmonares. decúbito lo normal es que el flujo sanguíneo sea en su totali-
Responden a estímulos de estiramiento/aumento de pre- dad de zona 3.

19
Manual ENARM · Neumología y Cirugía Torácica

Adecuación ventilación/perfusión En las fases iniciales de la insuficiencia cardiaca congestiva,


Para un correcto intercambio gaseoso es necesario que los debido al elevado volumen sanguíneo capilar, aumentará la
alveolos bien ventilados estén además bien perfundidos, es DLCO; sin embargo, en fases más tardías, el edema intersticial
decir (relación V/Q 1). Cualquier discordancia en un sentido y alveolar dificultarán la difusión, reduciendo la DLCO.
u otro causará ineficiencia en el intercambio gaseoso (lo que se En el tromboembolismo pulmonar y la hipertensión pulmonar,
traducirá en hipoxemia ± hipercapnia). Las alteraciones extre- dado que disminuyen tanto el volumen capilar pulmonar como
mas posibles son las siguientes: la superficie total de capilares pulmonares, la DLCO disminuye.
- Unidad de espacio muerto.
Zona alveolar bien ventilada pero no perfundida. La relación
V/Q tiende a infinito. La poca sangre que atraviese estas uni- DLCO DESCENDIDA DLCO AUMENTADA
dades tendrá valores de PO2 y PCO2 similares a las del aire
inspirado. - Enfisema - Fases iniciales de insuficiencia
- Unidad de shunt (cortocircuito). - Enfermedades intersticiales cardiaca congestiva
Zona alveolar bien perfundida pero no ventilada. La relación - TEP - Hemorragia alveolar
V/Q tiende a cero. La composición de la sangre que sale de - Anemia (falsamente - Poliglobulia (falsamente
dicha unidad será similar a la de la sangre venosa que entró descendido) aumentado)
en el capilar pulmonar. - Hipertensión pulmonar - Embarazo
- Unidad silente. - Edema pulmonar - Asma bronquial
Unidad alveolar no perfundida ni ventilada.
Tabla 4. Alteraciones de la DLCO.

Recuerda que...
Recuerda que, aunque tanto la ventilación como la perfusión son 1.2. Valoración del intercambio gaseoso
mayores en las bases, la relación ventilación/perfusión es mayor en
los vértices, por lo que la sangre que sale de los vértices sale mejor
oxigenada que la que sale de las bases. El intercambio gaseoso depende de la ventilación, de la perfu-
sión, de la relación V/Q y de la difusión. Los valores gasométri-
cos normales varían de acuerdo a la altura y presión baromé-
trica, sin embargo, los valores de referencia más utilizados son
Difusión calculados a nivel del mar. Por lo que, mediante la gasometría
Paso de gases a través de la membrana alveolocapilar. El CO2 podemos diferenciar diversas situaciones patológicas.
tiene una capacidad de difusión 20 veces mayor que el O2, La eficacia de este intercambio se valora mediante la dife-
por lo que ante un fallo respiratorio primero disminuye la rencia o gradiente alveoloarterial de O2: D(A-a)O2. Se
PaO2, y posteriormente aumenta la PaCO2. En condiciones considera como normal un gradiente menor de 15 mmHg
normales, en un sujeto en reposo, el intercambio gaseoso en jóvenes, si bien en ancianos puede llegar a superar los 25
tiene lugar en el tercio inicial del recorrido capilar, quedando mmHg (pero un gradiente >30 mmHg es siempre patológico).
los dos tercios restantes como reserva funcional (que se utiliza Para calcular el gradiente utilizamos la siguiente fórmula:
cuando aumenta el gasto cardiaco, como en el ejercicio). Sin
embargo, en las patologías con afectación de la difusión ocurre
que ya en reposo precisan de todo el recorrido capilar para D(A-a)O2 = PAO2 − PaO2
mantener el intercambio gaseoso (quedándose por tanto sin
reserva funcional); es por ello que en dichas patologías es típi- PAO2 = FiO2 · (Pb − PH2O) − (PaCO2 · R)
co encontrar normoxemia en reposo pero hipoxemia inducible
con el ejercicio.
- PAO2: presión alveolar de O2.
Para cuantificar la capacidad de difusión se emplea la difu-
- FiO2: fracción de O2 en el aire inspirado (0,21).
sión pulmonar medida con monóxido de carbono (DLCO).
- Pb: presión barométrica ambiental.
Depende de:
- PH2O: presión parcial de vapor de agua del aire inspirado (47
- La superficie y grosor de la membrana alveolocapilar. mmHg).
La causa más frecuente de disminución de la DLCO es la dis- - PaCO2: presión arterial de CO2.
minución de superficie de intercambio, típica del enfisema - R: cociente respiratorio, o relación entre la producción de CO2
pulmonar, seguida del engrosamiento de la membrana al- y el O2 consumido (0,8 en dietas normales).
veolo-capilar (como ocurre en las enfermedades intersticiales).
- Adecuación de la relación ventilación-perfusión. Asumiendo condiciones atmosféricas estándar (760 mmHg de
- Concentración de Hb en sangre y volumen de sangre en los Pb a nivel del mar), se puede simplificar la ecuación:
capilares pulmonares.
Ya que el CO tiene 220 veces mayor afinidad por la Hb que el
PAO2 = 0,21 · (760 − 47) − (PaCO2 / 0,8) = 150 − (PaCO2 / 1,25)
O2, la DLCO es muy sensible a incrementos o disminuciones
en la cantidad de Hb pulmonar:
• En las hemorragias alveolares (Wegener, PAN microscópica, D(A-a)O2 = 150 − (PaCO2 · 1,25) − PaO2
LES, hemosiderosis pulmonar idiopática, síndrome de Goo-
dpasture, etc.) existe un aumento de la DLCO.
Un gradiente alveolocapilar normal implica el adecuado aco-
• La anemia disminuye la DLCO; lo contrario ocurre en la poli-
plamiento de ventilación, perfusión y difusión; es decir, implica
globulia.
que la función pulmonar está preservada (gradiente normal =
pulmón normal). Un gradiente aumentado supone alteración

20
Tema 1 · Fisiología y fisiopatología

de al menos uno (o más) de los anteriores, es decir, implica que


la función pulmonar está alterada, a cualquier nivel (gradiente SaO2%
aumentado = pulmón dañado). 100%
Para valorar el intercambio gaseoso, podemos utilizar la gaso-
metría arterial y la pulsioximetría. Mediante la gasometría
podemos diferenciar diversas situaciones patológicas:
75%
- Hipoxemia: PaO2 <80 mmHg (con FiO2 0,21). Tª
- Hipercapnia: PaCO2 >45 mmHg. PaCO2
- Insuficiencia respiratoria parcial o hipoxémica: PaO2 <60 P50
P50
[H+]
mmHg (con FiO2 0,21). 50%
- Insuficiencia respiratoria global: PaO2 <60 mmHg y PaCO2 2,3-DPG
>45 mmHg (con FiO2 0,21).

25%
La gasometría también permite conocer el pH (acidosis respi-
ratoria: pH <7.35 y PCO2 >45 mmHg; alcalosis respiratoria: pH Afinidad: Liberación O2 a los tejidos
>7.45 y PCO2 <35 mmHg), el bicarbonato, el exceso de bases
y el gradiente alveoloarterial de oxígeno. PaO2
27
Es importante asegurar que la muestra de sangre es arterial y 20 40 60 80 100
no venosa. Ante un paciente clínicamente asintomático que
presenta una PO2 40 mmHg y/o una PCO2 46 mmHg debe- Figura 5. Curva de disociación de la hemoglobina.
mos sospechar que la extracción no se ha realizado correcta-
mente y que se corresponde con datos de sangre venosa.

Valoración de la oxigenación
El mejor parámetro que evalúa el estado de oxigenación del - La acidosis (↑ H+ y ↓ pH –efecto Bohr–).
organismo es el contenido arterial de O2. Para calcularlo se - El aumento de la temperatura.
utiliza la fórmula: - El aumento del 2-3 difosfoglicerato (DPG).
- El aumento de la PaCO2.
Contenido arterial de O2 = O2 disuelto + O2 unido a Hb
Desplazan la curva hacia la izquierda los contrarios: el aumen-
to del pH, el descenso de la PaCO2, el descenso del DPG y el
Tan sólo el 3% del total del O2 va disuelto en el plasma (0,0031 descenso de la temperatura.
ml de O2 por decilitro de sangre). El O2 se transporta funda-
Se denomina P50 al valor de PaO2 para el que la hemoglobina
mentalmente combinado junto a la hemoglobina, formando la
se encuentra saturada al 50%; en condiciones normales es
oxihemoglobina, de forma que en 1 g de oxiHb hay 1,34 ml de
cercano a 27 mmHg. En el tratamiento de la insuficiencia
O2. El% de saturación (SaO2) se puede extraer de la curva de
respiratoria, la oxigenoterapia intenta conseguir una PaO2
disociación de la hemoglobina conociendo la PaO2:
>60 mmHg y una SaO2 90%, que asegura un aporte sufi-
ciente de oxígeno a los tejidos. Si con flujos bajos de oxígeno
Contenido arterial de O2 = (0,0031 · PaO2) + (1,34 · [Hb] · SaO2) suplementario no aumentamos la PaO2, habrá que aumentar
el aporte de oxígeno vigilando la ventilación (PaCO2) y el nivel
de conciencia. Si no hay una respuesta adecuada, habrá que
La SaO2 es el mejor parámetro para valorar el transporte de
pasar a la ventilación mecánica para mantener un pH normal y
oxígeno. Se puede obtener de forma directa o calculada en
una PaO2 >60 mmHg.
una gasometría arterial; o medir de forma no invasiva median-
te la pulsioximetría.
El trasporte de CO2 es diferente. Un 7% del CO2 va disuelto
en plasma. El resto se distribuye del siguiente modo: 70% en
Recuerda que...
forma de anión bicarbonato y un 20-30% en forma de carba-
minohemoglobina. Efecto Bohr
Aumento de la liberación de oxígeno a los tejidos cuando el dióxi-
do de carbono y los hidrogeniones desplazan la curva de disocia-
Curva de disociación de la hemoglobina ción de la oxihemoglobina a la derecha. Esto determina
La SaO2 de la hemoglobina se relaciona con la PaO2 mediante que cuando la sangre alcanza los capilares tisulares, el dióxido de
la curva de disociación de la hemoglobina; en ella se obser- carbono que penetra en la sangre procedente de los tejidos des-
va cómo en ambientes ricos en oxígeno (p. ej., circulación plaza la curva a la derecha, lo que suelta oxígeno de la
alveolar) la Hb se satura al máximo de O2, y cómo en aquellos hemoglobina y por tanto se libera más oxígeno a los tejidos.
territorios pobres en O2 (tejidos con alta demanda de O2) la
Hb pierde afinidad por el O2, cediéndolo a los tejidos. Además, Efecto Haldane
diversos factores desplazan la curva a la derecha o izquierda La unión del oxígeno a la hemoglobina reduce su afinidad por el
haciendo que disminuya o aumente aún más la afinidad de la dióxido de carbono. Este efecto determina que en los pulmones se
hemoglobina por el oxígeno (ver figura 5). Los factores que produzca un aumento de la liberación del dióxido de carbono al
desplazan la curva hacia la derecha (por tanto aumentan la captar oxígeno la hemoglobina.
cesión tisular de O2) son:

21
Manual ENARM · Neumología y Cirugía Torácica

1.3. Hipoxemia • Sin embargo, en muchos gasómetros lo que se hace es una


estimación de SaO2 a partir del pH, la PaO2, y la curva de
disociación de Hb estándar. Dado que la PaO2 se mantiene
La hipoxia consiste en el déficit de aprovechamiento de O2 por
normal, muchos gasómetros informarán una SaO2 calcu-
los tejidos. Existen distintas causas de hipoxia (ver tabla 5), y
lada normal.
la hipoxemia es sólo una de ellas.
• La pulsioximetría no es un método fiable, pues la carboxiHb
se confunde con la oxiHb en la mayoría de pulsioxímetros
antiguos, dando una falsa normalidad de SatO2.
CAUSA EJEMPLO
El diagnóstico de intoxicación por CO se establece por la medi-
HIPOXÉMICA Disminución de la PO2 EPOC ción directa de los niveles de carboxiHb (gasometría arterial
con co-oximetría).
Disminución del Anemia
ANÉMICA contenido de O2 Intoxicación por CO
Mecanismos causales de hipoxemia (ver tabla 6)
Disminución del Shock
CIRCULATORIA 1. Hipoxia ambiental.
caudal de O2 Obstrucción local
El paciente respira un aire pobre en oxígeno (escapes de gas,
Disminución de hacinamiento, grandes alturas). La mecánica ventilatoria y el
Intoxicación por intercambio gaseoso son adecuados, luego cursa con PaCO2
CITOTÓXICA la utilización celular
cianuros
de O2 y D(A-a)O2 normales.
2. Hipoventilación.
La mecánica ventilatoria está disminuida, limitando la reno-
Tabla 5. Causas de hipoxia.
vación del aire alveolar. La PaCO2 estará aumentada.
• Extrapulmonar, con D(A-a)O2 normal (intoxicación por
depresores, enfermedades neuromusculares).
Se define como hipoxemia el descenso de O2 en la sangre, • Pulmonar, con alteración del intercambio gaseoso (D(A-a)
definido como una PaO2 <80 mmHg. Un grado mayor es la O2 aumentada): neumonía con fatiga muscular, EPOC
insuficiencia respiratoria hipoxémica, con PaO2 <60 mmHg. con hipoventilación....
La PaO2 es el mejor parámetro para evaluar el intercambio En estos casos asociará un mecanismo adicional de los
pulmonar de gases. siguientes.
La intoxicación por CO es una situación excepcional, pues 3. Discordancia ventilación/perfusión (V/Q).
cursa con disminución del contenido arterial de O2 con Un incorrecto acoplamiento limita el correcto intercambio
PaO2 normal: gaseoso. Es el mecanismo más frecuente de hipoxemia
- La PaO2 suele ser normal; puesto que el intercambio ga- (EPOC, asma, TEP, enfermedades alveolares, infección respi-
seoso a nivel pulmonar se mantiene conservado. ratoria…). Corrige con oxigenoterapia, y provoca hipoxemia
- El CO convierte la hemoglobina reducida en carboxihemog- de reposo.
lobina (HbCO), que es incapaz de transportar O2 (por lo que 4. Alteración de la difusión.
disminuyen los gramos totales de oxiHb, siendo ésta la causa Limita el intercambio gaseoso; sólo provoca hipoxemia de
de la disminución del contenido arterial de O2). esfuerzo dado que en reposo el flujo sanguíneo pulmonar
Fisiopatológicamente, la intoxicación por CO se asemeja a una lento permite que el intercambio gaseoso, aunque dismi-
hipoxia anémica (por disminución de oxiHb). nuido, sea suficiente. Cursa con D(A-a)O2 aumentada y
- Hay que evaluar la SaO2 con prudencia: corrige con oxigenoterapia. Se ve en el enfisema y las en-
• La SaO2, medida directamente en una gasometría arterial fermedades intersticiales. La única enfermedad en la que la
con co-oximetría, (calculando los mg/dl totales de oxiHb y de alteración de la difusión provoca hipoxemia en reposo sin
Hb total), reflejará la disminución real que ocurre en la SaO2 otro mecanismo asociado es la fibrosis pulmonar idiopática.
medida.

¿CORRIGE CON ¿HIPOXEMIA DE


PaCO2 D(A-a)O2
OXIGENOTERAPIA? REPOSO?
HIPOXIA Normal Normal Sí Sí

HIPOVENTILACIÓN ↑ Normal Sí Sí
(AISLADA)

EFECTO SHUNT Normal / ↑ ↑ No Sí

ALTERACIÓN V/Q Normal / ↑ ↑ Sí Sí

ALTERACIÓN Normal / ↑ ↑ Sí No
DIFUSIÓN

Tabla 6. Resumen de los mecanismos causales de hipoxemia.

22
Tema 1 · Fisiología y fisiopatología

5. Efecto shunt. Recuerda que...


Clínicamente se define como una hipoxemia refractaria al
tratamiento con O2 a alto flujo. Es debido a la presencia de La insuficiencia respiratoria es un diagnóstico gasométrico; se da
alveolos perfundidos pero no ventilados (relación V/Q que cuando a una FiO2 ambiente (21%), existe una PaO2 menor de
tiende a 0), por lo que la sangre que sale de esa unidad 60 mmHg (tipo I o hipoxémica). Cuando la PaCO2 es mayor de 50
alveolar será muy parecida a la sangre venosa que llegó al mmHg, se denomina insuficiencia respiratoria tipo II (hipercápnica).
capilar pulmonar, pasando así de cavidades derechas a iz-
quierdas sin oxigenarse. La D(A-a)O2 está aumentada. Aun-
que la administración de oxígeno no corrige la hipoxemia, PO2 <80 mmHg
en la práctica clínica se emplea oxígeno como tratamiento,
para que ayude a elevar la PaO2 según se vaya resolviendo ¿PCO2?
el proceso.
Causas del efecto shunt:
• Ocupación alveolar (edema agudo pulmonar, síndrome PCO2 PCO2 n ó
de distrés respiratorio del adulto, neumonía extensa).
• Atelectasia (o colapso alveolar). Hipoventila No hipoventila
• Cortocircuitos vasculares:
- Pulmonares: adquiridos, como la cirrosis, o heredita- ¿Aa O2? ¿Aa O2?
rios, como la enfermedad de Rendu-Osler.
- Extrapulmonares: cardiopatías cianógenas (shunt dere- Normal Aumentado Normal
cha-izquierda).
Hipoventilación Patología Hipoxia ambiental
parenquimatosa
Algoritmo diagnóstico del mecanismo causal de hipoxemia
Para identificar el mecanismo responsable ante un paciente ¿Responde O2?
con hipoxemia, lo primero que debemos mirar es la PaCO2:
1. PaCO2 aumentada.
El mecanismo responsable más probable será la hipoven- No Sí
tilación. Debe calcularse a continuación la D(A-a)O2: si es
normal existe hipoventilación aislada; si está aumentada, Efecto Shunt Alteración V/Q Alteración difusión
pensaremos en la existencia de un mecanismo adicional
(shunt, alteración de la ventilación-perfusión, etc.) que con- Figura 6. Algoritmo resumen de las causas de hipoxemia.
tribuya a la hipoxemia. Una excepción es la obesidad, donde
la hipoxemia por hipoventilación aislada puede acompañarse
de aumento de la D(A-a)O2 en las bases pulmonares. Mecanismos de corrección de hipoxemia
2. PaCO2 normal o disminuida. La hipoxemia desencadena una serie de mecanismos compen-
Debe evaluarse la D(A-a)O2 sadores en el organismo.
• D(A-a)O2 normal (<15 mmHg). Los mecanismos inmediatos son:
Hipoxia ambiental. Mejorará con oxígeno.
• D(A-a)O2 aumentada (>15 mmHg). - Taquipnea.
Debemos evaluar la respuesta a la administración de oxí- Aumenta el aporte de aire a los alveolos.
geno a alto flujo: - Taquicardia.
- Corrige la hipoxemia. La causa más habitual será la al- Aumenta la sangre que pasa por la circulación pulmonar para
teración de la relación ventilación/perfusión (V/Q), oxigenarse.
especialmente si hay hipoxemia de reposo, aunque si - Vasodilatación local periférica.
la hipoxemia es tan sólo con el ejercicio sospechare- Mayor aporte de sangre a los tejidos hipóxicos.
mos alteración de la difusión. - Vasoconstricción local pulmonar.
- No corrige la hipoxemia. La causa es un efecto shunt. Las zonas con peor intercambio gaseoso sufren mayor vaso-
constricción, distribuyendo la sangre hacia las áreas con mejor
intercambio.
Recuerda que...
A medio plazo:
Hipoventilación es sinónimo de hipercapnia.
- Aumento de la eritropoyesis, que aumenta la capacidad de
transporte de oxígeno a tejidos.
- Aumento de la eliminación renal de bicarbonato (para compen-
sar la alcalosis respiratoria inducida por la hiperventilación).
VALORES NORMALES

pH 7,35-7,45 Síntomas de hipoxemia


La hipoxia aguda provoca, por descenso del aporte de oxíge-
HCO3 22-28 mEq/l
no al SNC, inicialmente un estado de nerviosismo o ansiedad,
PCO2 35-45 mmHg seguido de euforia y posteriormente somnolencia. Aparece
fatigabilidad muscular, torpeza motora y mental, náuseas y
PO2 >80 mmHg vómitos, y, si la hemoglobina reducida supera 5 g/dl, cianosis
central, que es un dato de gravedad. Si además hay hiper-
capnia aparecerá la encefalopatía hipercápnica (bradipsiquia,
Tabla 7. Valores normales de la gasometría arterial.

23
Manual ENARM · Neumología y Cirugía Torácica

bradilalia, somnolencia y flapping tremor), que puede conducir Recuerda que...


a un coma hipercápnico.
Del tema de Fisiología y Fisiopatología,
La hipoxemia crónica presenta los mismos síntomas pero de
los conceptos más importantes son:
forma más larvada. Además, la hipoxia crónica conduce a
1. Mejor indicador del estado de ventilación: PaCO2 (↑ ventilación
debilidad muscular y osteopenia, por activación inflamatoria.
→ ↓ PaCO2).
Incluso puede aparecer a largo plazo un cuadro caquéctico y
2. Mejor indicador del estado de oxigenación: contenido arterial de
anemia de enfermedad crónica. Por otra parte, pueden apare-
O2.
cer síntomas derivados de los mecanismos de compensación:
3. Mejor indicador del transporte arterial de O2: SaO2.
- Hiperviscosidad. 4. Mejor indicador del intercambio pulmonar de gases: D(A-a)O2.
Por el aumento de eritropoyesis. 5. Para el seguimiento y pronóstico de las enfermedades obstructi-
- Cefalea. vas se utiliza el FEV1, mientras que en las enfermedades restric-
Por vasodilatación cerebral o por hipercapnia. tivas se usa la DLCO.
- Hipertensión pulmonar y cor pulmonale crónico. 6. El enfisema ↑ la distensibilidad y ↓ la DLCO.
Por la vasoconstricción pulmonar. 7. El principal factor que genera hipertensión pulmonar es la hi-
- Arritmias auriculares. poxemia: cualquier enfermedad que produzca hipoxemia a la
Por dilatación de aurícula derecha secundaria a hipertensión larga puede producir hipertensión pulmonar y cor pulmonale.
pulmonar. 8. Ante una hipoxemia, para diferenciar entre efecto shunt y alte-
ración de la relación ventilación/perfusión, se debe realizar una
gasometría con FiO2 100% y valorar si existe respuesta a la ad-
ministración de oxígeno.

Pregunta ENARM Pregunta ENARM


1. Un paciente fumador presenta en la espirome- 2. La afinidad de la hemoglobina por el oxígeno
tría forzada un volumen espiratorio forzado en disminuye al:
el primer segundo (FEV1) del 31% del predicho,
una capacidad vital forzada (FVC) del 80% del A. Disminuir la PCO2.
predicho y una relación de FEV1/FVC del 40%. B. Disminuir la concentración de hidrogeniones.
Su saturación por oximetría es 88%. ¿Cuál de las C. Aumentar la concentración de 2,3 difosfoglicerato.
siguientes opciones define obstrucción del flujo D. Aumentar la concentración de bicarbonato.
aéreo?

A. FVC.
B. FEV1.
C. FEV1/FVC.
D. Saturación.

· Encuentra las respuestas y comentarios de las preguntas ENARM al final del manual ·

24
Tema 2
Neoplasias pulmonares

2.1. Nódulo pulmonar solitario resección para biopsia bien mediante VATS (video-assisted
thoracoscopic surgery), mínimamente invasiva y con baja mor-
bimortalidad, o si resultara inaccesible mediante toracotomía.
Se define nódulo pulmonar solitario (NPS) a una radiopacidad
bien definida de 0.8 a 3 cm de diámetro rodeada de parén- - Si la probabilidad de malignidad es intermedia o hay discordan-
quima pulmonar normal o enfermo (enfisema pulmonar). Se cia entre la probabilidad clínica y radiológica se pueden plantear
denomina tumor a cualquier opacidad de más de 3 cm; dado distintas alternativas. Básicamente tendremos tres opciones:
su alto riesgo de malignidad, el planteamiento ante una masa • PET.
debe ser más agresivo, debiendo obtenerse una biopsia, gene- Dado su alto coste y baja disponibilidad, sólo se realizará si
ralmente mediante resección. siendo la probabilidad clínica baja o intermedia tiene una TC
sospechosa, o si siendo probabilidad alta la TC no muestra
Ante un nódulo pulmonar solitario, son datos de benignidad
signos de malignidad. La positividad del PET obliga a realizar
la coexistencia de:
directamente una resección (puede dar falsos positivos por
- Edad menor a 35 años. infección, sarcoidosis, nódulos reumatoides). Si es negativo,
- Paciente no fumador. se puede valorar una punción transtorácica con aguja fina
- Algún criterio radiológico de benignidad. (pues puede dar falsos negativos por tumores de baja tasa
• Tamaño menor de 2 cm o ausencia de crecimiento a los 2 metabólica o por hiperglucemia) o asumir benignidad.
años del diagnóstico (es importante al diagnóstico comparar • La punción transtorácica con aguja fina.
la imagen con radiografías previas del paciente, si las hu- Es una prueba fundamental en los pacientes con probabili-
biera). dad clínica intermedia, especialmente útiles para los nódulos
• Tiempo de duplicación menor a 20 o mayor a 400 días. periféricos.
• Presencia de lesiones satélites o imágenes radiográficas de • La fibrobroncoscopia con toma de biopsia.
calcificaciones en palomita de maíz (hamartoma), en ojo de Es una técnica que requiere guía por ultrasonido o fluoros-
buey (granuloma), nido central denso o múltiples focos pun- copía para nódulos periféricos. Se realiza sólo en centros al-
tiformes. tamente especializados.

Figura 1. Calcificación en forma de “palomita de maíz” de los hamartomas. Figura 2. Nódulo pulmonar solitario: radiografía de tórax.

Si reúne criterios de benignidad el manejo es conservador: se


vigilará con radiografías o TC trimestrales durante el primer
año y, posteriormente una prueba de imagen anual, gene-
ralmente hasta el quinto año; si se produce variación del
nódulo se procederá a su diagnóstico anatomopatológico, o
directamente se realizará resección (dependiendo del grado de
sospecha de malignidad).
Si el paciente no reúne criterios de benignidad, se debe proce-
der en función de la probabilidad de malignidad:
- Si la probabilidad de malignidad es baja (<5%), debe reali-
zarse seguimiento radiológico estrecho.
- Si el paciente presenta alta probabilidad clínica y radiológica
de malignidad (>65%), se debería proceder directamente a la Figura 3. Nódulo pulmonar solitario: TC del mismo paciente.

25
Manual ENARM · Neumología y Cirugía Torácica

Recuerda que... Existen otros carcinógenos pulmonares como el asbesto, que


presenta un papel sinérgico con el tabaco en el desarrollo del
Todo NPS es maligno mientras no se demuestre lo contrario. cáncer de pulmón (RR = 6 en exposición a asbesto y RR =
59 en exposición a asbesto + tabaco), sobre todo con fibras
En todo paciente con un NPS de reciente diagnóstico de anfibolita; también son carcinógenos el radón, el humo
se debe realizar un TC torácico con contraste. de leña (la exposición por más de 50 años se ha relacionado
con adenocarcinoma en mujeres mexicanas no fumadoras),
la tuberculosis (puede aumentar el riesgo hasta 11 veces), el
humo de motores diésel, arsénico, sílice, ácido crómico, éter,
- Realizar CT tórax clorometilo, níquel, hidrocarburos policíclicos aromáticos, la
Nódulo pulmonar - Comparar con Rx previas
contaminación ambiental y la desnutrición.
En las células tumorales se encuentran frecuentemente lesio-
¿Criterios benignidad? nes genéticas adquiridas. Así, en los carcinomas de células no
- <35 años pequeñas (CNCP) pueden encontrarse mutaciones del EGFR
- No fumador
- Imagen radiológica benigna: (Epidermal Grow Factor Receptor), o traslocaciones del gen
duplicación <1 mes; ALK (Anaplastic Lymphoma Kinase) o ROS-1 (c-ROS oncogene
no crecimiento en 2 años; 1). El EGFR forma parte de una red de señalización que regula
calcificacion palomita de maíz varios procesos celulares críticos, como el crecimiento, la proli-
(hamartoma), ojo de buey
(granuloma), nido central, feración y la motilidad celulares. Las mutaciones del EGFR, ALK
puntiforme; lesiones satélites y ROS1 son más prevalentes en no fumadores (la del EGFR,
especialmente en asiáticos), y confieren mejor pronóstico por
su buena respuesta a fármacos biológicos. En todos los pacien-
tes con CNCP se deben estudiar la mutación del gen EGFR y
Sí: benigno No: sospecha maligno ALK (y se recomienda del ROS1) porque pueden beneficiarse
¿Riesgo de malignidad?
de tratamientos específicos. Otras mutaciones de menor tras-
cendencia clínica (pues no hay fármaco dirigido a ellas) afectan
al K-RAS, Her-2 o BRAF. Por otra parte, tanto los CNCP como
Seguimiento Intermedio o discordancia carcinomas de células pequeñas (CCP u “oat cell”) pueden
radiológico Bajo Alto
clinicorradiológica
presentar mutaciones en el ongogen c-myc.
Por último, la expresión de PD-L1 (Programmed Death-Ligand 1)
PET tumoral es predictora de la respuesta a ciertas inmunoterapias.
- Si periférico:
punción transtorácica Resección:
- Si central: videotoracoscopia
fibrobroncoscopia _ + o toracotomía Clínica


crecimiento Recuerda que...
De acuerdo a la GPC, la tos es un síntoma presente al
Figura 4. Manejo del nódulo pulmonar solitario. momento del diagnóstico en más del 65% de los pacientes
con cáncer pulmonar.

2.2. Neoplasias malignas de pulmón


El cáncer de pulmón puede producir síntomas locales (que
Generalidades dependerán de si la localización del tumor es central o periféri-
ca), así como manifestaciones por invasión de órganos vecinos,
Su máxima incidencia ronda los 60 años. por afectación metastásica linfática o a distancia, y síntomas
La mayoría de tumores pulmonares son primarios y malig- paraneoplásicos.
nos. El pulmón, no obstante, es también uno de los principales En el momento del diagnóstico, un 20% de los pacientes tiene
lugares de aparición de metástasis a distancia. enfermedad localizada, un 25% afectación linfática regional y
El tipo histológico más frecuente tanto en México como en el un 55% metástasis (casi un 95% en el de células pequeñas, que
mundo hasta hace unos años era el epidermoide, pero actual- es el que con más frecuencia metastatiza por vía sanguínea).
mente es el adenocarcinoma, tanto en fumadores (se cree En cuanto a los síntomas locales, los tumores que se pre-
que por el cambio a cigarrillos bajos en alquitrán en los años sentan con localización central (carcinoma epidermoide y
60-70) como en no fumadores. La incidencia en mujeres se ha microcítico) dan clínica por obstrucción bronquial: cursan con
igualado a la de varones. tos productiva (lo más frecuente) con/sin hemoptisis (ante
todo fumador mayor de 40 años con aumento de la tos y/o
Etiología expectoración hemoptoica se debe realizar una broncoscopia
aunque la radiografía de tórax sea normal). Asimismo, cuando
Se ha demostrado que existe una relación entre el número de
obstruyen completamente la luz bronquial pueden ocasionar
cigarrillos consumidos y todos los tipos de cánceres pulmona-
atelectasia (la neoplasia pulmonar es la causa más frecuente
res. La relación es dosis dependiente; el riesgo relativo (RR) tras
de colapso obstructivo) y, si se infecta el parénquima pulmonar
haber consumido 40 paquetes-año es de 20. El riesgo de desa-
distal, producen neumonías de repetición en el mismo lóbulo.
rrollar un cáncer de pulmón en los exfumadores se aproxima al
La atelectasia de un bronquio principal asociada a derrame
de los no fumadores (aunque no llega a igualarse); al cabo de
pleural maligno (típico del subtipo epidermoide) puede dar
cada 10 años disminuye a RR = 0,6 (respecto a un fumador).
lugar a una imagen radiológica de pulmón blanco ipsilateral

26
Tema 2 · Neoplasias pulmonares

sin desviación del mediastino, mientras que si hay derrame


o atelectasia masivos aislados sí aparece desviación del medias-
tino, ipsilateral en la atelectasia y contralateral en el derrame.
El carcinoma anaplásico de células grandes y el adenocarcinoma
suelen presentarse como una masa periférica y suelen ser asin-
tomáticos hasta fases avanzadas; la clínica local deriva de irrita-
ción pleural, cursan con dolor pleurítico, tos irritativa y disnea.
Todos ellos pueden invadir estructuras vecinas y presentarse
con un cuadro de afectación regional que curse con disfonía
(por infiltración del nervio laríngeo recurrente), enfermedad
pleural con/sin derrame (por extensión pleural), elevación
de hemidiafragma (por infiltración del frénico), síndrome de
Horner, síndrome de vena cava superior, arritmias e insufi-
ciencia cardiaca (por infiltración miocárdica), taponamiento
cardiaco (por infiltración pericárdica), etc.
La afectación linfática regional consiste en adenopatías y,
en ocasiones, linfangitis carcinomatosa, que cursa con disnea
e infiltrados intersticiales.
Las acropaquias pueden deberse a distintas patologías: enfer-
medades pulmonares (80-85% de las acropaquias), enferme-
Figura 5. Metástasis cerebral de un CCP.
dades digestivas (cirrosis, enfermedad inflamatoria intestinal),
cardiopatías cianógenas, endocarditis y algunos tumores.
Dentro de las enfermedades pulmonares, lo más frecuente es
el cáncer de pulmón (habitualmente no microcítico y metastá-
sico), y su aparición en un paciente fumador obliga a descartar
esta entidad. También se pueden observar acropaquias en los
pacientes con bronquiectasias, abscesos pulmonares, meso-
telioma y neumopatías intersticiales, así como por causas no
pulmonares como cardiopatías cianógenas.
Las regiones donde con más frecuencia metastatiza el carci-
noma pulmonar son hígado, hueso, suprarrenales (sobre todo
los CNCP) y cerebro (sobre todo los CCP). Es rara la disemina-
ción hematógena al pulmón contralateral.

(Ver figuras 5, 6 y 7)

A continuación se exponen las características más importantes


de cada tumor:
Figura 6. Metástasis hepáticas de cáncer de pulmón.

Carcinoma epidermoide o escamoso


Actualmente el segundo en prevalencia, en el siglo XX fue el Periférico Centrales proximales
cáncer de pulmón más frecuente en México y en el mundo.
Tipos: Tipos:
Afecta sobre todo a lóbulos superiores y suele presentarse - Adenocarcinoma - Epidermoide
como una masa central. En el momento del diagnóstico - Anaplásico de - Anaplásico de
presenta metástasis extratorácicas en el 50% de los casos, a células grandes células pequeñas
pesar de lo cual es el tipo histológico de mejor pronóstico. Clínica: Clínica:
Histológicamente, se caracteriza por la formación de puentes - Asintomático - Tos
intercelulares y/o queratina (globo o perla córnea, pero puede - Dolor pleurítico - Hemoptisis
faltar en subtipos muy indiferenciados). Es la causa más fre- - Derrame pleural - Atelectasia: causa
maligno más frecuente de
cuente de masa maligna cavitada (cavitan el 20%) y de colapso obstructivo
síndrome de Pancoast. El síndrome de Pancoast aparece en - Neumonía:
tumores que asientan en los vértices pulmonares, fundamen- recidivante, no
resolución con ATB
talmente carcinomas epidermoides, que producen destrucción
de la primera y segunda costillas así como afectación de las
raíces nerviosas C8, T1 y T2. Cursa con dolor localizado en
hombro-brazo, especialmente de la región cubital, y/o atrofia
de músculos de la mano (por afectación del plexo braquial),
así como síndrome de Horner (por afectación del simpático
cervicotorácico) que consiste en miosis, ptosis, enoftalmos,
y, en ocasiones, trastornos de la sudoración facial (anhidrosis
ipsilateral). Este tipo de tumor es el que con más frecuencia Biopsia transbronquial Biopsia bronquial
puede secretar una sustancia PTH-like, pudiendo producir
hipercalcemia e hipofosfatemia. Figura 7. Clínica según la localización del tumor. ©3drenderings / AdobeStock.

27
Manual ENARM · Neumología y Cirugía Torácica

En los subtipos de adenocarcinoma invasivo mucoso y lepídico


(antiguo bronquiolo-alveolar) las células neoplásicas se dispo-
nen dentro de los alveolos y tienden a diseminar por vía endo-
bronquial, sin infiltración del parénquima adyacente. Pueden
presentarse como un nódulo periférico o como un infiltrado
parenquimatoso intersticial en la radiografía del tórax.

Carcinoma anaplásico de células pequeñas (CCP), en


grano de avena, microcítico u “oat cell”
Radiológicamente es típica la presencia de una masa central
y adenopatías. Es el tumor que más frecuentemente secreta
Figura 8. Síndrome de Horner provocado por un tumor de Pancoast. hormonas-péptidos, por lo que es el que con más frecuencia
produce síndromes paraneoplásicos:
- Hiponatremia por secreción inadecuada de ADH o por secre-
ción del péptido atrial natriurético.
- Hipopotasemia por secreción ectópica de ACTH.
- Es la causa más frecuente de síndrome de vena cava su-
perior (junto con los linfomas), del síndrome de Eaton-Lam-
bert, de la ceguera retiniana, de la degeneración cerebelosa
subaguda y de la disfunción cortical paraneoplásica.

Presenta metástasis extratorácicas en el 95% de los casos. Se


considera diseminado al diagnóstico casi por definición.
En el 90% de los CCP se encuentra la delección 3p.

Figura 9. Carcinoma escamoso de pulmón de tipo queratinizante.

Adenocarcinoma
Es la causa más frecuente de nódulo solitario maligno, pero
puede presentarse como una imagen sólida o únicamente
como vidrio despulido.
Actualmente es el más prevalente en general, y siempre ha
sido el más frecuente entre los no fumadores. Es el tumor que
característicamente asienta sobre las cicatrices pulmonares
crónicas, como son las cicatrices tuberculosas, y cuando así Figura 10. Anatomía patológica de carcinoma microcítico de pulmón.
ocurre se llama "scar cancer". Suele presentarse como una
masa periférica bien delimitada y no suele cavitarse. Es la
Carcinoma anaplásico de células grandes
causa más frecuente de nódulo pulmonar solitario malig-
no. Característicamente afecta a la pleura, siendo la causa Es el subtipo con más tendencia a cavitarse después del car-
más frecuente de derrame pleural maligno. Puede produ- cinoma epidermoide. Suele presentarse como una masa peri-
cir osteoartropatía hipertrófica, que consiste en la presencia férica o nódulo. Presenta metástasis extratorácicas en el 80%
de acropaquias junto con periostitis y que cursa con dolor y de los casos. Como síndrome paraneoplásico puede producir
tumefacción. Presenta metástasis extratorácicas en el 80% de ginecomastia.
los casos. Dentro de los CNCP es el que tiene más tendencia a
metastatizar vía hematógena. Tumores pulmonares metastásicos
En el examen no es frecuente que se profundice en la clasifica- Clínicamente, los tumores pulmonares metastásicos cursan
ción de los adenocarcinomas, pero sí que se ha nombrado en más frecuentemente con linfangitis carcinomatosa que los
alguna ocasión el concepto de “adenocarcinoma bronquiolo- primarios.
alveolar” que en la actualidad está en desuso. Los adenocar-
cinomas se clasifican en acinar, papilar, invasivo muco- La enfermedad metastásica pulmonar suele ser incurable, pero
so (antiguo bronquiolo-alveolar mucoso), lepídico (antiguo en ocasiones es posible extirpar metástasis pulmonares únicas
bronquiolo-alveolar no mucoso), micropapilar y sólido. Esta o múltiples con intención curativa. Para ello se deben cumplir
clasificación es importante pues el adenocarcinoma de predo- estas condiciones:
minio lepídico tiene mejor pronóstico (supervivencia 95% a - Tumor primario tratado y sin evidencia de enfermedad resi-
los 5 años) que el de predominio papilar y acinar. El subtipo dual.
micropapilar es el de peor pronóstico. - Ausencia de metástasis extrapulmonares.

28
Tema 2 · Neoplasias pulmonares

- Todas las metástasis quedan incluidas en la resección prevista. Diagnóstico, técnicas de estadificación y estadiaje
- El paciente puede tolerar la resección pulmonar que se con- Tanto la historia clínica como la exploración física son dos
templa. aspectos fundamentales a tener en cuenta en la evaluación del
paciente con sospecha de cáncer de pulmón.
El tumor primario con mejores tasas de éxito de la metastasec- Las personas en quienes se deben realizar estudios de tamizaje
tomía pulmonar es el osteosarcoma. de acuerdo con la GPC, son:
- Edad entre 55 y 74 años, fumadores activos o que dejaron de
fumar en los últimos 15 años y que tengan índice tabáquico
30 paquetes/año.
- Edad 50 años, antecedente de índice tabáquico 20 paque-
tes/año y con otros factores de riesgo para cáncer pulmonar.
- Presencia de factores de riesgo diferentes del tabaquismo, con
o sin síntomas respiratorios o pérdida de peso involuntaria,
independientemente de la edad.

En estos grupos de sospecha clínica de cáncer de pulmón,


anteriormente se sugería realizar una radiografía de tórax
como paso inicial. No obstante, en la actualidad la GPC
recomienda solicitar de primera instancia una tomografía
de tórax simple, helicoidal y de dosis baja como estudio de
tamizaje, debido a la sensibilidad y especificidad superiores
y demostración de menor mortalidad por mayor número de
detecciones. Deberá realizarse cada año durante al menos 3
años. Actualmente tampoco se recomienda realizar citología
de esputo como prueba de cribado para cáncer pulmonar.
Como la mayor parte de tumores se desarrollan en la vía aérea
Figura 11. Tumor pulmonar central. principal, tanto en tomografía de tórax como en radiografías
de tórax que resulten con datos positivos, los signos radiográ-
ficos más frecuentes van a ser el ensanchamiento hiliar (por el
Regla mnemotécnica tumor o por la presencia de adenopatías mediastínicas) y la
atelectasia. La obstrucción de un bronquio principal o lobar
El carcinoma ePidermoide se asocia a
provoca una imagen de atelectasia parcial o total, que se aso-
la secreción de PTH y al tumor de Pancoast
cia con frecuencia a una condensación distal (denominada
signo de Golden).
El aDEnocarcinoma produce DErrame pleural
El diagnóstico específico (histológico) de los
El carcinoma de células en aVENA (oat cell) se asocia a tumores malignos en la mayoría de casos es
síndrome de VENA cava superior obtenido a partir de la fibrobroncoscopia.
Para que la rentabilidad diagnóstica sea la
El carcinoma de células GRANDES se asocia a adecuada se deben realizar 3-4 biopsias y
mamas GRANDES (ginecomastia) combinarlo con el correspondiente estudio citológico del bron-
coaspirado. En el caso de los tumores centrales bastará con

LOCALIZACIÓN DATOS CARACTERÍSTICOS

- Es el que se cavita con mayor frecuencia


EPIDERMOIDE - Síndrome de Pancoast
- PTH-like (hipercalcemia)
Central (atelectasia) - El de peor pronóstico
ANAPLÁSICO DE CÉLULAS - Síndrome de vena cava superior
PEQUEÑAS U “OAT CELL” - Síndromes paraneoplásicos
- Degeneración cerebelosa subaguda

- El más frecuente (tanto en fumadores como no fumadores)


- Más frecuente sobre áreas cicatriciales (TBC)
ADENOCARCINOMA - Afecta a pleura
- Alta tasa de metástasis hematógenas
Periférico - Osteoartropatía hipertrófica

ANAPLÁSICO DE - Cavita 20%


CÉLULAS GRANDES - Ginecomastia

Tabla 1. Características principales de los tumores pulmonares.

29
Manual ENARM · Neumología y Cirugía Torácica

la realización de una biopsia bronquial, mientras que en los


periféricos y de tamaño <3 cm, la rentabilidad diagnóstica será Estudio ganglionar
menor, por lo que deberá realizarse una biopsia transbron-
quial o incluso una punción transtorácica o transparietal. Tumor LSI o
Una vez diagnosticado, es necesario establecer la extensión Medio Mediastinoscopia
Broncoscopia bronquio principal izquierdo
tumoral, encaminada a descartar la afectación mediastínica o ecoguiada
la presencia de metástasis a distancia que contraindiquen la - Paratraqueales
cirugía con intención curativa. - Subcarinales
- Paratraqueales
Para llevar a cabo la evaluación de la extensión tumoral intrato- - Subcarinales
rácica, la técnica de elección es la TC torácica que permite eva- Mediastinotomía
luar la presencia de adenopatías mediastínicas. Como norma,
Anterior Mediastinotomía
se considera que la presencia de adenopatías mediastínicas izquierdo - Preaórtico
mayores a >1 cm de diámetro en la TC obliga a confirmar su - Ventana
naturaleza neoplásica mediante histología por exploración qui- - Prevasculares aortopulmonar
(preaórticos)
rúrgica del mediastino (mediastinoscopia, mediastinotomía - Ventana
o toracoscopia) o mediante broncoscopia ecoguiada, antes aortopulmonar Mediastinotomía
de plantearse la exéresis tumoral.
La exploración quirúrgica está contraindicada cuando exista Toracoscopia
enfermedad neoplásica diseminada más allá del mediastino,
y cuando el enfermo no pueda tolerar una futura cirugía de
- Aortopulmonar
resección pulmonar (inoperable). Está indicada ante la exis- - Paratraqueales
tencia de adenopatías patológicas en la TC torácica en los derechos
CNCP, y en los carcinomas de células pequeñas en estadio - Paraesofágicos
- Ligamento
I. En ausencia de adenopatías en la TC torácica (N0) se debe pulmonar
valorar la realización de exploración quirúrgica en los tumores
hiliares o en el tumor de Pancoast; en el resto de casos hay dos
opciones: proceder a cirugía con intención curativa (asumiendo Figura 12. Técnicas quirúrgicas para el estadiaje ganglionar.
el N0), o valorar si precisa exploración quirúrgica mediante PET-
TC. La técnica de exploración quirúrgica del mediastino más
La resonancia magnética es superior a la TC en las siguientes
habitual es la mediastinoscopia.
situaciones:
- La mediastinoscopia.
- Adenopatías en la región subcarinal y/o ventana aortopulmonar.
Es la técnica de elección en la evaluación de las adenopatías
- Afección de la vena cava superior, arterias pulmonares y corazón.
paratraqueales derechas, traqueo-bronquiales proximales y
- Evaluación de los tumores apicales (tumor de Pancoast) pues
subcarinales. Esta técnica es útil para ver el mediastino medio.
delimita mejor la invasión del plexo braquial, arteria subclavia
De su uso pueden surgir diversas complicaciones, de las que
y canal medular.
las más frecuentes son la parálisis del nervio recurrente iz-
quierdo y la infección de la herida quirúrgica y, las más graves,
la mediastinitis y la hemorragia. Las adenopatías situadas en la La gammagrafía ósea está indicada si hay dolores óseos u
ventana aortopulmonar y las cadenas preaórticas son inacce- otros datos indicativos de metástasis óseas (hipercalcemia, ↑
sibles a la mediastinoscopia y requieren de mediastinotomía fosfatasa alcalina…)
anterior o toracoscopia. La TC craneal está indicada cuando existe sintomatología neu-
- Ultrasonido endobronquial. rológica y en el microcítico de pulmón.
Consiste en la realización de una broncoscopia
y punción transbronquial o transtraqueal de las
Estadiaje TNM: carcinoma no microcítico (CCNP)
adenopatías guiada mediante ecografía. Se trata
(8.ª edición 2017)
de una técnica en auge en los últimos años, con
una sensibilidad del 87%, y con muchas menos En el año 2017 entró en vigor la octava edición del TNM del
complicaciones que la mediastinoscopia. Es útil cáncer de pulmón, basada en datos actualizados de super-
para el estudio de las adenopatías paratraqueales bilaterales, vivencia, en los que se observa una gran concordancia entre
traqueobronquiales, subcarinales e hiliares. En el caso de un tamaño tumoral y pronóstico.
resultado negativo en la punción mediante ecobroncoscopia - T (tumor).
se debe realizar una mediastinoscopia, pues sigue siendo el • T0.
“gold standard”. No hay evidencia de tumor.
- La mediastinotomía. • Tx.
Se indica en carcinomas del lóbulo superior izquierdo y en los Células tumorales (en esputo o muestra del lavado bron-
del bronquio principal izquierdo. Esta técnica es útil para ver coalveolar) sin que el tumor sea visible por imagen ni por
el mediastino anterior. fibroscopia.
- La toracoscopia. • Tis.
Está indicada cuando se observan en la TC torácica adenopatías Cáncer in situ.
paratraqueales derechas, aortopulmonares, paraesofágicas y en - Tis (AIS): Adenocarcinoma.
el ligamento pulmonar y en el estudio del derrame pleural. - Tis (SCIS): Carcinoma de células escamosas.

30
Tema 2 · Neoplasias pulmonares

• T1. • N2.
Tumor 3 cm (diámetro mayor) rodeado de parénquima o Afectación de ganglios mediastínicos ipsilaterales y/o sub-
pleura visceral, sin evidencia de invasión más allá del bron- carinales. N2a si solo afecta una estación ganglionar, N2b si
quio lobar. Se subdivide en T1a ( 1 cm), T1b (1,1-2 cm) y T1c afecta más de una.
(2,1-3 cm). • N3.
• T2. Cualquier otro: afectación de ganglios hiliares o mediastí-
Se considera T2 con cualquiera de las siguientes características: nicos contralaterales; ganglios escalenos o supraclaviculares
- Tumor >3 cm y 5 cm. (ipsi- o contralaterales).
Se subdivide en T2a (3,1-4 cm), y T2b (4,1-5 cm).
- Cualquier tamaño si produce:
• Invasión de pleura visceral, sin invasión de la parietal. N1: Peribronquiales y/o hiliares ipsilaterales
• Atelectasia y/o neumonitis postobstructiva lobar o total.
• Invasión del bronquio principal sin invadir la carina. N2: Mediastínicos ipsilaterales y/o subcarinales
• T3.
Se considera T3 con cualquiera de las siguientes características: N3: Mediastínicos o hiliares contralaterales; escalenos o
- Tumor >5 cm y 7 cm. supraclaviculares ipsi o contralaterales
- Cualquier tamaño con invasión de pleura parietal pero sin
invasión de estructuras viscerales o vitales. Ejemplos:
• Pared torácica (típico de Pancoast).
• Pleura mediastínica.
• Pericardio parietal.
• Nervio frénico.
- Nódulos múltiples en un mismo lóbulo pulmonar.
• T4. Ca pulmón
Se considera T4 con cualquiera de las siguientes características:
- Tumor >7 cm.
- Cualquier tamaño con invasión de pleura parietal y de es-
tructuras viscerales o vitales, tales como:
• Diafragma.
• Mediastino: corazón, grandes vasos, esófago, tráquea.
• Cuerpo vertebral.
• Nervio recurrente. Son T4 la infiltración del nervio re-
currente y el síndrome de vena cava superior, cuando
son producidos por invasión directa tumoral (pueden NX: No se puede valorar los ganglios regionales
producirse también por compresión por metástasis gan- N0: Sin metástasis ganglionares regionales
glionares).
- Nódulos múltiples en distintos lóbulos pulmonares ipsilate-
rales. Figura 13. Estadificación ganglionar del cáncer de pulmón.

Regla mnemotécnica - M (metástasis).


• Mx.
T3: “PAPÁ ESQUIZOFRÉNICO”
No se puede valorar la presencia de metástasis.
Pleura y pericardio PArietales,
• M0.
PAred torácica (tumor de Pancoast),
Sin metástasis.
Nervio FRÉ/NI/CO (3 sílabas)
• M1.
Enfermedad metastásica. Se subclasifica en:
T4: “DIME QUE LOS 4 RECURRIERON A LA CAVA”
- M1a. Metástasis intratorácica, definida como nódulos pul-
DIafragma,
monares contralaterales, derrame pleural o implantes pleu-
Pleura MEdiastínica
rales.
Nervio RE/CU/RREN/TE (4 sílabas)
- M1b. Metástasis a distancia única extratorácica.
Síndrome de vena CAVA superior
- M1c. Metástasis a distancia múltiples extratorácicas.

(Ver figura 14 en la página siguiente)


- N (ganglios).
• Nx.
Los ganglios regionales no pueden ser evaluados. Estadificación del carcinoma microcítico (CCP)
• N0. En el CCP clásicamente se ha usado una estadificación simpli-
No hay afectación ganglionar (para poder afirmar que es- ficada, suficiente para el manejo terapéutico. No obstante, se
tamos ante un estadio N0 debemos biopsiar seis o más recomienda usar el TNM a efectos de estudios y seguimiento.
ganglios). La estadificación simplificada sería:
• N1. - Enfermedad limitada (30%).
Afectación de ganglios peribronquiales o hiliares ipsilatera- Enfermedad confinada en el hemitórax ipsilateral a la lesión
les. N1a si solo afecta una estación ganglionar, N1b si afecta que se puede abarcar en un campo de radioterapia. Puede ser
más de una. cualquier T (excepto con multiples nódulos en un hemitórax
que no pueden abarcarse en un campo para la RT), N1/N2, M0.

31
Manual ENARM · Neumología y Cirugía Torácica

T2a - Criterios generales.


T1 T3 T4
T2b Enfermedades asociadas graves e incontrolables. Mal estado
clínico (Índice de Karnofsky 40%).
Ib
N0 Ia IIb IIIa - Criterios cardiológicos.
IIa IAM en los 3 meses previos, arritmia ventricular no controlable.
- Criterios neumológicos.
N1 IIb IIb IIIa IIIa El prioritario es el FEV1:
• FEV1 >2000 ml o >80% del teórico:
Paciente operable.
N2 IIIa IIIa IIIb IIIb
• FEV1 <2000 ml o <80% del teórico.

N3 IIIb IIIb IIIc IIIc Complicaciones posquirúrgicas


Las principales complicaciones posquirúrgicas inmediatas tras
T2a la realización de una lobectomía o neumectomía para el trata-
T1 T3 T4 miento del cáncer de pulmón son:
T2b
Cx Cx ¿QT + - Derivadas de la propia intervención.
N0 Cx RT neo + Cx + QT Hemorragia, insuficiencia respiratoria, fístula bronquial, ate-
Cx + QT Cx? lectasia, neumotórax, torsión lobar, enfisema subcutáneo e
infecciones (del pulmón, líquido pleural o herida quirúrgica).
N1 Cx + QT Cx + QT Cx + QT Cx + QT - Derivadas del tratamiento anestésico.
Hipoventilación, insuficiencia respiratoria, atelectasia, acú-
N2 QT + RT
mulo de secreciones…
QT neo + Cx QT neo + Cx QT neo + Cx
- Derivadas de las comorbilidades.
Broncoespasmo, TEP, edema de pulmón, IAM, arritmias…
N3 QT ± RT QT ± RT QT ± RT QT + RT
Tratamiento del carcinoma no microcítico de pulmón
(CNCP)
Cx = cirugía
Carcinoma in situ
Figura 14. Nuevo estadiaje del CNCP y terapias de primera línea. Podemos optar por la resección conservadora.

- Enfermedad diseminada (70%).


Estadios I-IIIa
Enfermedad que supera los límites de lo radiable, incluyendo
metástasis a distancia, afectación del hemitórax contralateral, Estos estadios son en general candidatos a cirugía con inten-
derrame pericárdico o pleural, o múltiples nódulos en un he- ción curativa. La extensión de la resección dependerá de la
mitórax no abarcables. Son por tanto enfermedad diseminada localización del tumor, siendo lo más habitual la lobectomía,
todos los M1, las adenopatías N3 y algunos T4. pero posibles tanto resecciones más amplias (bilobectomía,
neumectomía) como más conservadoras (segmentectomía).
Cirugía en el cáncer de pulmón Si el paciente es inoperable, se puede usar quimioterapia y
radioterapia concomitante.
La mejor opción terapéutica en el cáncer de pulmón, especial-
mente en el no microcítico (CNCP) donde es el tratamiento En este grupo de tumores lo principal es la cirugía, pero en
de elección, es la exéresis quirúrgica. Para poder proceder a casos seleccionados del estadio IIb, la quimioterapia (QT) y
la cirugía, se debe confirmar la resecabilidad del tumor y la radioterapia (RT) concurrentes, preoperatorias (neoadyuvan-
operabilidad del paciente: tes), son fundamentales.
- Estadio I.
Cirugía sin QT. En el estadio Ia, la QT está contraindicada. En
Resecabilidad
el estadio Ib, la adición de quimioterapia podría considerarse
Es la posibilidad de conseguir una extirpación completa; depen- en pacientes seleccionados (aquellos con factores de muy mal
de de características del tumor (su localización y extensión). pronóstico). En pacientes no candidatos a cirugía, el trata-
Indican irresecabilidad las siguientes situaciones: miento alternativo es con radioterapia.
- CNCP estadio IV (aunque en ocasiones concretas como metás- - Estadio II.
tasis cerebral única se plantea la resección) Cirugía + QT adyuvante postoperatoria, que ha demostrado
- CNCP estadio IIIb. mejorar la supervivencia (solo en pacientes completamente re-
- CNCP estadio IIIa-T4, en algunos casos con afectación de ór- secados y con buen estado funcional). Hay casos seleccionados
ganos nobles. donde se realiza cirugía aislada. Específicamente en pacientes
- Carcinoma microcítico en estadio >I. IIb – T3N0M0 del sulcus superior, el manejo de 1.ª elección es
QT + RT concurrentes neoadyuvantes + cirugía. En este estadio,
el manejo alternativo es la radioterapia, con o sin quimioterapia.
Criterios de resección quirúrgica - Estadio IIIa-T3N1 y T4N0-N1.
Los criterios de operabilidad definen el riesgo de mortalidad Cirugía + QT adyuvante postoperatoria. La QT mejora las po-
perioperatoria del paciente, no la tolerancia funcional tras sibilidades de remisión completa y aumenta la supervivencia
la cirugía. Así, la operabilidad depende de características del (solo se otorga a pacientes completamente resecados y con
paciente. Hay criterios de inoperabilidad generales, cardioló- buen estado funcional). Como terapia de 2.ª línea, se puede
gicos y neumológicos. El principal criterio de operabilidad se considerar en pacientes seleccionados QT (con o sin radiote-
basa en el FEV1. rapia) neoadyuvante + cirugía.

32
Tema 2 · Neoplasias pulmonares

- Estadio IIIa-N2.
QT neoadyuvante, y en el caso de que el estadio N baje a N0/ Sospecha clinicorradiológica
N1, se realizará posteriormente cirugía.
CT Tórax – extensión local
Fibrobroncoscopia / PTT – confirmación histológica
Estadios IIIb-IV
Los estadios IIIb y IV son no resecables (recuerda que un N3,
Resecabilidad: estudio extensión
independientemente del T, siempre es una contraindicación - Adenopatias: PET, mediastinoscopia
absoluta para la cirugía). El tratamiento en estadio IIIb consiste - Metástasis: TAC craneal, TAC abdomen, gammagrafía
en quimiorradioterapia (en casos seleccionados T4N2 se puede
ofrecer alternativamente QT + RT neoadyuvante + cirugía). En
el estadio IV (cualquier T y N, M1) el manejo está basado en Resecable: Irresecable:
quimioterapia (con o sin radioterapia) combinada con fármacos estudio operabilidad quimiorradioterapia
biológicos, o en algunos casos biológicos en monoterapia. La
quimiorradioterapia combinada mejoraba la supervivencia en FEV1 preoperatorio con tratamiento
un año respecto a la RT exclusiva, pero la terapia biológica broncodilatador completo
puede llegar a aumentar en más de 12-18 meses adicionales.
Los regímenes de QT con cisplatino o carboplatino mejoran
la supervivencia y la calidad de vida de los pacientes. El trata- >80% <80%
miento de cáncer de pulmón avanzado implica la determina-
ción de marcadores biológicos (mutación de EGFR, mutación FEV1 postoperatorio
de PD-L1).
- Paciente sin mutación de EGFR o con mutación PD-L1:
Dar inmunoterapia contra PD-L1 con pembrolizumab y QT con >40% 30-40% <30%
platinos.
- Pacientes con mutación EGFR: Valorar otros criterios
Gefitinib. Operable (consumo O2, DLCO) Inoperable
para decidir
operabilidad
Recuerda que...
Figura 15. Algoritmo diagnosticoterapéutico del CNCP.
Estadios I-IIIa:
Cirugía con intención curativa
Enfermedad localizada
(IIIaN2: QT neoadyuvante y cirugía sólo si ↓ N)
En algunos casos excepcionales en los que el tumor se diag-
Estadios IIIb-IV: nostica muy precozmente (equivalente al estadio I del TNM) se
QT + RT con intención paliativa puede aplicar cirugía + QT posterior. Si al extirpar los ganglios
mediastínicos durante la intervención se detecta malignidad en
alguno de ellos se añadirá RT.
En el resto de casos, el tratamiento de elección es QT + RT
Otras terapias en el tratamiento del CNCP
coadyuvante sobre el tumor y los ganglios afectados. Si se
La radioterapia local ayuda al control de los síntomas locales. observa remisión con el tratamiento, se debe realizar RT holo-
Se puede aplicar radioterapia paliativa ante metástasis óseas craneal profiláctica.
dolorosas, compresión medular, disnea, parálisis de cuerda
vocal, o taponamiento cardiaco por derrame pericárdico
tumoral. Enfermedad diseminada
En el tumor de Pancoast está indicada la cirugía precedida El tratamiento de elección es la QT con intención paliativa
de RT neoadyuvante. Si el paciente rechaza la cirugía, una asociada a un tratamiento de soporte. Si se observa remisión
alternativa válida sería el tratamiento exclusivamente con RT también se debe realizar RT holocraneal profilactica.
local, que en ausencia de afectación ganglionar (T3N0M0,
estadio IIb) presenta supervivencias sólo ligeramente inferiores
a los pacientes sometidos también a cirugía.
Recuerda que...
La radioterapia también está indicada como tratamiento Enfermedad localizada:
adyuvante ante resecciones incompletas, con márgenes de QT + RT (estadio I: cirugía + QT)
resección afectos, para disminuir la tasa de recidivas locales.
Enfermedad diseminada:
En caso de derrame pleural maligno, se procederá a su drenaje
QT
y, si recidiva, se realizará pleurodesis. El tratamiento de elec-
ción en el síndrome de vena cava superior se basa en diuréti-
Si remisión completa:
cos, corticoides y radioterapia locorregional.
RT holocraneal profiláctica
(Ver figura 15)

Tratamiento del carcinoma microcítico de pulmón (CCP) Pronóstico


El carcinoma microcítico es por historia natural una enferme- El cáncer de pulmón es la principal causa de muerte por cáncer.
dad sistémica. Por este motivo la base del tratamiento es la La estirpe de peor pronóstico es el microcítico por su rápida
QT, tanto en la enfermedad limitada como en la extendida.

33
Manual ENARM · Neumología y Cirugía Torácica

capacidad de metastatizar (la supervivencia a los 5 años en Hamartoma


estadios localizados es del 31%, mientras que en estadios Es un tumor derivado de tejido pulmonar normal, que suele
extendidos no supera el 2%). debutar en mayores de 60 años, de localización periférica.
Para el resto de tumores (CNCP) el tamaño del tumor primario Suele ser silente. No precisa tratamiento. Radiográficamente
influye en la supervivencia en estadio I, mientras que el número presenta una imagen patognomónica que es la “calcificación
de ganglios afectos (N1) es el factor pronóstico más importante en palomita de maíz”. Si ésta no aparece, se debe proceder
en el estadio II. La supervivencia a los 5 años oscila entre el a la resección quirúrgica para descartar un posible origen
95% en estadios IA a tan solo el 1% en estadios IV. maligno.

Recuerda que...
De acuerdo con la GPC, las únicas medidas recomendadas para
disminuir la probabilidad de cáncer de pulmón, además de evitar la
exposición a carcinógenos, son: realizar actividad física de modera-
da a alta intensidad, el consumo de frutas y verduras (flavonoides) y
de té negro o verde elaborado a partir de Camellia sinensis.

2.3. Neoplasias pulmonares benignas

Representan <5% de los tumores pulmonares primarios. Los


tipos más frecuentes son el carcinoide (el más frecuente) y los
hamartomas.

Carcinoide bronquial
Se consideran parte del espectro de tumores neuroendocrinos,
cuya peor expresión es el carcinoma microcítico de pulmón.
Aunque también puede presentarse como un tumor periférico,
en el 80% de los casos el carcinoide suele ser de localización Figura 16. Hamartoma pulmonar. Calcificaciones en palomita de maíz.
central, y suele cursar con tos crónica, hemoptisis (ya que está
muy vascularizado), o atelectasia. Ante un paciente <40 años,
no fumador, que presenta una tumoración pulmonar central
2.4. Neoplasias de pared torácica
deberemos sospechar un tumor carcinoide.

La mayoría de tumores de pared torácica son primarios,


Recuerda que... benignos, y de estirpe condral. Suelen tener crecimiento
Carcinoide, Central, y < Cuarenta años local o extratorácico; cuando crecen hacia la cavidad torácica
generan imágenes de ángulos obtusos, y pueden dar patología
ventilatoria restrictiva extraparenquimatosa.
El carcinoide es un tumor derivado de las células de Kultchitzsky
del sistema APUD, que presentan gránulos neuroendocrinos y Tumores benignos
secretan sustancias vasoactivas, por lo que puede presentar - Osteocondroma. 30-50% de las lesiones benignas.
síntomas característicos (pero poco frecuentes) de síndrome Asienta en metáfisis anterior de costilla y forma una masa
carcinoide: rubor facial, hipotensión, fiebre, diarrea y vómitos. pedunculada indolora. Bajo riesgo de malignización (<1%).
En el carcinoide bronquial el síndrome carcinoide puede apare- - Displasia fibrosa. 20-30%.
cer sin metástasis hepáticas, ya que las sustancias vasoactivas Asienta en arco costal posterior como lesión dolorosa o frac-
se secretan directamente a la circulación sistémica (en cambio, tura patológica. Radiológicamente es lesión lítica. Típica de
los carcinoides del tracto digestivo necesitan de la presencia de adolescentes, suele dejar de crecer en la pubertad.
metástasis hepáticas que secreten las sustancias). Otros síndro- - Condroma. 15-20%.
mes endocrinos, aún más raros, sería el síndrome de Cushing Es el tumor más frecuente del esternón (aunque es mucho
ectópico benigno y la acromegalia. más prevalente en otras localizaciones, como manos, húmero
(Se estudia en Endocrinología) o femur). Masa indolora de crecimiento lento en la unión con-
drocostal. Radiológicamente es lítica con reborde escleroso, y
se confunde con condrosarcoma. Requiere biopsia excisional
En un 10% de ocasiones se comporta como un tumor agresivo
con márgenes amplios, con riesgo de recidiva local tras la exé-
(carcinoide atípico, maligno o tipo 2), y presenta metástasis,
resis. Puede degenerar a condrosarcoma.
fundamentalmente en hígado y en ganglios linfáticos.
- Granuloma eosinófilo (o histiocitosis X ósea). 10-20%.
A pesar de ser tumores muy vascularizados, el diagnóstico Provoca dolor local y febrícula o fiebre, por infiltración ósea
debe hacerse con biopsia. difusa (no suele dar nódulo local). Imagen destructiva ósea
El tratamiento de elección es la resección quirúrgica. compatible con osteomielitis o tumor maligno, requiere biop-
sia excisional.

34
Tema 2 · Neoplasias pulmonares

Tumores malignos - Osteosarcoma.


- Condrosarcoma. Muy raro en costillas, suele afectar huesos largos de extremi-
El tumor maligno más frecuente en pared torácica. De loca- dades. Lesión dolorosa de crecimiento rápido, habitualmente
lización anterior, en esternón o arcos condrocostales. Provoca metastásica al diagnóstico. Requiere quimioterapia y cirugía.
lesión expansiva de crecimiento lento y habitualmente dolo- - Plasmocitoma.
roso. 10% De los pacientes tienen metástasis pulmonares al Versión localizada del mieloma, provoca lesión lítica dolorosa
diagnóstico. Requiere biopsia excisional con bordes amplios, diafisaria. Se trata con quimioterapia y radioterapia local.
pues existe riesgo de recaída local con biopsias incisionales o
resecciones parciales, y puede asociarse radioterapia local.
- Sarcoma de Ewing.
Poco frecuente. Llamado en costillas tumor de Askin, puede
ser doloroso o indoloro con similar frecuencia. Presentan me-
tástasis al diagnóstico el 25% de pacientes. Imagen radioló-
gica típica de “capas de cebolla”. Requiere quimioterapia y
cirugía y/o radioterapia local.

Pregunta ENARM Pregunta ENARM


3. Hombre de 67 años, padece tabaquismo (35 Hombre de 65 años de edad, el cual es llevado por
paquetes/año) e hipertensión, sin otros padeci- vecinos al servicio de urgencias tras sufrir caída de su
mientos detectados. Acude a su consultorio para propia altura en la entrada de su domicilio. A la explo-
una revisión de rutina. ¿Debería realizarse algún ración física se observa caquéctico, somnoliento, con
estudio para detección de cáncer pulmonar? palidez de piel, presenta tos expectorante y hemopti-
sis en una ocasión, se auscultan estertores difusos en
A. No, no requiere estudio alguno. ambos pulmones, abdomen blando sin organomega-
B. Si, una citología de esputo. lias. Se solicita radiografía de tórax encontrando una
C. Si, una tomografía de tórax. lesión radioopaca, redonda, de aproximadamente 5 cm
D. Si, una radiografía de tórax PA. de diámetro mayor, cercana al hilio pulmonar derecho.

4. Con base en la exploración física y la imagen


radiológica, el diagnóstico probable del paciente
es el de:

A. Tuberculosis miliar.
B. Metástasis pulmonares.
C. Neumopatía intersticial.
D. Cáncer broncoalveolar.

5. El principal antecedente que debe ser investiga-


do de forma intencionada en el paciente es:

A. La presencia de tumor sólido primario.


B. La convivencia con personas con tuberculosis activa.
C. La exposición ocupacional a agentes nocivos pulmo-
nares.
D. El tabaquismo crónico.

· Encuentra las respuestas y comentarios de las preguntas ENARM al final del manual ·

35
Tema 3
Enfermedades de la pleura

3.1. Derrame pleural costofrénico posterior en la radiografía lateral de tórax. Si la


cuantía del derrame es mayor, se puede apreciar también el
borramiento del ángulo costofrénico lateral en la radiografía
Por sus características, y con fines diagnósticos, se clasifican en
posteroanterior. Pero la imagen más típica de derrame pleural
trasudados y exudados.
es la presencia de una opacidad homogénea de borde cóncavo
superior (menisco pleural o línea de Ellis-Damoisseau). Otras
Clínica posibilidades, más atípicas, son la elevación de un hemidia-
Es característico el dolor pleurítico (manifestación clínica más fragma en el derrame subpulmonar o la opacificación de todo
frecuente), agudo, punzante que aumenta con los movimien- un hemitórax con desplazamiento contralateral del mediastino
tos inspiratorios, tos o estornudo. Suele localizarse en el cos- ante un derrame masivo. Su sensibilidad para detectar derrame
tado, aunque puede tener otras localizaciones por la diferente pleural es del 24 – 100%, con especificidad del 85 – 100%.
inervación de la pleura. Otros síntomas con los que puede La ecografía torácica es la prueba más sensible para detectar
cursar son disnea (depende más de la rapidez de instauración derrame pleural (sensibilidad del 93%, con especificidad del
que de la cantidad de líquido), tos seca o fiebre (este último 96%) y es muy útil para detectar formas atípicas, como los
dato orienta hacia patología infecciosa). derrames loculados o encapsulados (otras de sus indicaciones
son identificar la localización apropiada para efectuar toraco-
centesis, biopsia pleural o colocación de sonda pleural y dife-
rencial entre derrame y engrosamiento pleural).
La tomografía de tórax debe realizarse para evaluar derrame
pleural de tipo exudado sin diagnóstico, y es particularmente
útil para diferenciar consolidación pulmonar de derrame pleu-
ral, diagnosticar empiemas (y distinguirlos de abscesos pulmo-
nares) y ante probable causa maligna.

Recuerda que...
Los derrames loculados ocurren con más frecuencia en asociación
con condiciones acompañadas de intensa inflamación pleural. Sus
causas más comunes son empiema, hemotórax y tuberculosis.

Si se confirma la presencia de líquido pleural libre, se debe rea-


lizar una toracocentesis para el estudio bioquímico, microbio-
lógico y citológico del líquido. El estudio bioquímico permite
la clasificación de los derrames pleurales en exudados y trasu-
dados. Si el derrame cumple al menos uno de los siguientes
criterios de Light se clasificará como exudado:
Figura 1. Derrame pleural izquierdo. - Proteínas en líquido pleural/proteínas séricas >0,5.
- LDH en líquido pleural/LDH sérica >0,6.
- LDH en líquido pleural >2/3 del límite de la normalidad en
Recuerda que... suero.
De acuerdo a los lineamientos de la GPC, se debe realizar toraco-
centesis en pacientes adultos o pediátricos con derrame pleural Además permite analizar otros parámetros como la glucosa
unilateral, con sospecha de exudado, derrame mayor a 10 mm de (<30 mg/dl típico del derrame reumatoideo), colesterol, TAG,
profundidad o sospecha de derrame paraneumónico. amilasa (típico de las pancreatitis, neoplasias o en perforación
esofágica).
El estudio citológico también puede orientarnos:
Diagnóstico - Hematocrito (líquido sanguinolento con hematocrito >1%
pensamos en derrame pleural maligno, TEP o traumático).
En la exploración física destaca la disminución del murmullo
- Predominio de polimorfonucleares.
vesicular, con abolición de las vibraciones vocales y matidez a la
Se correlaciona con procesos agudos (>10.000/cc derrame
percusión, en la zona del derrame. En la auscultación pulmonar
paraneumónico o empiema) y TBC inicial.
puede oírse el típico roce pleural.
- Predominio mononuclear.
Ante la sospecha de un derrame pleural, lo primero que debe Procesos crónicos (linfocitos >50% orienta a origen tubercu-
realizarse es una radiografía de tórax en proyecciones PA y loso o tumoral).
lateral para el diagnóstico; la proyección en decúbito lateral - Eosinofilia en el líquido.
sobre el hemitórax afecto permite valorar si es significativo Puede aparecer en presencia de sangre, hidroneumotórax, y
(>1 cm). Lo más frecuente es ver el borramiento del ángulo en derrames secundarios a vasculitis o fármacos.

36
Tema 3 · Enfermedades de la pleura

El estudio microbiológico se realizará mediante la determi- que siempre es de tipo exudado), absceso pulmonar o bron-
nación de Gram, de BAAR (bacilos ácido-alcohol-resistentes) y quiectasias.
cultivo del líquido. La Adenosin Deaminasa (ADA) >35 UI en En general se debe obtener una muestra del líquido pleural
líquido pleural sugiere el diagnóstico de tuberculosis pleural, con (toracocentesis) en los derrames paraneumónicos, salvo si el
una sensibilidad de 88-100% y especificidad de 81-97% (GPC). derrame es menor de 10 mm en decúbito lateral.
Los estudios de laboratorio séricos recomendados para la Hablamos de empiema cuando existe en el líquido pleural pus
evaluación inicial y seguimiento en el derrame pleural de tipo y/o tinción de Gram o cultivo positivos.. La presencia de un pH
infeccioso son biometría hemática (el valor de los leucocitos, menor de 7,20 favorece el desarrollo de un empiema. La mayo-
particularmente el nivel de neutrófilos, están elevados al inicio ría de empiemas proceden de un derrame paraneumónico y
y regresan a lo normal conforme se resuelve el proceso infec- los gérmenes más frecuentemente implicados son el S. aureus
cioso) y proteína C reactiva (la falla en la caída de los niveles en y el H. influenzae.
un 50% está asociada con resultados adversos e incremento
El tratamiento dependerá de las características del exudado
de la incidencia de empiema).
(GPC):
La biopsia pleural cerrada o ciega es un método poco cruen-
- Si la glucosa en el líquido pleural es mayor de 60 mg/dl, la
to que suele realizarse cuando no se ha llegado a un diagnós-
LDH pleural es menor de 1000 U/L y el pH es mayor de 7.20,
tico etiológico del exudado con la toracocentesis. Si la biopsia
el tratamiento será médico. En ausencia de un cultivo positivo,
pleural cerrada tampoco es diagnóstica, se realizará toracos-
el tratamiento antibiótico debe comenzar de forma empírica
copia o videotoracoscopia con toma de biopsias dirigida.
y precoz, y ajustarlo al resultado de los cultivos. De acuerdo
con la GPC, se recomienda iniciar con penicilinas con inhi-
bidores de beta-lactamasas o cefalosporinas de 3.ª genera-
ICC ción. En alérgicos a penicilina, utilizar clindamicina sola o en
combinación con una quinolona y/o una cefalosporina de 3ª
generación. Iniciar con vancomicina o linezolid si se sospecha
No Sí o se confirma S. aureus meticilino-resistente o S. pneumoniae
resistente a cefalosporinas.
- Asimétrico - Si la glucosa en el líquido pleural es menor de 60 mg/dl, la
Toracocentesis - Dolor pleurítico
- Fiebre LDH pleural es mayor de 1000 U/L o el pH es menor de 7.2 o
se visualiza pus macroscópico o microorganismos en el Gram,
Criterios de Light: se requiere la colocación de un tubo de drenaje para vaciar el
- Prot lp/ Prot suero >0,5 Sí No líquido pleural.
- LDH lp / LDH suero >0,6 - Ante la presencia de un empiema tabicado, se debe proceder
- LDH lp >2/3 límite
superior normalidad suero Diuresis y a la instilación de fibrinolíticos (estreptoquinasa, uroquinasa
observación o alteplasa) en la cavidad pleural y, si éstos fracasan, se prac-
ticará una toracotomía con desbridamiento de la cavidad. En
S 98% última instancia se procederá a la decorticación.
No Sí E 83% Persiste >3 días

Trasudado: Exudado: Derrame neoplásico


- ICC - Recuento celular
- Cirrosis - Glucosa
Es la causa más frecuente de derrame sanguinolento y la
- Nefrosis - Cultivo segunda causa más frecuente de exudado. Lo más frecuente
- Anatomía patológica es que sea secundario a metástasis de cáncer de pulmón (sobre
todo adenocarcinoma), mama y linfoma. La citología es diag-
Figura 2. Esquema diagnóstico del derrame pleural. nóstica en el 60-80% y la sensibilidad aumenta con la biopsia
pleural. El tratamiento es el de la enfermedad de base. Si el
derrame es sintomático con disnea intensa, se debe realizar
Diagnóstico diferencial de los trasudados evacuación del derrame con tubo de drenaje. En los derrames
Insuficiencia cardiaca congestiva malignos que precisan toracocentesis evacuadoras repetidas
Es la causa más frecuente de trasudado. Además, es la causa está indicada la realización de pleurodesis química (con talco o
más frecuente de derrame en países desarrollados. bleomicina). Previamente debemos descartar que exista atelec-
tasia secundaria a una obstrucción bronquial tumoral, porque
Casi siempre es bilateral (si es unilateral suele localizarse en el en ese caso no habría reexpansión pulmonar y provocaría un
lado derecho). El diagnóstico es clínico, no siendo necesario neumotórax permanente.
realizar toracocentesis, excepto cuando el derrame sea unilate-
ral, curse con dolor pleurítico o fiebre, o no se resuelva con el
tratamiento de la ICC. Derrame tuberculoso
(Se estudia en Infecciosas y Microbiología)
Síndrome nefrótico y cirrosis hepática
El derrame está en relación con la hipoalbuminemia. El trata- Enfermedades autoinmunes
miento es el de la patología de base. - Artritis reumatoide.
En un 5% de las artritis reumatoides (sobre todo en las de
larga evolución, en varones y si hay nódulos subcutáneos),
Diagnóstico diferencial de los exudados
puede aparecer derrame pleural. Suele ser de localización
Derrame paraneumónico unilateral derecha, tipo exudado, rico en linfocitos, con un
Es la causa más frecuente de exudado. Se asocia a neumonía pH menor de 7,20, una glucosa menor de 30 mg/dl y valores
bacteriana (40% de las neumonías bacterianas tienen derrame, elevados de factor reumatoide (FR). Puede existir aumento de

37
Manual ENARM · Neumología y Cirugía Torácica

colesterol y descenso del complemento. No suele tratarse y - Pseudoquilotórax.


responde mal al tratamiento de la AR. Comparte con el quilotórax el aspecto lechoso y se diferen-
- Lupus. cia por el bajo contenido en triglicéridos (TG <50 mg/dl) y el
Derrame bilateral con glucosa y pH normales, anticuerpos an- elevado nivel de colesterol (>250 mg/dl). Suelen ser derrames
tinucleares (ANA) positivos y complemento bajo. La presencia crónicos, en los que en ocasiones existen cristales de coles-
de células LE en el líquido pleural es patognomónica. Suele terol. Las causas más frecuentes de pseudoquilotórax son la
responder a los esteroides. artritis reumatoide y la tuberculosis.

Recuerda que...
Derrame lechoso
El derrame pleural más frecuente es el de la ICC,
que es un trasudado
Colesterol
El exudado más frecuente es el derrame paraneumónico

El exudado sanguinolento más frecuente es el tumoral Normal o disminuido Elevado (>250 mg/dl)

Quilotórax Pseudoquilotórax

Exudados con características peculiares Dos etiologías


Confirmar posibles
- Derrame sanguinolento-hemotórax.
• Derrame sanguinolento.
Hematocrito <50% del hematocrito sanguíneo. Las principa- - TG >110 mg/dl
- Índice colesterol/TG <1 - TBC: glucosa , C
les causas son el derrame neoplásico (causa más frecuente), - Qm elevados - AR: glucosa ,C
el TEP y el derrame traumático. (patognomónico)
• Hemotórax.
Hematocrito >50% del hematocrito sanguíneo. La principal Figura 3. Diagnóstico diferencial del quilotórax y pseudoquilotórax.
causa son los traumatismos. Otras causas son la rotura vas-
cular (aórtica, arteria intercostal, arteria mamaria interna),
lesiones del parénquima pulmonar, y iatrogénico (biopsia Recuerda que...
pleural, anticoagulación). Ante un pH <7,20 y glucosa <60 se debe hacer el diagnóstico
El tratamiento dependerá de la cuantía del hemotórax: diferencial entre: tuberculosis, artritis reumatoide, derrame para-
• Si es <350 ml se tomará una actitud expectante. neumónico complicado, neoplasia y rotura de esófago.
• Si es >350 ml requiere colocación de un tubo de drenaje.
• Si el sangrado inicial es >1500 ml en las primeras 24 horas o
la velocidad de sangrado >200 ml/h durante 4-5 horas conse-
cutivas, se procederá a la realización de toracotomía urgente. DATO CLÍNICO ETIOLOGÍA
Ascitis Cirrosis hepática
Recuerda que...
Distención de venas del cuello Insuficiencia cardiaca, pericarditis
Ante un derrame serosanguinolento siempre
debes descartar las tres T: Disnea al ejercicio Insuficiencia cardiaca
Tumor
TEP Absceso abdominal, empiema,
Trauma Fiebre malignidad, neumonía,
tuberculosis

- Quilotórax. Malignidad, embolismo


Hemoptisis
Se caracteriza por la acumulación de triglicéridos (TG >110 pulmonar, tuberculosis
mg/dl) y quilomicrones en el espacio pleural (la presencia de
quilomicrones es patognomónica), siendo el colesterol normal Hepatoesplenomegalia Malignidad
o bajo (índice colesterol/triglicéridos <1). El aspecto del líquido
es lechoso. Linfadenopatía Malignidad, tuberculosis
Se debe, en la mayoría de los casos, a la rotura del conducto
torácico por un traumatismo o por invasión neoplásica, gene- Ortopnea Insuficiencia cardiaca, pericarditis
ralmente a partir de adenopatías malignas (especialmente en
linfomas). Es la principal causa de derrame en el neonato. Edema periférico Insuficiencia cardiaca
El tratamiento en los de origen traumático consiste en la co-
locación de un drenaje endotorácico con la consiguiente cica- S3, galope Insuficiencia cardiaca
trización espontánea. Se plantea la reconstrucción quirúrgica
si no se resuelve en 14 días. Puesto que el quilotórax es causa Hinchazón de una
Embolismo pulmonar
importante de malnutrición, siempre se debe instaurar nutri- extremidad inferior
ción parenteral o enteral rica en triglicéridos de cadena media
(que pasan directamente al sistema porta sin necesidad de Pérdida de peso Malignidad, tuberculosis
pasar por el sistema linfático).
Tabla 1. Datos clínicos que sugieren una etiología del derrame pleural.

38
Tema 3 · Enfermedades de la pleura

3.2. Neumotórax

Es la presencia de aire en el espacio pleural. Puede ser espontá-


neo (sin traumatismo torácico previo) o traumático. Dentro de
los traumáticos encontramos los penetrantes (puñalada) y los
cerrados; otro tipo son los yatrógenos producidos por técnicas
como la toracentesis, cateterización vías centrales, etc.
Si la presión intrapleural es positiva en todo el ciclo respiratorio
se llama neumotórax a tensión y es una urgencia vital.

Diagnóstico
Debe sospecharse en pacientes con dolor y disnea brusca con
abolición del murmullo vesicular.
Ante un paciente con dolor torácico la primera exploración
radiológica que se hace es una Rx PA y lateral del tórax en ins- Figura 4. Neumotórax normal y a tensión.
piración, pero si se sospecha neumotórax y en los casos dudo-
sos se debe practicar una radiografía de tórax en inspiración y El tratamiento convencional de pleurodesis y hormonoterapia
espiración máxima (técnica también útil para el diagnóstico de tiene una elevada tasa de fracasos. En pacientes que requieran
cuerpos extraños bronquiales), que mostrará despegamiento histerectomía o anexectomía por otro motivo, este tratamiento
de las hojas pleurales, colapso pulmonar subyacente, hipercla- sería la solución definitiva.
ridad donde el aire sustituye al parénquima pulmonar y, si es a
tensión, desviación contralateral del mediastino.
Tratamiento
Manejo inicial
Tipos
- Neumotórax pequeño (<2-3 cm de cámara), paciente esta-
Neumotórax idiopático (espontáneo primario) ble y poco sintomático.
Suele deberse a la ruptura de una bulla apical subpleural. Es Observación con O2 suplementario (pues favorece la re-
típico de varones jóvenes, de hábito asténico (altos y delgados) absorción de aire intrapleural). Se debe repetir una Rx como
y, en más del 90% de los casos, fumadores. Recidiva en la mínimo a las 6 h. Si el neumotórax no ha progresado y el
mitad de los casos, siendo la recurrencia la complicación más paciente tiene acceso rápido al Hospital, puede proponerse
frecuente del neumotórax espontáneo. observación domiciliaria. En caso contrario, ingreso y vigilancia
clínico-radiológica.
Neumotórax espontáneo secundario - Neumotórax grande (>2-3 cm de cámara) y/o muy sinto-
mático, en paciente estable.
Se presenta en pacientes con enfermedad pulmonar, sobre
• Puede intentarse inicialmente aspiración manual a través
todo EPOC, fibrosis pulmonar idiopática o secuelas de tuber-
de catéter intrapleural (que se coloca con aguja): el aire se
culosis. Suelen ser mucho más sintomáticos que los anteriores
aspira manualmente hasta que deje de drenar; entonces se
y de evolución más tórpida.
cierra el catéter y se esperan al menos 4 h con Rx de con-
El tratamiento inicial debe ser más agresivo (tubo de tórax y/o trol; si no hay recurrencia, se retira el catéter, se comprueba
pleurodesis). nueva Rx a las 2 h, y si sigue estable, alta domiciliaria.
• Colocación de un tubo de drenaje endotorácico bajo sello
Neumotórax traumático de agua (Pleur-Evac®, 3 compartimentos).
Si no existe burbujeo espontáneo, se debe conectar la aspi-
Es frecuente el hemoneumotórax. El tratamiento consiste en
ración (aplicar presión negativa). Cuando se consiga el cese
colocar dos tubos de drenaje simultáneamente, uno superior
de la fuga (deje de burbujear el drenaje), se debe pinzar el
(para evacuar aire) y otro inferior (para evacuar sangre).
tubo durante 12-24 horas, repetir Rx, y si el neumotórax
En el neumotórax traumático abierto es típico encontrar un no se ha reproducido, retirar el drenaje. Se considerará fra-
bamboleo mediastínico. caso terapéutico cuando, tras 72h persista burbujeo (fuga
persistente). En esos casos, según la reexpansión pulmonar
Neumotórax a tensión lograda, se puede optar por:
- Reexpansión pulmonar >90%: hay experiencias positivas
Suele producir compromiso respiratorio y/o hemodinámico instilando sangre autóloga en el espacio pleural para
(disnea, hipotensión). No debe esperarse a la radiografía de sellar la fuga, o colocando una válvula unidireccional (de
tórax, ya que es una situación urgente que requiere la rápida Heimlich) en el drenaje y manejando ambulatoriamente al
descompresión de la cavidad pleural. paciente.
(Ver figura 4) - Reexpansión pulmonar <90%: requiere reparación quirúr-
gica (VATS con reparación de la fístula broncopleural).
Neumotórax catamenial
Se produce coincidiendo con el ciclo menstrual. Su mecanismo
de producción es incierto, pero no suele aparecer durante el
embarazo y la lactancia, por lo que la amenorrea es probable-
mente el factor más eficaz para evitar este tipo de neumotórax.

39
Manual ENARM · Neumología y Cirugía Torácica

Prevención de la recurrencia
INDICACIONES DE TUBO DE DRENAJE ENDOTORÁCICO
Para evitar la recurrencia se puede optar por pleurodesis
- Derrame o neumotórax con inestabilidad hemodinámica o física (mediante cirugía, por lo general VATS; es más agresiva
compromiso ventilatorio. pero tiene menor índice de recurrencias), o bien pleurodesis
- Empiema (ph<7,20, glucosa <50, pus o baciloscopia +). química (instilando talco o tetraciclinas a través del propio
- Quilotórax (TG>110, índice col/TG<1, QM). tubo endotorácico). Indicaciones:
- Hemotórax (Hcto >50%) >350 ml.
- 2.º episodio de neumotórax espontáneo primario (neumotó-
- Neumotórax gran tamaño (>30%).
rax recurrente).
- 1.er episodio si:
Tabla 2. Indicaciones de tubo de drenaje endotorácico.
• Es bilateral simultáneo.
• Requiere VATS para el manejo agudo (se aprovecha el acto
quirúrgico).
• Profesión con riesgo personal o para terceras personas
(buzos, pilotos).
• Si existe patología pulmonar predisponente (p. ej., bullas de
enfisema, cavernas tuberculosas, etc.), es decir, neumotórax
espontáneo secundario.

Paciente 3.3. Tumores pleurales

Las lesiones de origen pleural, al igual que las de origen extra-


pulmonar (aquéllas que afectan a la pared torácica, p. ej.,
Succión plasmocitoma), pueden presentarse en la radiografía de tórax
como densidades que forman ángulos obtusos con la pared
torácica (“signo de la embarazada”), a diferencia de las lesio-
nes de origen intrapulmonar, que aparecen como densidades
de contornos nítidos que forman ángulos agudos con la pared
torácica.

Cámara Cámara Cámara de


colectora de sello control de
hidráulico succión

Figura 5. Drenaje endotorácico.


Ángulo agudo

- Neumotórax inestable.
Requiere la colocación de tubo endotorácico urgente; si no
está disponible de inmediato, la colocación de una aguja o un
catéter venoso de 14 french en el 2.º ó 3.er espacio intercos-
tal (línea medioclavicular) permite ganar tiempo descompri-
miendo el espacio pleural. Ángulo obtuso

Figura 7. Lesión pulmonar (ángulo agudo) y lesión pleural/extrapulmonar


Figura 6. Neumotórax. Puede apreciarse que el pulmón derecho aparece más (ángulo obtuso, signo de la embarazada).
negro por colapso del parénquima pulmonar (punta de flecha superior). A su
vez aparece borrado el hemidiafragma derecho por presencia de líquido que
aparece formando un nivel hidroaéreo o hidroneumotórax (punta de flecha Mesotelioma
inferior). La imagen del TC corresponde al mismo paciente. Una vez colocado
un tubo de tórax el pulmón se expandió y se apreció la presencia de una masa
Clínicamente cursa con dolor torácico, tos y derrame pleural.
pulmonar central que comprimía el bronquio principal derecho. Hay dos tipos muy diferentes:

40
Tema 3 · Enfermedades de la pleura

Localizado
Benigno, resecable y sin relación con la exposición a asbesto.
Puede ser asintomático o presentar síndromes paraneoplásicos
como osteoartropatía hipertrófica o hipoglucemia.

Difuso
Maligno, relacionado con la exposición al asbesto e indepen-
diente del tabaquismo. Es poco frecuente. Se presenta con
un periodo de latencia de más de 20 años. La manifestación
inicial suele ser un derrame pleural seroso o serosanguinolen-
to. El diagnóstico en raras ocasiones se consigue con biopsia
pleural a ciegas y suele requerir toracoscopia o toracotomía. Si
el derrame presenta glucosa o pH bajos, el pronóstico es aún
peor.
No hay tratamiento eficaz y su supervivencia suele ser inferior Figura 8. Mesotelioma pleural maligno.
a 6 meses.

Metástasis pleurales
Los tumores pleurales más frecuentes son las metástasis, sobre
todo de adenocarcinomas; los primarios más frecuentes son los
de pulmón y mama.

Pregunta ENARM Pregunta ENARM


6. Hombre de 62 años. Al puncionar un derrame 7. Hombre de 35 años de edad, el cual ingresa al
pleural, obtiene una relación de proteínas líqui- servicio de urgencias tras accidente automovi-
do/suero de 0.7 y relación DHL de 0.9. ¿Cuál de lístico hace 30 minutos. Clínicamente con datos
las siguientes etiologías es la más probable? de dificultad respiratoria. La radiografía de tórax
muestra una zona periférica completamente
A. Nefropatía membranosa. radiolúcida del lado derecho, con el pulmón ipsi-
B. Cirrosis hepática. lateral colapsado. Con base en los antecedentes y
C. Pericarditis constrictiva. los hallazgos radiológicos deberá sospecharse la
D. Mesotelioma. presencia de:

A. Neumotórax.
B. Neumopericardio.
C. Hemotórax.
D. Contusión pulmonar.

· Encuentra las respuestas y comentarios de las preguntas ENARM al final del manual ·

41
Tema 4
Enfermedad pulmonar obstructiva crónica (EPOC)

Recuerda que... 4.1. Concepto y anatomía patológica


De acuerdo a la GPC, la ventilación mecánica está indicada cuan-
do, a pesar del tratamiento farmacológico y de la oxigenoterapia, Bronquitis crónica (BC)
el paciente sigue presentando pH <7.35. Es un concepto clínico y se define por tos y/o expectoración
crónica al menos 3 meses al año durante al menos 2 años con-
secutivos. No todo paciente con bronquitis crónica presenta
La EPOC es una enfermedad definida espirométricamente por EPOC en la espirometría, ni es inevitable su evolución a EPOC.
la presencia de obstrucción poco reversible al flujo espiratorio. Anatomopatológicamente encontramos en los bronquios
Agrupa a dos entidades clínicamente diferenciadas, bronqui- grandes (cartilaginosos), hiperplasia e hipertrofia de las glán-
tis crónica y enfisema, que en la práctica suelen coexistir en dulas submucosas, con un índice de Reid (relación entre el
el mismo paciente. Comparten unas características comunes, espesor glandular y el espesor de la pared bronquial) mayor a
como son la asociación con tabaco, clínica de disnea de esfuer- 0,6 (normal menor a 0,25).
zo progresiva e hiperreactividad bronquial. Sin embargo, ana-
tomopatológicamente están bien diferenciadas; la bronquitis
crónica es una enfermedad propia de vías aéreas, mientras que
el enfisema afecta al parénquima pulmonar. Normal Bronquitis crónica
La prevalencia de EPOC en México, según el proyecto latinoa-
mericano de investigación en obstrucción pulmonar (PLATINO),
es de 7.8%. El tabaco es el principal factor de riesgo, aunque
sólo un 15% de los fumadores desarrollará una EPOC. En
México, el segundo factor de riesgo es la exposición a humo Inflamación Moco
de leña, sobre todo en comunidades rurales; afecta a mujeres
con exposición de más de 200 horas/año. El humo del tabaco
produce una inflamación crónica con acumulación de neutró-
filos en el pulmón (aumento de la actividad elastasa y destruc-
ción de las fibras elásticas de la pared alveolar), hipertrofia e
hiperplasia de las glándulas secretoras de moco, constricción
del músculo liso bronquial, inhibición de la función de los
macrófagos, disminución de la movilidad ciliar y aumento de la
resistencia de las vías aéreas. Figura 2. Bronquitis crónica.

Enfisema
Se define por la anatomía patológica. Se trata de un atra-
pamiento aéreo distal al bronquiolo terminal (bronquiolo
respiratorio, conductos alveolares, sacos alveolares y alveolos),
con dilatación anormal y destrucción de la pared alveolar. En

Alvéolos normales Alvéolos enfisematosos

Figura 1. El tabaco es el principal factor de riesgo de EPOC.

Figura 3. Esquema de alvéolos enfisematosos. Nótese también la diferencia y


desestructuración alveolar condicionada por el atrapamiento aéreo.

42
Tema 4 · Enfermedad pulmonar obstructiva crónica (EPOC)

su génesis intervienen enzimas elastolíticas (como la elastasa Enfisema paraseptal


de los neutrófilos y de los macrófagos alveolares). Afecta a la parte distal del lobulillo: ductus y sacos alveolares
Clínicamente, los enfisematosos tienen mayor sensibilidad a de la periferia del acino (en relación con bullas subpleurales
la disnea. Las complicaciones, como la insuficiencia cardiaca fundamentalmente en campos superiores). Es más frecuente
derecha y la insuficiencia respiratoria global, aparecen en fases en jóvenes, en quienes ocasiona neumotórax espontáneo.
más avanzadas de la enfermedad.
Anatomopatológicamente pueden diferenciarse tres variantes
de enfisema:

Normal Enfisema centrolobulillar Enfisema panlobulillar

RB RB RB
TB TB TB
A

A
A
A A
A

TB: bronquiolo terminal; RB: bronquiolo respiratorio; A: alvéolo.

Figura 4. Tipos de enfisema.


Figura 7. Enfisema paraseptal: afectación subpleural (periférica) en los campos
pulmonares superiores en TC de tórax.
Enfisema centroacinar (centrolobulillar)
Afecta a la parte central (acino proximal): bronquiolo respira-
Enfisema panacinar (panlobulillar)
torio y conductos alveolares centrales. Es el tipo de enfisema
relacionado con el tabaco y a menudo se asocia a bronquitis Se afecta todo el acino de forma uniforme. Es típico del déficit
crónica. Afecta predominantemente a lóbulos superiores. de α1-antitripsina y afecta fundamentalmente a los lóbulos
inferiores. Esta entidad corresponde a menos del 1% de los
casos de EPOC.
El déficit de α-1-antitripsina afecta también al hígado. Las
manifestaciones hepáticas incluyen la hepatitis crónica, la
cirrosis (hasta un 40% de los pacientes con fenotipo PiZZ la
desarrolla en edad avanzada) y el hepatocarcinoma. En niños
se puede producir una hepatitis neonatal, más frecuente en el
fenotipo PiZZ. Estos a su vez son más susceptibles de acabar
desarrollando una cirrosis juvenil.
Se sospecha ante pacientes con antecedentes familiares de
EPOC, enfisema en una mujer, en hombres menores de 40
años o en no fumadores. Es una enfermedad frecuente en
población europea y extremadamente rara en México.
El diagnóstico se establece por los niveles plasmáticos de α1-
antitripsina (normal entre 150-350 mg-dl).

Figura 6. Enfisema centroacinar: afectación central en los campos pulmonares


superiores en TC de tórax.

Enfisema:
- Atrapamiento distal al bronquio
terminal (bronquiolo respiratorio,
conductos alveolares,
sacos alveolares y alvéolos)
Panacinar
- Afecta a todo el acino Centrolobular o centroacinar Pareseptal
EPOC - Localización: - Afectación bronquiolo - Distal (ductus y sacos
• Campos inferiores respiratorio alveolares)
• Difuso en déficit ATT - Afectación campos superiores - Bullas subpleurales en
Bronquitis crónica: • Ocasionalmente afectación - Tabaquismo campos superiores en gente
- Hiperplasia e hipertrofia focal en campos superiores, joven
glándulas mucosas si asociado a tabaquismo
- Aumento células caliciformes
- Indice de Reid >0,6

Figura 5. Anatomía patológica de la EPOC.

43
Manual ENARM · Neumología y Cirugía Torácica

4.2. Características diferenciales de enfisema y


Deficit α1-antitripsina
bronquitis crónica

Inhibición: (Ver tabla 1 en la página siguiente)


Enfisema panacinar Cromosoma 14
Cirrosis hepática - Elastasa neutrófilos
- Tripsina

Herencia
- >70 alelos
- 20 producen enfermedad
- Alelos M, S, Z

- PiMM normal
- PiMS >50% enzima
- PiSS >50% enzima
- PiZZ enfisema <35% (80 mg/dl)
- PiZM enfisema >60 años <35% (80 mg/dl)

Tratamiento: prolastina i.v.

Figura 8. Características del déficit de α1-antitripsina.

Figura 9. Imagen radiográfica del enfisema.

Regla mnemotécnica 4.3. Diagnóstico

El diagnóstico de la bronquitis crónica es clínico y el del enfise-


ma anatomopatológico (aunque suele establecerse por clínica
y radiología compatible). Ante todo paciente con sospecha de
EPOC realizaremos inicialmente una espirometría con prueba
broncodilatadora y una radiografía del tórax. El diagnóstico
de EPOC requiere demostrar la presencia de una obstrucción
bronquial crónica poco reversible con tratamiento. Este hecho
se confirma a través de la espirometría: disminución del FEV1,
capacidad pulmonar total conservada o aumentada y disminu-
ción del índice de VEF1/CVF. La relación VEF1/CVF menor a 0,7
es el criterio de patrón obstructivo, aunque la clasificación de
la gravedad se establece por el FEV1. Sin embargo, el primer
parámetro que se afecta en fumadores es la disminución de los
flujos mesoespiratorios (MMEF o FEF 75/25). La primera alte-
ración respiratoria demostrable en fumadores, reversible tras
suspender el hábito tabáquico, es la obstrucción de vías aéreas
distales. En fases avanzadas, dado el atrapamiento aéreo
característico de la EPOC, hay un incremento de la capacidad
pulmonar total a expensas del volumen residual (aumento VR/
CPT). En esta fase, el índice de Tiffeneau no será valorable.
Por otra parte, el parámetro que mejor indica la gravedad del
enfisema es el DLCO, que está en relación con la cantidad de
El humo del tabaco ASCIENDE al CENTRO parénquima pulmonar destruido, aunque no tiene valor pro-
nóstico. En estos pacientes estaría indicada la realización de
El enfisema por tabaco (CENTROacinar) una TC de tórax para confirmar la presencia de enfisema, así
predomina en lóbulos SUPERIORES como la determinación de los niveles de α1-antitripsina.

44
Tema 4 · Enfermedad pulmonar obstructiva crónica (EPOC)

BRONQUITIS CRÓNICA ENFISEMA

EDAD 50 años 60 años

ASPECTO Cianótico, abotargado, pícnico No cianóticos. Sopladores rosados, asténico

ESPUTO Abundante, purulento Escaso, mucoso

INFECCIONES RESPIRATORIAS Frecuentes Escasas

IC
COR PULMONALE Frecuente Raro, excepto en fases finales
HTP

- Aumento trama broncovascular - Oligohemia pulmonar


- Engrosamiento paredes bronquiales - Atrapamiento aéreo
RX TÓRAX - Cardiomegalia - Silueta cardiaca alargada
- Hemidiafragma - Hemidiafragmas aplanados

PAO2 45-60 60-75

PACO2 Aumentada Normal

POLIGLOBULIA Sí No

DLCO Normal/poco disminuida Disminuida

DISTENSIBILIDAD Normal Aumentada

RETRACCIÓN ELÁSTICA Normal Muy disminuida

DISNEA Leve Grave

REAGUDIZACIONES Frecuentes Fases terminales

AUSCULTACIÓN PULMONAR Roncus, sibilantes que se modifican con la tos Disminución MV

Tabla 1. Características diferenciales de los tipos de EPOC.

4.4. Estadificación GOLD


ESCALA SÍNTOMA
Grado 0 Ausencia de disnea.
La nueva clasificación GOLD 2017 propone una evaluación
combinada de la EPOC. Por una lado clasifica la gravedad Grado 1 Disnea al correr en llano o subir una cuesta ligera.
de la limitación al flujo aéreo, de acuerdo al grado de dis-
minución de la FEV1 en la espirometría, otorgando un grado Disnea que obliga a andar más despacio que las
espirométrico GOLD 1, 2, 3 ó 4 según sea la obstrucción leve, Grado 2 personas de igual edad o que hace detenerse
moderada, grave o muy grave respectivamente. Por otro lado, para respirar al andar a su paso en llano.
clasifica la gravedad de los síntomas (es decir, de la disnea
y las exacerbaciones), estableciendo los grupos A, B, C, y D. La Disnea que obliga a detenerse para respirar tras
disnea se puede evaluar con las clasificaciones m-MRC o con Grado 3 andar 100 metros o después de andar varios
la CAT; las exacerbaciones se evalúan en función del número minutos por terreno llano.
de episodios ocurridos en los últimos 12 meses: se considera
“bajo riesgo exacerbador” si como máximo ocurrió 1 sólo epi- Grado 4 Disnea que impide salir de casa o
sodio que no conllevó ingreso hospitalario en el último año; 2 que se presenta al vestirse o desvestirse.
episodios (o 1 episodio único con hospitalización) se considera
“alto riesgo exacerbador”.
Tabla 2. Escala mMRC (modified Medical Research Council).
De esta forma, cada paciente recibe dos categorías clasifica-
torias, su grado espirométrico y su grupo según síntomas y
riesgo exacerbador. Esta clasificación se emplea para guiar el
tratamiento farmacológico.

(Ver figura 10 y tabla 3 en la página siguiente)

45
Manual ENARM · Neumología y Cirugía Torácica

Síntomas

FEV1 Historia mMRC 0-1 mMRC 2


(% ref) exacerbaciones CAT <10 CAT 10

GOLD 1 >80 2o
FEV1/FVC post-broncodilatador <0,7 1 hospitalización C D
GOLD 2 50-79
GOLD 3 30-49 0o1
Diagnóstico confirmado espirométricamente A B
GOLD 4 <30 sin hospitalización

Gravedad de la obstrucción Valoración de síntomas y


o limitación al flujo aéreo riesgo de exacerbaciones
(”grado espirométrico”) Grupos A, B, C y D
GOLD 1, 2, 3, 4

Figura 10. Clasificación GOLD 2017. Evaluación combinada modificada de la EPOC: los pacientes reciben un grado espirométrico (GOLD 1, 2, 3 ó 4) en función de la
gravedad de su limitación al flujo aéreo; y, a su vez, un grupo (A, B, C, ó D) según la gravedad de sus síntomas y riesgo de exacerbación. De esta forma, un paciente
podría clasificarse GOLD 4, grupo A si presenta obstrucción muy grave (FEV1 <30%) pero poca disnea y ninguna exacerbación en el último año.

EXACERBACIONES SÍNTOMAS TRATAMIENTO

GRADO A Escasos (mMRC 0-1, CAT<10) SABA o SAMA


Bajo riesgo exacerbador
GRADO B Frecuentes (mMRC 2, CAT 10) LAMA y/o LABA

GRADO C Escasos (mMRC 0-1, CAT<10)


LAMA ± LABA ± CI ±
Alto riesgo exacerbador
Roflumilast ± Azitromicina
GRADO D Frecuentes (mMRC 2, CAT 10)

SABA: beta-agonistas de acción corta; LABA: beta-agonistas de acción larga; SAMA: anticolinérgicos de acción corta; LAMA: anticolinérgicos de acción larga; CI: cor-
ticoides inhalados.

Tabla 3. Tratamiento según clasificación GOLD 2017.

nivel bronquial, a diferencia de los corticoides. También care-


4.5. Tratamiento cen de efecto antisecretor, a diferencia de los anticolinérgicos.
Los de acción corta son de elección en los episodios de reagu-
Está demostrado que lo único que aumenta la supervivencia dización, mientras que los de acción larga lo son en la enfer-
en estos pacientes es el abandono del hábito tabáquico medad estable. Por su rapidez de acción, los β2-agonistas de
(actualmente se prefiere la vareniclina sobre el bupropión) y la acción corta inhalados constituyen el tratamiento de elección
oxigenoterapia cuando esté indicada. en situaciones agudas, por lo que se recomienda su empleo
en las reagudizaciones, y en pacientes con EPOC estable cuan-
do, de forma circunstancial, exista deterioro sintomático. En
Broncodilatadores pacientes que requieren el uso regular de broncodilatadores
Son el pilar fundamental del tratamiento, preferiblemente de acción corta, el uso de dispositivos que asocian bromuro de
inhalados: ipratropio y salbutamol tiene mayor efecto que el de ambos de
forma aislada. El uso de β2-agonistas de acción larga se con-
sidera indicado en pacientes con síntomas persistentes, ya sea
Agonistas beta-2 adrenérgicos en monoterapia o generalmente asociados a anticolinérgicos.
Existen fármacos de acción corta (duración de 4-5 horas: sal-
butamol, fenoterol y terbutalina), de acción larga (duración
Anticolinérgicos
de 12 horas: salmeterol y formoterol), y de acción muy larga
(duración de 24 horas: indacaterol). Se emplean por vía inha- Son inhibidores competitivos del receptor muscarínico de ace-
lada como norma general, aunque en crisis graves pueden tilcolina. Se consideran de acción corta (bromuro de ipratropio
emplearse por vía subcutánea o intravenosa, y su efecto adver- cada 4-6 horas) o acción larga (bromuro de aclidinio cada 12
so más común es el temblor dosisdependiente. No carecen de horas; bromuro de tiotropio y glicopirronio cada 24 horas).
efectos cardioestimulantes pese a ser β2selectivos, por lo que el Tienen efecto broncodilatador (menos que los beta-agonistas)
uso sistémico se asocia a riesgo arritmogénico y se reserva para y además reducen la secreción bronquial. Se usan por vía inha-
casos muy graves, de forma casi excepcional. Son los bronco- lada o nebulizada (bromuro ipratropio). Como efectos secun-
dilatadores más potentes, superiores a los anticolinérgicos (y darios pueden producir xerostomía, tos, midriasis o glaucoma.
éstos a las aminofilinas). Actúan sobre vías aéreas de pequeño Son el fármaco más importante para el manejo sintomático
calibre (bronquiolos) y gran calibre (bronquios). Aunque a nivel del EPOC en fase estable junto con los β2-agonistas de acción
experimental producen un efecto antiinflamatorio sobre algu- prolongada, pues además del efecto broncodilatador, reducen
nas células inflamatorias, carecen de efecto antiinflamatorio a la secreción bronquial.

46
Tema 4 · Enfermedad pulmonar obstructiva crónica (EPOC)

Teofilinas Macrólidos a dosis bajas en ciclos prolongados


Se usan por vía oral a nivel ambulatorio, y por vía intravenosa Hay cierta evidencia de que el empleo de azitromicina a dosis
en las reagudizaciones. Cuando se superan los niveles terapéu- bajas (diarias, o semanales alternas) en ciclos prolongados (de
ticos, pueden aparecer náuseas, vómitos, cefalea e inquietud; varios meses) podría disminuir la incidencia de exacerbaciones
si superan los 30 µg/dl los efectos secundarios pueden ser en EPOC muy grave, sin que aparentemente aumenten las
graves llegando a causar convulsiones, hipotensión o arritmias. resistencias comunitarias a macrólidos del neumococo. Se
Su efecto broncodilatador es muy escaso; su efecto beneficioso especula con que este efecto pueda deberse a un potencial
se basa en la estimulación del centro respiratorio, el aumento inmunomodulador de la azitromicina, más que a su acción
de la resistencia de la musculatura respiratoria, la mejora de la puramente antibiótica.
función cardiaca y el aclaramiento mucociliar.
Puesto que sufren metabolismo hepático, pueden presentar Roflumilast
múltiples interacciones. Aumentan el metabolismo de la teo-
filina: el tabaco, la marihuana, edad menor de 16 años, bajo Nuevo fármaco para el manejo de la EPOC. Inhibidor de la fos-
peso, dieta rica en proteínas o fármacos como el fenobarbital, fodiesterasa 4 (PDE-4), actúa como antiinflamatorio bronquial.
fenitoína o la rifampicina. Lo disminuyen las edades extremas NO tiene efecto broncodilatador pero mejora el FEV1 por des-
de la vida, obesidad, insuficiencia cardiaca, estados febriles, censo del daño bronquial inflamatorio progresivo.
dieta rica en hidratos de carbono y fármacos como los esteroi- En GOLD 3-4 ha demostrado reducir las reagudizaciones en
des, cimetidina, eritromicina, propanolol, etc. comparación con β2-agonistas de acción prolongada y corticoi-
des orales, pero no hay estudios que lo comparen con los corti-
coides inhalados (que son el fármaco de elección para reducir las
Regla mnemotécnica reagudizaciones), por lo que su utilidad real es dudosa.
Factores que modifican el metabolismo de la teofilina: Tiene más tasa de efectos secundarios que los fármacos habi-
tuales (cefalea, náuseas, hiporexia, dolor abdominal, diarrea).
Disminuyen los niveles Se sugiere que podría ser útil en pacientes en estadio C o D con
“Jóvenes fumadores, delgados, tuberculosos y epilépticos” mal control a pesar de medicación habitual.
Son factores que disminuyen los niveles de teofilina (por aumento
del aclaramiento) la edad jóven, el tabaco y la marihuana, el bajo
peso, y fármacos como tuberculostáticos (isoniacida, rifampicina) Rehabilitación
y antiepilépticos (fenitoína y fenobarbital). Evitar el sedentarismo y realizar ejercicio físico regular debe
recomendarse de forma generalizada a todos los pacientes
Incrementan los niveles con EPOC.
“Ancianos obesos, cirróticos en insuficiencia cardiaca Según las guías nacionales sobre la EPOC, la rehabilitación
con infección, protector gástrico y corticoides” pulmonar en estos pacientes mejora la disnea, la calidad de
Son factores que incrementan los niveles de teofilina (por disminu- vida y la capacidad de ejercicio, y disminuye la utilización de
ción de su aclaramiento) la edad avanzada, la obesidad, la cirrosis los servicios sanitarios y los ingresos hospitalarios. No se ha
e insuficiencia cardiaca y fármacos como los macrólidos, quinolo- observado una mejora en la supervivencia en estos pacientes.
nas o la clindamicina, la cimetidina y los esteroides. También la
hipoxemia grave (<45 mmHg) y la acidosis respiratoria.
Oxigenoterapia
La oxigenoterapia crónica domiciliaria incrementa la supervi-
vencia de aquellos pacientes con EPOC en los que esté indicada
Corticoides y siempre que se emplee un mínimo de dieciséis horas diarias,
Corticoides inhalados (CI: budesonida, beclometasona, incluidas las nocturnas. Además, los pacientes con EPOC
fluticasona) deben recibir vacunación anual antigripal, y antineumocócica
En el pasado han sido extensamente empleados en la fase los mayores de 65 años o con EPOC muy grave.
estable de la EPOC. Sin embargo, a la luz de la evidencia acu-
mulada, las recomendaciones actuales restringen su empleo
en la mayoría de pacientes. Sus riesgos (entre los que destaca Importante
una mayor incidencia de neumonía) parecen ser mayores Indicaciones de oxigenoterapia crónica o domiciliaria:
que sus beneficios (disminuir las exacerbaciones). Por ello, han - Si tras el abandono del tabaco y tras tratamiento broncodilatador
quedado relegados a una 2.ª línea de tratamiento; concre- correcto y completo, el paciente en situación estable presenta:
tamente para aquellos pacientes en los grupos C y D que no • PaO2 55 mmHg (o bien SatO2 88) en situación basal.
logren un adecuado control de las exacerbaciones con mono- • PaO2 entre 55-60 mmHg (o bien SatO2 = 89) y alguna de las
terapia LAMA o con combinaciones fijas LAMA + LABA. No siguientes situaciones: reducción del intelecto, policitemia
obstante hay algunos subgrupos de pacientes que se podrían (hematocrito >55%), trastornos del ritmo cardiaco, insuficiencia
beneficiar en mayor medida de los CI: pacientes con fenotipo cardiaca congestiva, hipertensión pulmonar o cor pulmonale.
mixto EPOC-asma; y pacientes EPOC con eosinofilia periférica. - Si presenta una PaO2 mayor a 60 mmHg durante el día pero al-
canza niveles menores a 55 mmHg con el esfuerzo o con el sueño.
Corticoides sistémicos
En general, el empleo de corticoides sistémicos debe quedar
restringido al manejo de las exacerbaciones, siempre en ciclos Trasplante pulmonar
cortos. El empleo de esteroides sistémicos en ciclos largos produce El enfisema es la indicación más frecuente de trasplante pul-
importantes efectos secundarios sin haber demostrado mejorar la monar. Éste se reserva para los pacientes menores de 65 años
supervivencia ni la calidad de vida, por lo que está contraindicado. con EPOC muy avanzada, es decir, aquellos que presentan

47
Manual ENARM · Neumología y Cirugía Torácica

un FEV menor del 25% del previsto a pesar del tratamiento


MEDIDAS GENERALES
médico, con hipertensión pulmonar o cor pulmonale, con una
PaCO2 mayor a 55 mmHg. Sin embargo, no se ha demostrado Abandono del consumo del tabaco;
que el trasplante pulmonar aumente la supervivencia. inclusión en programa de deshabituación si es preciso.
Practicar ejercicio de forma regular.
Vacunación de gripe y neumococo.
Cirugía de reducción de volumen
La cirugía de reducción de volumen pulmonar (CRVP) es un EPOC LEVE Y MODERADA
procedimiento quirúrgico dirigido a la resección de las áreas
con mayor grado de destrucción parenquimatosa, en pacientes Pacientes pocos sintomáticos
con enfisema grave de distribución heterogénea. Esta cirugía Pacientes poco sintomáticos (Grupo A)
tiene como objetivo resecar el tejido pulmonar enfisematoso, SABA/SAMA a demanda
para descomprimir el tejido pulmonar y mejorar la retracción
elástica del parénquima pulmonar restante. Se emplea funda- Pacientes sintomáticos (Grupo B)
mentalmente en pacientes con bullas únicas localizadas, en 1. LAMA o LABA en monoterapia.
quienes se obtienen los mejores resultados. La complicación 2. LAMA + LABA en combinación fija.
más frecuente de esta cirugía es la fuga aérea. Se utilizan crite-
rios de operabilidad similar a los del cáncer broncogénico para EPOC GRAVE Y MUY GRAVE (GRUPOS C Y D)
valorar el riesgo quirúrgico (no predicen la capacidad funcional
posquirúrgica, sino el riesgo de mortalidad perioperatoria); 1. Si no existe mejoría sintomática con los broncodilatadores ante-
cuanto mejor sea la situación clínica y las pruebas funcionales, riores, añadir glucocorticoides inhalados.
mejor pronóstico. La rehabilitación pre- y post-intervención en 2. Si hay exacerbaciones persistentes, considerar añadir roflumilast
estos pacientes es esencial. (sobre todo en fenotipo bronquitis crónica) o ciclos prolongados
de azitromicina a dosis bajas.
3. Considerar la inclusión en programa de rehabilitación.
Reagudizaciones
4. Evaluar la posible indicación de oxigenoterapia domiciliaria.
Las infecciones respiratorias son la causa más frecuente de 5. Evaluar la severidad del enfisema pulmonar. Si es severo consi-
reagudización en la EPOC. Antiguamente se consideraba que derar cirugía de reducción de volumen pulmonar.
el principal agente eran los virus (rinovirus), siendo las bacterias 6. En pacientes <65 años, considerar la indicación de trasplante
(H. influenzae, S. pneumoniae, M. catarrhalis) responsables pulmonar.
de las reagudizaciones más graves y sintomáticas); hoy día se
admite que la principal causa es bacteriana tanto en el ámbito
hospitalario como en atención primaria. Sospecharemos una SABA: beta-agonistas de acción corta; LABA: beta-agonistas de acción larga;
SAMA: anticolinérgicos de acción corta; LAMA: anticolinérgicos de acción
reagudización ante la presencia de los siguientes criterios de
larga; CI: corticoides inhalados.
exacerbación: aumento de la expectoración, purulencia del
esputo y/o aumento de la disnea. En cuanto al tratamiento, Tabla 4. Tratamiento del paciente con EPOC estable.
habrá que optimizar el tratamiento broncodilatador por vía
inhalada y utilizar antibióticos en los casos que cumplan dos o
más criterios de exacerbación. El antibiótico de elección ante
una reagudización infecciosa en la EPOC es la amoxicilina-
clavulánico. De acuerdo con el espectro bacteriano, también MEDIDAS GENERALES
pueden emplearse macrólidos y fluorquinolonas. Cuando
la reagudización se acompañe de broncoespasmo, o sea - Mantener su tratamiento habitual
moderada o grave, o no evolucione favorablemente, deberán - Tratamiento broncodilatador en urgencias: anticolinérgico +
añadirse corticoides sistémicos al tratamiento. Si hay datos de agonista-ß2 de acción corta a dosis altas, inhalado o nebulizado
cor pulmonale deben añadirse diuréticos. Puede estar indicada - Antibioterapia, si están presentes dos o más criterios de
tromboprofilaxis para prevenir el desarrollo de ETEV. exacerbación
- Considerar la administración de glucocorticoides sistémicos
Cuando, a pesar del tratamiento médico pautado, el paciente - Considerar necesidad de oxigenoterapia si presenta insuficiencia
no evoluciona favorablemente y presenta elevación progresiva respiratoria
de la PaCO2 junto a descenso del pH (acidosis), implica que - Considerar necesidad de ventilación mecánica si presenta
tiene insuficiencia respiratoria aguda hipercápnica, siendo bajo nivel de conciencia, inestabilidad hemodinámica, fatiga
necesaria la ventilación mecánica (invasiva o no). En el caso muscular o acidosis respiratoria progresiva
de tratarse de un paciente sin gran repercusión del nivel de - Considerar necesidad de ingreso hospitalario si no hay mejoría
consciencia y acidosis con pH >7,20, estaría indicada la ven- o concurren datos de gravedad
tilación mecánica no invasiva (VMNI). Si el paciente está - Optimizar el tratamiento broncodilatador domiciliario al alta:
inconsciente o con pH <7,20, estaría indicada la intubación • Incrementar dosis anticolinérgico/agonista-ß2 de acción corta
orotraqueal y ventilación mecánica invasiva (VMI). • Añadir agonista-ß2 de acción prolongada
• Valorar corticoide inhalado si presenta broncoespasmo.
Valorar ciclo corto de corticoides orales
• Añadir diuréticos si insuficiencia cardiaca derecha
- Valorar necesidad de tromboprofilaxis
- Valorar evolución a las 48-72 horas. Si no hay mejoría,
considerar intensificación de tratamiento, causa alternativa,
complicación intercurrente, y necesidad de ingreso hospitalario

Tabla 5. Tratamiento de la EPOC reagudizada.

48
Tema 4 · Enfermedad pulmonar obstructiva crónica (EPOC)

4.6. Comorbilidades en el paciente EPOC 4.7. Pronóstico

La EPOC se asocia a múltiples comorbilidades derivadas de la La gravedad de la EPOC se clasifica en función del FEV1 (se
exposición a factores de riesgo común (tabaquismo principal- estudia en Estadificación GOLD). El valor de FEV1 en el
mente) como el cáncer de pulmón, infarto agudo de miocar- momento del diagnóstico y el ritmo de disminución anual del
dio, vasculopatía periférica... FEV1 son los mejores índices aislados para predecir el pronóstico
También se asocia a comorbilidades derivadas del tratamiento de un paciente estable. El mejor predictor global es el índice
recibido (osteoporosis por el tratamiento corticoideo) o de su BODE (Body Mass Index, Obstruction, Dyspnea, Exercise). En
limitación funcional (ansiedad/depresión). cambio, la tasa de supervivencia de los enfermos con EPOC que
requieren ventilación mecánica depende fundamentalmente
Los pacientes con EPOC e hipoxemia inicialmente pueden
del nivel de actividad física que tuvieran antes del ingreso.
presentar poliglobulia, pero en estadíos avanzados de la enfer-
medad es habitual encontrar anemia o incluso caquexia de (Ver tabla 6)
enfermedad crónica.
La EPOC se asocia con frecuencia a síndrome de apnea-hipop-
nea del sueño (SAOS). Se denomina síndrome “overlap” o
de solapamiento cuando coexisten las dos entidades. Se debe
sospechar en aquellos pacientes que, teniendo una alteración
funcional leve, presentan complicaciones que no se corres-
ponderían con su gravedad. La poliglobulia y la insuficiencia
respiratoria (sobre todo hipercápnica) en un EPOC leve deben
hacernos sospechar en la coexistencia de SAOS.
Por otra parte, una disnea elevada con signos de insuficiencia
cardiaca derecha en un paciente con EPOC debe hacernos sos-
pechar la coexistencia de hipertensión pulmonar.

PUNTUACIÓN
MARCADORES
0 1 2 3
B (Body mass Index) IMC >21 21 - -

O (Obstruction) FEV1 65 50-64 36-49 35

D (Dyspnea) mMRC 0-1 2 3 4

E (Exercise) Prueba de caminata a los 6 minutos 350 250-349 150-249 149

Supervivencia a los 52 meses: 1-2 puntos: 82%, 3-4 puntos: 69%, 5-6 puntos: 60%, 7-10 puntos: 25%.

Tabla 6. Índice BODE.

Pregunta ENARM
Mujer de 53 años de edad actualmente en estudio por
probable enfermedad pulmonar obstructiva crónica.

8. ¿Cuál es estudio de elección para realizar el diag-


nóstico?

A. Gasometría.
B. Radiografía de tórax.
C. Espirometría.
D. Tomografía computarizada de tórax.

9. Usted confirma el diagnóstico de sospecha. ¿Cuál


de los siguientes medicamentos son considera-
dos los más importantes en el manejo sintomáti-
co del paciente con EPOC?

A. Broncodilatadores.
B. Mucolíticos.
C. Oxígeno.
D. Esteroides.

· Encuentra las respuestas y comentarios de las preguntas ENARM al final del manual ·

49
Tema 5
Asma

Recuerda que... y el parainfluenza en niños y rinovirus (el más frecuente) e


influenza en adultos.
De acuerdo con la GPC, los ácaros del polvo caseros
Especial interés merece la tríada ASA (o tríada de Widal), que
son la primera causa alérgica de asma.
afecta al 10% de los asmáticos y consiste en la asociación de
asma, poliposis nasal e intolerancia a la aspirina y otros AINE.
En estos pacientes debe evitarse la administración de aspirina ya
Concepto que puede desencadenar un episodio severo de asma. En gene-
Es un proceso inflamatorio crónico de la vía aérea, que se ral, el paracetamol (acetaminofén) es bien tolerado.
caracteriza por una hiperrespuesta traqueobronquial que
condiciona una obstrucción variable al flujo aéreo total o
parcialmente reversible. Anatomía patológica
La reacción asmática se debe a un mecanismo de hipersensi- El dato macroscópico más llamativo es la hiperinsuflación
bilidad de tipo inmediato (tipo I), con liberación de histamina, pulmonar, con ausencia de colapso alveolar al abrir la cavidad
bradiquinina y SRS-A (sustancia de reacción lenta: leucotrienos pleural en la necropsia.
C4 y D4). Microscópicamente destaca la hipertrofia de las células muscu-
lares bronquiales, el aumento de glándulas y células mucosas,
inflamación de la mucosa, el edema e infiltrado eosinófilo y la
Fenotipos de asma descamación epitelial.
El asma es un síndrome heterogéneo resultante de la interac- Al finalizar las crisis de asma suele aparecer tos productiva,
ción entre factores ambientales y genéticos. Se han caracteri- encontrándose en el esputo espirales de Curschmann
zado diversos fenotipos, aunque no existe evidencia suficiente (material mucinoso acumulado en los bronquios distales que
para recomendar un tratamiento específico basándose en ellos. se desprende, manteniendo la forma tubular del bronquio),
Los fenotipos de asma se pueden agrupar en tres grandes cristales de Charcot-Leyden (productos de degeneración de
bloques no excluyentes entre sí: clínicos o fisiológicos, relacio- los eosinófilos), y cuerpos de Creola (agregados de células
nados con desencadenantes e inflamatorios. epiteliales).

Clínicos o fisiológicos
- Asma grave. Factores Infecciones:
- Asma con exacerbaciones graves. predisponentes - Niños:
Predisponen al asma: • VRS
- Asma refractaria al tratamiento. • Atopia • Parainfluenza
No alérgicos y con corticodependencia. • Intolerancia AINE - Adultos:
- Asma de inicio precoz. • Rinovirus
• Adenovirus
Menores de 12 años, suele ser alérgica (asma extrínseca). • Influenza
- Asma de inicio tardío. Factores causales
Principalmente mujeres adultas. Sin alergia (asma intrínseca). Sensibilización y comienzo:
• Alérgenos del interior
- Asma con limitación fija al flujo aéreo. (ácaros, cucarachas, Factores
Remodelación bronquial, solapamiento con EPOC. pelos y pieles animales, Asma estable desencadenantes
- Asma y obesidad, con síntomas más graves. hongos) Provocan exacerbaciones:
• Alérgenos del exterior • Alérgenos
(polen, hongos) • Ejercicio
Relacionados con desencadenantes • Aspirina y AINE • Frío, cambios
temperatura
- Asma alérgica. • Gases irritantes
Alérgenos ambientales u ocupacionales. • Estrés y emociones
Exacerbación • Infecciones víricas
- Asma inducida por antinflamatorios no esteroideos (AINE). Factores favorecedores • Conservantes /
- Asma inducida por menstruación. Aumentan posibilidad asmática
colorantes
desarrollo asma ante • AAS, AINE
- Asma inducida por ejercicio. factor causal:
• Tabaco
• Polución
Inflamatorios • Infecciones víricas,
- Asma eosinofílica. parasitarias
• Baja talla al nacer
Suele ser alérgica y buena respuesta a glucocorticoides inha- • Obesidad
lados.
- Asma neutrofílica.
Figura 1. Factores de riesgo para el asma bronquial.
Suele darse en pacientes con enfermedad grave y exacerba-
ciones graves, con peor respuesta a corticoides inhalados.
- Asma paucigranulocítica. Clínica
Los síntomas típicos son tos, disnea, sibilancias espirato-
Patogenia rias y sensación de opresión torácica, de carácter variable. No
El desencadenante más común de crisis asmática son las obstante, en algunos pacientes la única manifestación es la tos
infecciones, principalmente víricas: virus respiratorio sincitial crónica escasamente productiva. Los síntomas de asma suelen

50
Tema 5 · Asma

aparecer de forma episódica, muchas veces relacionados con Por otro lado, cifras muy altas sugieren otras enfermedades,
factores desencadenantes o con el esfuerzo. Es característica la como la aspergilosis broncopulmonar alérgica, neumonía eosi-
aparición de los síntomas durante la noche o en las primeras nófila crónica, etc.
horas de la mañana.
Pruebas de atopia
Test cutáneo (prick test), pruebas de provocación bronquial
y determinación de IgE total. Sirven para identificar agentes
desencadenantes, pero no tienen utilidad para el diagnóstico.

Radiografía de tórax
Es poco útil en intercrisis, donde suele ser normal. En la fase
aguda puede mostrar datos de hiperinsuflación, y es necesaria
para descartar procesos concomitantes y posibles complicacio-
nes (neumotoráx, neumomediastino). No obstante, el hallazgo
más frecuente tanto en fase estable como en crisis es una
radiografía normal.

Pruebas funcionales
La espirometría demuestra un típico patrón obstructivo, con
disminución del FEV1 y de la relación FEV1/CVF. Asimismo se
objetiva un aumento del VR de hasta un 400% y una CVF
menor al 50%. La reversibilidad se evidencia mediante un
test de broncodilatación: el aumento del FEV1 mayor o igual
al 12% o 200 ml tras la administración de un broncodilatador
Figura 2. Bronquiolo asmático. Mucosa inflamada, acúmulo de moco y bron- inhalado confirma el diagnóstico; pero la negatividad de la
coconstricción. prueba broncodilatadora no lo descarta.
La variabilidad de la obstrucción bronquial se valora con la medi-
Diagnóstico ción del pico de flujo espiratorio máximo (FEM, PEF o peak espi-
ratory flow). Existen medidores portátiles del PEF que permiten
El diagnóstico de sospecha de asma se basa en la historia clínica,
realizar mediciones diarias matutinas y vespertinas. Variabilidad
pero la confirmación se establece por la exploración funcional
mayor del 20% del PEF es diagnóstica de asma. Ante una crisis
que objetive una obstrucción bronquial. Puede demostrarse la
asmática, el PEF es mejor indicador de gravedad que los signos
hiperreactividad de la vía aérea frente a los distintos estímulos,
clínicos. Además, permite valorar la respuesta al tratamiento.
la reversibilidad de la obstrucción bronquial y la variabilidad
de la misma. La ausencia de obstrucción o de reversibilidad en
un momento dado no descarta el diagnóstico de asma.

Exploración física
El síntoma más común es tos, seguida por sibilancias desenca-
denadas por el ejercicio. En las crisis de asma la auscultación
pulmonar muestra sibilancias y espiración alargada. En casos
leves, las sibilancias (de predominio espiratorio) aparecen en las
dos fases del ciclo respiratorio y son de tonalidad más grave.
En casos graves, se vuelven más agudas e incluso pueden des-
aparecer en situaciones extremas, existiendo silencio ausculta-
torio. Puede haber tiraje o uso de muscualtura accesoria. Debe
presentar taquipnea, su ausencia es dato de gravedad. En crisis
graves pueden aparecer signos de fatiga muscular.

Figura 3. Peak flow.


Gasometría arterial
Es la prueba que mejor indica el estado de oxigenación en
una crisis asmática. Durante la crisis suele haber hipoxemia Prueba de provocación bronquial
y alcalosis respiratoria por hiperventilación (PaCO2 menor a Puesto que el asma es una enfermedad reversible y episódica,
35 mmHg). El agotamiento de la musculatura respiratoria o la espirometría puede ser normal si se realiza en un momento
una obstrucción más grave condicionan una normo o hiper- puntual en el que el paciente se encuentra asintomático. En
capnia, que es un signo de gravedad importante, con acidosis estos casos, se debe realizar una prueba de provocación bron-
respiratoria, que al final puede asociarse a una acidosis láctica quial para poner de manifiesto la hiperreactividad bronquial, es
apareciendo acidosis mixta. decir, una respuesta exagerada de la vía aérea frente a ciertos
agentes inhalados o a estímulos físicos. Los tests más utilizados
son el de la provocación con metacolina o el ejercicio, pero
Analítica también pueden realizarse con histamina. La prueba se consi-
Tanto en el asma intrínseco como en el extrínseco suele existir dera positiva si se detecta una caída del FEV1 superior al 20%
eosinofilia. Sin embargo, su ausencia no excluye el diagnóstico. respecto al valor basal tras la administración del estímulo.

51
Manual ENARM · Neumología y Cirugía Torácica

Fracción exhalada de óxido nítrico (FeNO) siendo los efectos secundarios más frecuentes la disfonía y
El NO se sintetiza por tres isoformas de NO-sintasa, la endo- la candidiasis orofaríngea. Los preparados inhalados son la
telial (eNOS), la neuronal (nNOS) y la inducible (iNOS). En budesonida, la ciclesonida, el dipropionato de beclometa-
el asma, las células inflamatorias y el epitelio presentan un sona y la fluticasona. Por vía oral se emplean la prednisona y
aumento de iNOS, que condiciona un aumento de NO en el prednisolona, cuando el asma no se controla con corticoides
aire exhalado. La cuantificación de FeNO se correlaciona con el inhalados. La vía intravenosa se emplea en las exacerbacio-
grado de inflamación traqueobronquial. Es dudosa su utilidad nes graves (hidrocortisona y metilprednisolona).
práctica, aunque podría guiar el tratamiento, conduciendo a Están indicados ante cualquiera de las siguientes situaciones:
menores dosis de corticoides inhalados. pacientes que emplean beta2-agonistas inhalados 3 veces
a la semana o más, en pacientes sintomáticos 3 veces a la
semana o más y que se despiertan 1 noche a la semana,
así como aquellos que han tenido 1 ataque de asma y que
Historia clínica sugestiva requirieron corticosteroides orales en los últimos 2 años. Los
agonistas beta2 inhalados de acción prolongada, no
Espirometría deben utilizarse sin corticosteroides inhalados.
• Cromonas.
El cromoglicato sódico y el nedocromil sódico no tienen un
Obstrucción Normal efecto broncodilatador, sino que actúan estabilizando la
membrana de los mastocitos, impidiendo la degranulación
de éstos y la consiguiente liberación de mediadores. Se em-
Test broncodilatador Test de provocación
plean vía inhalada. El cromoglicato sódico es un fármaco
muy seguro, por lo que con frecuencia se emplea en niños
con asma leve. También está indicado en el tratamiento de
Positivo Negativo Positivo Negativo
fondo del asma ocupacional y en el asma de esfuerzo.
• Antagonistas de los receptores de leucotrienos.
ASMA Descartar Son el montelukast y el zafirlukast. Están especialmente in-
- Valoración de gravedad asma
- Valoración etiología dicados en el tratamiento del asma inducida por el esfuerzo
y en la tríada ASA. También son útiles como tratamiento
Figura 4. Algoritmo diagnóstico del asma bronquial. complementario en el asma leve y moderada. No se usan en
monoterapia o como primera elección en el tratamiento del
asma.
• Anticuerpos anti IgE.
5.1. Tratamiento Omalizumab, subcutáneo y de administración mensual, in-
dicado en el tratamiento del asma extrínseca grave mal con-
Objetivos: trolada con el tratamiento convencional, y con niveles de IgE
en sangre superiores a 100 UI/ml.
- Eliminar la sintomatología o disminuirla, de manera que no
• Otras terapias.
entorpezca la actividad cotidiana del paciente ni altere su ca-
Los inmunosupresores (metotrexato, ciclosporina y oro oral)
lidad de vida.
se pueden administrar como prueba una vez que otros tra-
- Mantener una función pulmonar normal o cercana a la nor-
tamientos han fracasado; sin embargo, se recomienda que
malidad.
esto sea en 3er nivel de atención por expertos en asma di-
- Prevenir las agudizaciones y, si ocurren, acortar su duración.
fícil. No existe evidencia de beneficio del uso de colchicina,
- Evitar al máximo los efectos secundarios de la medicación.
inmunoglobulina intravenosa, terbutalina, terapias anti-TNF
- Uso de agonista β2-adrenérgico de acción corta no más de 2
alfa o inmunoterapia.
veces a la semana.

Asma crónica
A nivel terapéutico, los fármacos de uso en el asma se divi-
den en controladores (mejoran el grado de control al reducir El tratamiento del asma debe ser individualizado y se debe
la inflamación subyacente) y aliviadores o sintomáticos o de definir un plan terapéutico escalonado, por lo mismo no debe
rescate (proporcionan un alivio sintomático). El tratamiento de retirarse de forma súbita, sino hacer una reducción gradual de
rescate fundamental es el beta-agonista de acción corta. los fármacos inhalados. Se debe evitar la exposición a los fac-
tores desencadenantes y vacunar anualmente contra la gripe.
Los fármacos que se utilizan en el tratamiento del asma pue-
Asimismo es conveniente realizar una valoración periódica
den dividirse fisiológicamente en:
de la respuesta al tratamiento mediante el PEF. En el caso de
- Broncodilatadores (se estudia en EPOC). coexistir asma con rinitis también es importante tratarla.
• Agonistas ß2-adrenérgicos.
Se puede establecer un tratamiento según la gravedad del
• Xantinas.
asma (clasificación útil para establecer el tratamiento inicial)
• Anticolinérgicos.
o según el grado de control (útil en el seguimiento evolutivo,
- Antiinflamatorios. especialmente tras una reagudización).
• Corticoides.
Según la gravedad del asma (ver tabla 1 en la página
Son los fármacos antiinflamatorios por excelencia en el trata-
siguiente) tenemos asma intermitente, leve persistente, [per-
miento del asma bronquial. Sin embargo, no ejercen ningún
sistente] moderado y [persistente] grave:
efecto frente a la respuesta broncoconstrictora inmediata al
alérgeno o al ejercicio, pero sí que inhiben la respuesta tardía - Asma intermitente.
y disminuyen la hiperreactividad bronquial. Se pueden admi- ß2-adrenérgicos de acción corta, inhalados o nebulizados, a
nistrar por vía inhalatoria o por vía sistémica. Por vía inha- demanda. En el asma de esfuerzo o profesional pueden usarse
lada no inducen supresión suprarrenal ni efectos sistémicos, el cromoglicato o nedocromil sódico o antagonistas de los re-

52
Tema 5 · Asma

CRISIS VARIABILIDAD
CRISIS PEF BASAL OTROS TRATAMIENTO
NOCTURNAS PEF
- β2-adr acción corta a demanda
INTER- 1/sem 2/mes 80% <20% Asma esfuerzo - Cromoglicato/nedocromil*
MITENTE - Antagonistas leucotrienos*

- β2-adr acción corta a demanda


PERSISTENTE >1/sem y
>2/mes 80% 20-30% - Corticoides inhalados
LEVE <1/día
- Niños: cromoglicato**

- β2-adr acción corta a demanda


PERSISTENTE Síntomas diarios >1/sem 60-80% >30%
Limita actividad
- Corticoides inhalados
MODERADO no continuos física y sueño
- β2-adr acción larga

- β2-adr acción corta a demanda


PERSISTENTE Continuos Diario 60% >30%
Actividad física y - Corticoides inhalados
SEVERO sueño limitados - β2-adr acción larga
- Corticoides orales*

*Considerar
**Alternativa para ahorrar corticoides

Tabla 1. Clasificación en estadios clínicos y tratamiento del asma crónico según gravedad.

CONTROLADA PARCIALMENTE CONTROLADA


INCONTROLADA
(TODOS LOS DATOS) (1 O 2 EN UNA SEMANA)

SÍNTOMAS DIURNOS Máximo 2/semana >2/semana

LIMITACIÓN ACTIVIDAD No Sí
Tres o más datos de asma
SÍNTOMAS NOCTURNOS No Sí parcialmente controlada en la
O DESPERTAR
misma semana
NECESIDAD RESCATE Máximo 2/semana >2/semana

PEF O FEV1 Normal, >80% <80%

REAGUDIZACIÓN No en año previo En año previo En última semana

Tabla 2. Clasificación y tratamiento del asma crónica según grados de control.

ceptores de leucotrienos como el montelukast, vía oral, antes - Asma controlada.


de la exposición al posible desencadenante. Mantener tratamiento, intentar reducción progresiva.
- Asma persistente leve. - Asma parcialmente controlada.
ß2-adrenérgicos de acción corta a demanda + corticoides in- Valorar mantener tratamiento o escalar, según tolerancia del
halados a dosis bajas (budesonida, beclometasona y flutica- paciente.
sona). Se asocian ß2-adrenérgicos de acción larga o teofilina si - Asma mal controlada o inestable.
hay síntomas nocturnos. En niños y jóvenes, como alternativa Escalar uno o dos peldaños el tratamiento hasta conseguir
y con la intención de ahorrar corticoides, se pueden usar cro- control.
moglicato o nedocromil.
- Asma persistente moderada. Los “escalones” de tratamiento del asma según grado de
ß2-adrenérgicos de acción corta a demanda + corticoides in- control son:
halados a dosis medias-altas + broncodilatadores de acción
1. Solo tratamiento de rescate, sin medicación de control.
prolongada.
2. Corticoides inhalados dosis bajas.
- Asma persistente severa.
3. Subir corticoide inhalado a dosis media, O añadir β2 de ac-
Similar al del asma persistente moderado pero a dosis más
ción prolongada (mejor opción), o añadir antileucotrieno, o
altas, probablemente combinando broncodilatadores de ac-
añadir teofilina.
ción prolongada y antiinflamatorios, y si no es suficiente, se
4. Subir corticoide inhalado a dosis media o alta Y añadir beta-
añaden corticoides orales (prednisona, prednisolona).
agonista de acción prolongada (mejor opción) y/o antileuco-
trieno y/o teofilina.
Según el grado de control se puede definir el asma controla- 5. Añadir a lo anterior corticoides orales y/o anti-IgE.
da, parcialmente controlada, o incontrolada (ver tabla 2). Su
tratamiento sería:

53
Manual ENARM · Neumología y Cirugía Torácica

LEVE MODERADA GRAVE

TRABAJO RESPIRATORIO… Al caminar Al hablar En reposo

AL HABLAR NO TERMINA... Párrafos Frases Palabras

FRECUENCIA RESPIRATORIA Aumentada Aumentada >30/min

USO DE MÚSCULOS No es habitual Habitual Fatiga, movimientos paradójicos


ACCESORIOS
Moderadas,
SIBILANCIAS al final de la espiración
Intensas Silencio auscultatorio

FRECUENCIA CARDIACA <100 100-120 >120

PULSO PARADÓJICO Ausente o <10 mmHg Posible, 10-25 mmHg Frecuente, >25 mmHg

PEF (% DEL TEÓRICO) 80% 60-80% <60

PEF (VALOR ABSOLUTO) >300 l/min 150-300 l/min <150 l/min

PaCO2 <45 mmHg <45 mmHg >45 mmHg

PaO2 Normal >60 mmHg <60 mmHg

Sat O2 >95% 91-95% <90%

Tabla 3. Clasificación de la gravedad de la crisis asmática.

Crisis asmática intravenosa. Se debe reevaluar el PEF a los 15-30 min para
Las exacerbaciones (ataques o crisis) de asma son episodios asegurarse de la evolución, y posteriormente de acuerdo con
agudos o subagudos caracterizados por un aumento progresi- la respuesta del paciente (si es >80% del teórico a las 2 horas
vo de uno o más de los síntomas típicos (disnea, tos, sibilancias y se mantiene por 3-4 horas), podrá considerarse su egreso,
y opresión torácica) acompañados de una disminución del flujo de lo contrario, deberá hospitalizarse. Los anticolinérgicos son
espiratorio (PEF o FEV1). La valoración de la gravedad de una útiles en las reagudizaciones graves. Los mucolíticos pueden
crisis asmática se realiza mediante datos clínicos, gasométri- empeorar la tos y por vía nebulizada provocar broncoespasmo,
cos y la medición del PEF, lo que permite clasificar las crisis por lo que están contraindicados.
asmáticas en leves, moderadas y graves (ver tabla 3). Los antibióticos se emplean únicamente si hay evidencia de
Son signos de gravedad la presencia de disnea en reposo proceso infeccioso. La epinefrina (vía intramuscular) solo está
moderada, la participación de músculos accesorios, sibilancias indicada en crisis asmática asociada a anafilaxia o angioedema.
intensas, diaforesis, pulso paradójico mayor a 10-25 mmHg, Una vez que el paciente sea dado de alta, se recomiendan
taquicardia mayor de 120 lpm y taquipnea mayor de 30 rpm. prednisona 40-60 mg vía oral por 7-10 días, corticosteroides
inhalados, beta2 agonista de acción prolongada y rescate con
Son signos de extrema gravedad o de riesgo vital inmediato, beta2 agonista de acción corta.
y por lo tanto de ingreso en UCI, la disnea muy intensa, la
cianosis, bradicardia, bradipnea, hipotensión, el movimiento
paradójico toracoabdominal, el silencio auscultatorio y la dis-
minución del nivel de conciencia. En esta situación está contra- Valorar clínica y PEF
indicada la ventilación mecánica no invasiva (retrasa una situa-
ción generalmente inevitable y empeora el pronóstico vital) y
se debe proceder a la intubación orotraqueal del paciente para Crisis asmática Crisis asmática Crisis asmática
su ventilación invasiva. leve PEF >80% moderada PEF 60-80% grave PEF <60%
El fármaco de elección para el tratamiento de las crisis asmáti-
cas es un ß2-adrenérgico de acción corta por vía inhalada, aña- β-agonista inhalado y
diéndose corticoides sistémicos si la crisis es moderada o grave β-agonista inhalado Valorar UCI
corticoides intravenosos
(incluso en niños). Cuando la vía aérea es poco permeable se
pueden usar ß2-adrenérgicos por vía subcutánea e incluso vía Figura 5. Manejo de la crisis asmática.

54
Tema 5 · Asma

Recuerda que... Recuerda que...


Recuerda que en el diagnóstico del asma se debe demostrar: Ante un paciente cuya única sintomatología es la tos crónica, se
- Reversibilidad: aumento del FEV1 un 12% tras broncodilatadores debe hacer el diagnóstico diferencial entre:
- Hiperreactividad: disminución del FEV1 un 20% tras el test de - Asma
metacolina o el ejercicio - RGE
- Variabilidad: se demuestra mediante el PEF - Patología de ORL (goteo nasal posterior)

Con respecto al diagnóstico también debes saber que una espiro- Para ello se sigue el siguiente plan diagnóstico de forma
metría normal no descarta el diagnóstico de asma pero un test de escalonada, ante la normalidad de las pruebas previas:
metacolina normal sí lo descarta. 1. Abandono del tabaco y de los fármacos tusígenos
2. Radiografía de tórax (suele ser normal en el asma, RGE y
No confundas las clasificaciones: patología ORL)
- Del asma crónico (intermitente/persistente leve, moderada, 3. Espirometría con test broncodilatador
severa): según criterios clínicos y espirométricos 4. Test de metacolina
- De la crisis asmática (leve/moderada/severa): según el PEF 5. pHmetría de 24 horas
6. TC, broncoscopia
En un caso clínico, sospecha un asma persistente moderada ante
la aparición de síntomas nocturnos o síntomas que aparecen con
las actividades habituales.

Pregunta ENARM Pregunta ENARM


10. Mujer de 45 años, fumadora de 20 años, actual- 11. Paciente femenino de 16 años de edad que pre-
mente en estudio por aparente neumopatía senta un cuadro repetitivo de dificultad respira-
crónica. Para hacer diagnóstico diferencial entre toria, de predominio matutino y nocturno, así
asma y EPOC, el dato fundamental de la espi- como sibilancias espiratorias al realizar actividad
rometría que nos confirmará el diagnóstico de física de larga evolución. Se le realiza una espiro-
asma es: metría y se le diagnostica asma. ¿A qué reacción
de hipersensibilidad se asocia el asma?
A. Patrón obstructivo no reversible.
B. Patrón restrictivo no reversible. A. Tipo I mediada por IgE.
C. Patrón obstructivo reversible. B. Reacción de hipersensibilidad mediada por anticuerpos.
D. Patrón restrictivo reversible. C. Mediada por inmunocomplejos insolubles.
D. Tipo IV, de respuesta celular.

· Encuentra las respuestas y comentarios de las preguntas ENARM al final del manual ·

55
Tema 6
Tromboembolismo pulmonar
Factores de riesgo Diagnóstico
Más del 90% de los casos de tromboembolismo pulmonar El abordaje diagnóstico inicial incluye la anamnesis, explora-
(TEP) se deben a trombosis en el sistema venoso profundo ción física, radiografía de tórax, gasometría arterial y ECG.
(TVP) en las extremidades inferiores (principalmente pantorrilla, Con estos datos se construye la sospecha clínica de TEP,
con extensión del trombo hacia venas proximales, por arriba que permite iniciar tratamiento si dicha sospecha es muy alta.
de rodilla y muslo). A su vez, al menos 40% de los casos de El diagnóstico definitivo de TEP se consigue mediante la
TVP proximal tendrán una TEP, que puede ser asintomática. Se angioTAC, la gammagrafía o la angiografía. Otras pruebas que
ve favorecido fundamentalmente por el inmovilismo, la estasis pueden ser útiles en el diagnóstico son la ecografía Doppler
venosa y estados procoagulantes. Entre los principales proce- de miembros inferiores y la determinación de D-dímeros, que
sos que favorecen el TEP destacan: fractura de extremidades evalúan la existencia de enfermedad tromboembólica pero no
inferiores, postcirugía, posparto, reposo prolongado en cama, de TEP en particular.
insuficiencia cardiaca, insuficiencia venosa profunda crónica, - Radiografía de tórax.
obesidad, cáncer activo (el que más frecuentemente se asocia Lo más frecuente es que sea inespecíficamente anormal,
al TEP es el adenocarcinoma de pulmón), hiperestronismo, siendo la anomalía más frecuente la elevación del hemidia-
embarazo, anticonceptivos, niveles elevados de anticuerpos fragma; otras son atelectasias laminares o pérdida de volu-
antifosfolípidos, anticoagulante lúpico y otros estados de men. Las manifestaciones específicas de TEP son el signo de
hipercoagulabilidad, como el déficit de antitrombina III, factor Westermark (hiperclaridad pulmonar debida a oligohemia
V de Leiden (trastorno de coagulación más frecuente en la focal), la asimetría de las arterias pulmonares (arteria pulmo-
población), o el déficit de las proteínas C o S. No obstante, el nar descendente derecha de mayor tamaño) o incluso ampu-
factor de riesgo más frecuente es el antecedente de un episo- tación del hilio pulmonar, y la joroba de Hampton, que es
dio tromboembólico. una condensación pulmonar triangular periférica con base en
Fisiopatológicamente, el TEP produce los siguientes efectos: la superficie pleural, típica del TEP con infarto pulmonar y que
1. Aumento de la resistencia vascular pulmonar. con frecuencia asocia derrame pleural serohemático (infartos
Por reducción del lecho vascular con alteración de la hemodi- periféricos).
námica de cavidades derechas (el fracaso ventricular derecho
es la principal causa de muerte en pacientes con TEP).
2. Hiperventilación alveolar.
Por estímulo de receptores pulmonares de distensión e irri-
tación, con PCO2 normal o disminuida.
3. Alteración en la relación ventilación/perfusión.
Por el aumento del espacio muerto alveolar, broncoconstric-
ción de pequeñas vías aéreas y colapso alveolar por pérdida
de surfactante en zonas hipóxicas, con la consiguiente hi-
poxemia.
4. Reducción de la distensibilidad pulmonar.
Por acumulación de líquido en las zonas afectadas.
5. Alteración del intercambio gaseoso.
Debida al aumento del espacio muerto, discordancia V/Q,
aparición de shunt derecha-izquierda y alteración de la difu-
sión (por el descenso de la superficie de intercambio).

Clínica
El síntoma más frecuente es la disnea de aparición brusca e
inexplicable, en ocasiones acompañada de taquipnea (signo
más frecuente) y taquicardia. El síncope es raro, revela altera-
ción hemodinámica transitoria. Puede ser asintomático.
En el TEP con obstrucción >50% de la arteria pulmonar prin- Figura 1. Signo de Westermark, con hiperclaridad en hemitórax superior derecho.
cipal o con oclusión de varias ramas lobares puede existir,
además, dolor retroesternal, galope ventricular derecho o
- Gasometría arterial.
desdoblamiento del segundo tono.
Suele existir hipoxemia (consecuencia de la pérdida de volu-
Cuando existe dolor pleurítico, hemoptisis, febrícula y derrame men e hipoperfusión pulmonares, la insuficiencia ventricular
pleural hemático, hay que sospechar complicación con infarto derecha y la disminución del gasto cardiaco) e hipocapnia.
pulmonar (normalmente periférico). Asimismo, es frecuente la existencia de un aumento del gra-
El TEP crónico puede cursar exclusivamente con disnea. Ante diente alveoloarterial de oxígeno.
una hipoxemia con hipertensión pulmonar, radiografía de - Electrocardiograma.
tórax normal y pruebas funcionales respiratorias normales, Lo más frecuente es que sea inespecíficamente anormal; la
hemos de descartar un TEP crónico. alteración más común es la taquicardia sinusal. Si el TEP es
extenso, aparecen signos de sobrecarga derecha: desviación

56
Tema 6 · Tromboembolismo pulmonar

del eje a la derecha, patrón SI-QIII-TIII (es lo más específico),


bloqueo de rama derecha y ondas “P pulmonale”.

VD VI VD

VI

Figura 4. TEP masivo con dilatación del ventrículo derecho (VD), que comprime
secundariamente al ventrículo izquierdo (VI). Izquierda: proyección apical 4
Figura 2. Patrón SI-QIII-TIII. cámaras. Derecha: proyección paraesternal eje corto.

- Gammagrafía de ventilación/perfusión.
- Determinación de dímero-D mediante ELISA. Prueba de elección cuando la angioTC está contraindicada.
Es la primera prueba a realizar cuando la sospecha clínica de Es de alta probabilidad diagnóstica la existencia de dos o más
TEP es baja. Tiene un elevado valor predictivo negativo (cer- defectos segmentarios de perfusión moderados o extensos,
cano al 100%), de forma que ayuda a excluir el diagnóstico con ventilación normal. No es interpretable en pacientes con
de TEP si resulta negativo. De acuerdo con la GPC, un valor patología pulmonar, como EPOC, en los que coinciden defec-
positivo se considera mayor de 1000 mcg/L en personas con tos de perfusión y ventilación, siendo necesaria entonces la
baja probabilidad clínica, y mayor de 500 mcg/L en casos de realización de una angioTC o una arteriografía.
riesgo mayor. Si es positivo, los D-dímeros pueden estar eleva- La gammagrafía de perfusión aislada es tan solo test de ta-
dos por otras causas que producen también fibrinólisis endó- mizaje, muy útil para descartar un TEP agudo clínicamente
gena como el IAM, la cirugía, los carcinomas, o casi cualquier importante. Una gammagrafía de ventilación/perfusión de
enfermedad sistémica. Esta determinación está disponible en baja probabilidad en un paciente con alta sospecha clínica no
casi todos los servicios de urgencias y se realiza rápidamente y descarta el diagnostico.
con un bajo coste.
- AngioTC con contraste: prueba de elección ante la sos-
pecha de TEP.
Es útil para visualizar TEP centrales (de grandes arterias) pero
tiene dificultades para detectar trombos periféricos. Tiene una
sensibilidad similar a la gammagrafía para descartar TEP (S
80%, E 90%) pero aporta información añadida sobre otras
entidades diagnósticas (p. ej., valorar la presencia de disec-
ción de aorta ante un cuadro de dolor torácico con elevación
de D-dímeros, compatible tanto con TEP como con síndrome
aórtico agudo).
Está especialmente indicada en pacientes con inestabilidad
hemodinámica. La GPC la recomienda como el método de
elección para valorar la vascularidad pulmonar en pacientes
con sospecha de TEP.
Debe evitarse en insuficiencia renal (es relativamente segura
con niveles de creatinina <2 mg/dl), durante el embarazo y en
alérgicos a contrastes yodados. Figura 5. Gammagrafía de ventilación/perfusión en una paciente con embolia
pulmonar masiva bilateral. Obsérvense los múltiples defectos de perfusión (P)
en zonas correctamente ventiladas (V).

- Angiografía pulmonar.
Es la prueba de certeza o gold standard, es decir, la más sen-
sible y específica. Actualmente tiene poco uso al ser susti-
tuida por métodos de diagnóstico menos invasivos. Permite
establecer el diagnóstico definitivo de TEP mediante la visua-
lización de defectos de llenado intraluminal en más de una
proyección. Su mayor utilidad aparece cuando la probabilidad
clínica de TEP difiere del resultado de la gammagrafía pulmo-
nar, o cuando ésta tiene una probabilidad intermedia de TEP.
Cuando la sospecha clínica sigue siendo alta y las pruebas
diagnósticas negativas, debe realizarse una angiografía.
- Tanto la angiografía pulmonar como el angioCT con contraste
están contraindicados en la insuficiencia renal y en pacientes con
Figura 3. AngioTC que muestra un TEP bilateral que afecta a las dos arterias alergia a contrastes yodados, siendo de elección en estos casos
pulmonares principales (flechas). para el diagnóstico la gammagrafía de ventilación/perfusión.

57
Manual ENARM · Neumología y Cirugía Torácica

Abordaje diagnóstico inicial / Sospecha clínica

Baja Intermedia Anticoagulación Alta

Técnica de imagen

Dímero-D Gammagrafía V/Q angio TAC Eco transtorácico

Negativo Positivo Baja probabilidad Alta probabilidad Negativo Positivo Sin disfunción Disfunción del VD
del VD
Descarta TEP Confirma TEP Confirma TEP Confirma TEP

ECO-Doppler de extremidades inferiores

Negativo Positivo

Confirma TVP
PC baja PC alta

Descarta TEP Arteriografía

Figura 6. Algoritmo diagnóstico del tromboembolismo pulmonar. PC (probabilidad clínica).

- Ecografía en la enfermedad tromboembólica venosa:


• La ecografía venosa de miembros inferiores es la prueba CARACTERÍSTICA CLÍNICA PUNTOS
más disponible ante el paciente con sospecha de TEP.
Síntomas clínicos de TVP 3
No sirve para hacer el diagnóstico de TEP, pero sí de TVP,
Otros diagnósticos menos probables que TEP 3
que a su vez apoya el diagnóstico de TEP. Posee una muy
Frecuencia cardiaca mayor de 100 latidos x minuto 1,5
buena relación coste-efectividad ante pacientes con alta sos-
Inmovilización o cirugía dentro de las últimas 4 sem. 1,5
pecha de TVP. La ecografía de miembros inferiores es de
TVP o TEP previo 1,5
elección en mujeres embarazadas con sospecha de TEP.
Hemoptisis 1
• La ecocardiografía se emplea ante pacientes con sospecha
Malignidad 1
de TEP y clínicamente graves.
Permite visualizar trombos importantes (baja sensibilidad) y Probabilidad de TEP:
signos de disfunción del ventrículo derecho, lo que permite >6 puntos: riesgo elevado
estratificar el riesgo y definir el pronóstico. 2 a 6 puntos: riesgo moderado
• La ecografía multiórgano es especialmente valiosa en la <2 puntos: riesgo bajo
valoración inicial de sospecha de tromboembolismo en pa-
cientes críticos (con inestabilidad tan importante que contra-
indique moverlos del Box Vital), embarazadas y alérgicos a Tabla 1. Criterios de Wells para el diagnóstico de TEP.
contrastes/ insuficiencia renal).
Se trata de realizar una ecografía pulmonar (buscando con-
solidaciones subpleurales típicas del TEP), un ecocardiograma Recuerda que...
(buscando disfunción del ventrículo derecho) y ecografía de - Gold standard – Angiografía pulmonar.
las venas de las piernas en Urgencias, a pie de cama. Está - De elección – AngioTAC de tórax.
emergiendo como una posible alternativa al TC torácico (con - 1er estudio en baja sospecha – Dímero D.
una S >90% y una E = 86%), especialmente en los casos - En insuficiencia renal y otras contraindicaciones para angioTAC –
arriba indicados. Gammagrafía perfusión/ ventilación (no recomendada en pacientes
- Criterios de Wells para el diagnóstico de TEP. con patología pulmonar).
De acuerdo a la GPC, se recomienda utilizar el Modelo clí-
nico de Wells para establecer la probabilidad de diagnóstico
de Tromboembolismo pulmonar. Pronóstico
Ante un diagnóstico de TEP, el siguiente paso es evaluar la
gravedad del mismo para determinar si el paciente precisa

58
Tema 6 · Tromboembolismo pulmonar

solamente tratamiento anticoagulante (heparina), o si se puede El rango terapéutico se consigue prolongando el aPTT de 1,5
beneficiar de medidas más agresivas como la fibrinólisis o la a 2,5 veces el valor del control. Las HBPM se administran vía
trombectomía. La principal causa de mortalidad en estos subcutánea y no precisan de controles.
pacientes es la disfunción ventricular derecha secundaria El tratamiento inicial con heparina se mantiene durante 5-7 días.
a la hipertensión pulmonar producida por el TEP. Es por ello
El tratamiento a largo plazo se realiza con anticoagulación
que debemos evaluar el estado del ventrículo derecho. En
oral (ACO), bien con dicumarínicos (acenocumarol o war-
el paciente estable, el primer paso es la determinación de los
farina, manteniendo un INR entre 2-3), o bien con nuevos
niveles de troponina y BNP o ProBNP, y en el caso de resultar
anticoagulantes orales (NACO: dabigatran, rivaroxaban,
positivos, la realización de un ecocardiograma. En el paciente
apixaban, edoxaban). No se inicia tratamiento directo con
inestable se realiza directamente el ecocardiograma.
dicumarínicos aislados por tener un efecto protrombótico
El TEP se clasifica en tres categorías pronósticas: en las primeras 24 horas y tardar 3-5 días en alcanzar rango
- TEP de alto riesgo (antes llamado masivo). terapéutico, pero los NACO sí se pueden usar sin tratamiento
TEP que se presenta con inestabilidad hemodinámica (TAS inicial con heparina.
<90 mmHg durante más de 15 minutos) o síncope. La duración del tratamiento depende de los factores de riesgo:
- TEP de riesgo intermedio (antes llamado submasivo).
- Primer episodio y causa reversible.
Estabilidad hemodinámica pero disfunción ecográfica de ven-
3-6 meses.
trículo derecho, marcadores de daño miocárdico elevados
- Episodios recurrentes o causa intratable.
(troponina, BNP), o puntuación alta en escala clínica de valo-
La anticoagulación se mantiene de por vida. En el paciente
ración de riesgo (PESI).
oncológico se prefieren las heparinas a la ACO en caso de que
- TEP de bajo riesgo (antes llamado leve).
esté en tratamiento con QT.
Estabilidad hemodinámica sin datos ecográficos de disfunción
de VD, ni analíticos de sobrecarga miocárdica, y con puntua-
ción baja en escala PESI. Son contraindicaciones absolutas para la anticoagulación el
sangrado activo (pero no la hemoptisis, que puede formar
parte del cuadro clínico, ni el derrame pleural serohemático,
La escala PESI (Pulmonary Embolism Severity Index) valora
propio del TEP que asocia infarto pulmonar periférico), la HTA
edad, sexo, FC, FR, PA, satO2, Tª, presencia de insuficiencia
severa, la hemorragia intracraneal, la presencia de aneurisma
cardiaca, estado mental alterado, EPOC y cáncer; es la más vali-
intracraneal, y la cirugía intracraneal, retiniana o medular
dada para establecer el riesgo de defunción tras un tromboem-
reciente.
bolismo pulmonar. Ofrece 5 clases de riesgo; considerándose
de bajo riesgo las clases I y II.
TEP de alto riesgo
Tratamiento El tratamiento de elección son los trombolíticos, siempre que
no exista alto riesgo de sangrado. Los trombolíticos se inician
Si existe una sospecha clínica media o alta y el estudio de una vez se llegue al diagnóstico de certeza (si se inicia trata-
imagen se puede demorar más de 4 horas, se debe iniciar miento antes debe ser anticoagulación) y los más empleados
antes del diagnóstico de certeza (e independientemente son los derivados recombinantes del activador tisular del plas-
del estado del paciente) tratamiento anticoagulante con minógeno (reteplase, alteplase, tenecteplase). Posteriormente
heparina. se continúa la anticoagulación con HNF y después con ACO.
En los TEP masivos que no responden a la trombólisis se debe
TEP de riesgo bajo o intermedio realizar fragmentación mecánica con tromboembolectomía
El tratamiento va dirigido a la prevención secundaria (evitar (cirugía muy agresiva mediante una esternotomía y extracción
nuevos episodios de TEP) mediante anticoagulación, inicial- del trombo) o una trombectomía endoarterial (lisis del trom-
mente con heparina; la fibrinolisis no está indicada. En el bo in situ mediante cateterismo arterial).
TEP de bajo riesgo puede darse de alta al paciente de manera
precoz y hacer manejo ambulatorio, mientras que en el TEP de Filtro de vena cava inferior
riesgo intermedio debe realizarse vigilancia hospitalaria para En los casos de contraindicación para la anticoagulación, TEP
poder detectar precozmente inestabilización hemodinámica recurrentes a pesar de la anticoagulación y presencia de gran
(que sí sería indicación de fibrinólisis). trombo flotante en la vena cava inferior, se procederá a la
La heparina no fraccionada (HNF) y las heparinas de bajo peso colocación de un filtro en la cava inferior o a la ligadura de
molecular (HBPM) son igual de eficaces y seguras, por lo que la misma. La ligadura de la vena cava inferior también se realiza
pueden usarse una u otra indistintamente, con dos excepcio- en las tromboflebitis sépticas de origen pélvico.
nes:
- Pacientes con alto riesgo de sangrado.
Se prefiere el uso de HNF, dado que tiene un efecto rápida- Recuerda que...
mente reversible con sulfato de protamina. Ante la sospecha clínica alta de TEP debe iniciarse anticoagulación
- Embarazadas. inmediatamente con heparina, antes de confirmar el diagnóstico.
Se prefiere utilizar HBPM, ya que posteriormente continuará En cambio, para iniciar fibrinólisis, ante una sospecha de TEP de
la anticoagulación con HBPM en vez de con anticoagulantes alto riesgo (masivo) es necesario confirmar el diagnóstico antes de
orales. En el embarazo están contraindicados los anticoagu- instaurar tratamiento con fibrinolíticos, dado el elevado riesgo de
lantes orales durante el primer trimestre y en las semanas pre- sangrado que producen éstos.
vias al parto, pero no la heparina.

La HNF se administra vía intravenosa y requiere monitorización (Ver figura 7 en la página siguiente)
mediante el tiempo de tromboplastina parcial activado (aPTT).

59
Manual ENARM · Neumología y Cirugía Torácica

¿Estabilidad hemodinámica?

Estable Inestable

¿Contraindicacion ¿Contraindicacion
anticoagulación? fibrinólisis?

Sí No No Sí

- AC inicial con heparina Tromboembolectomía


(HNF o HBPM) (quirúrgica)
Filtro de vena cava - AC definitiva con ACO Fibrinólisis
Trombectomía endoarterial
para INR 2-3 (percutánea)

¿Estabilidad hemodinámica?
Sí No

¿Contraindicacion anticoagulación? Se estabiliza Persiste inestable

Figura 7. Algoritmo terapéutico del tromboembolismo pulmonar.

Prevención del TEP


Está indicada tromboprofilaxis en pacientes con procesos médi-
cos o quirúrgicos de alto riesgo para desarrollar enfermedad
tromboembólica venosa. Se debe incentivar la deambulación
en cuanto sea posible. Según el perfil de riesgo, basta con
medidas físicas (compresión neumática intermitente, medias
de compresión elástica) o puede ser precisa anticoagulación
profiláctica con heparina (típicamente, enoxaparina 0.2-1 mg/
kg/día vía SC), heparina no fraccionada a dosis baja (5000 U
cada 8-12 horas vía SC), fondaparinux (2.5 mg/día vía SC),
NACO, aspirina (100 mg/día vía oral) o Warfarina (meta de
INR entre 1.5-2.5). Si hay contraindicación, solo se indicarán
medidas físicas. Es precisa la tromboprofilaxis mientras dure el
inmovilismo, pero puede ser recomendable hasta 4 semanas
tras la cirugía (sobre todo, fractura de cadera).
Estas medidas se emplean para evitar la trombosis venosa pro-
funda, lo que a su vez previene la aparición de TEP, por lo que
se denominan globalmente “prevención de la ETEV”.

60
Tema 6 · Tromboembolismo pulmonar

Pregunta ENARM Pregunta ENARM


12. Mujer de 71 años, previamente sana, que ayer Mujer de 58 años de edad, postoperada de safenec-
tuvo fractura de cadera tras una caída. Se encon- tomía. Se encuentra con sospecha de tromboembolia
traba esperando tiempo quirúrgico cuando desa- pulmonar, de baja probabilidad.
rrolló disnea súbita, hipotensión y desaturación
de oxígeno hasta 72% (SO2 basal de 91%). ¿Cuál 14. El estudio que tendrá el mayor valor predictivo
es el hallazgo electrocardiográfico más probable negativo para descartar el diagnóstico probable
en este caso? de la paciente es la determinación de:

A. Taquicardia sinusal. A. Monómeros.


B. S1Q3T3. B. Dímero D.
C. Bloqueo de rama derecha. C. Fibrinógeno sérico.
D. Taquicardia helicoidal. D. Transaminasa glutámico oxalacética.

15. Continuando con el abordaje diagnóstico de la


paciente, se corrobora el diagnóstico de trom-
Pregunta ENARM boembolia pulmonar aguda. Se mantiene hemo-
13. El diagnóstico de certeza de la tromboembolia dinámicamente estable. ¿Cuál es el tratamiento
pulmonar se establece por medio de: de elección para esta paciente?

A. Resonancia magnética nuclear. A. Anticoagulación con heparina.


B. Gammagrama pulmonar ventilatorio perfusorio. B. Trombólisis in situ.
C. Pruebas de función respiratoria. C. Antiagregación plaquetaria.
D. Angiografía pulmonar. D. Terapia endovascular.

· Encuentra las respuestas y comentarios de las preguntas ENARM al final del manual ·

61
Tema 7
Trastornos de la ventilación

7.1. Hipoventilación alveolar Hipoventilación alveolar primaria


Clínica
La hipoventilación alveolar produce como trastornos gaso- Se debe a alteración de la sensibilidad del centro ventilatorio,
métricos hipercapnia e hipoxemia, salvo que se respire aire con descenso del impulso ventilatorio involuntario. Durante el
enriquecido en oxígeno en intensidad proporcional a la hiper- día mantienen ventilación normal de forma voluntaria, pero
capnia concomitante (pues el CO2 desplaza al O2). La diferen- por la noche presentan hipoventilación y pueden incluso hacer
cia alveoloarterial de oxígeno suele ser normal, salvo que haya pausas centrales de apnea (“maldición de Ondina”). Es más
trastorno parenquimatoso acompañante. Sus repercusiones frecuente en varones de edad media y se ve favorecida por
dependen del exceso de CO2 y de la rapidez con la que el gas la obesidad. Generalmente se diagnostica cuando se produ-
se ha retenido (hipoventilación alveolar aguda y crónica). ce una depresión respiratoria grave tras la administración de
sedantes o anestésicos.
Clínica y diagnóstico
La gasometría arterial basal, además de mostrar la hiper- Diagnóstico
capnia, sugiere si se trata de un proceso agudo o crónico Suelen presentar hipercapnia e hipoxemia crónica, pero tienen
(aumento del HCO3) o si se trata de un trastorno mixto del capacidad para hiperventilar voluntariamente, por lo que pue-
equilibrio ácido-base. den no mostrar hipercapnia al realizar una gasometría durante
el día; no obstante, siempre tendrán una alcalosis metabólica
compensadora, con bicarbonato elevado.
Hipercapnia aguda
Suele deberse a una insuficiencia respiratoria aguda (IRA).
Produce trastornos del SNC (mareo, desorientación temporoes- Tratamiento
pacial, obnubilación, coma y muerte) y una respuesta cardio- La mayoría requiere ventilación mecánica de tipo BiPAP. En
vascular mixta: vasoconstricción (por hiperestimulación simpá- ocasiones se recurre a marcapasos diafragmático.
tica) y vasodilatación (por la acción local del CO2). Puede existir
sudoración, taquicardia y normo, hipo o hipertensión arterial.
7.2. Hiperventilación alveolar
Hipercapnia crónica
Casi siempre está relacionada con una insuficiencia respiratoria Clínica
crónica (IRC), por lo que mecanismos de compensación permi- Gasométricamente produce hipocapnia y, si la situación de
ten tolerar niveles de PaCO2 de hasta 100 mmHg (que serían hiperventilación es crónica, el bicarbonato estará disminuido
mortales de forma aguda). Suelen existir cefalea, mareo, sen- como consecuencia de la compensación renal de la alcalosis
sación de embotamiento, somnolencia, asterixis y papiledema respiratoria. Las consecuencias clínicas más relevantes depen-
(por la vasodilatación cerebral, que puede inducir a confusión den de la vasoconstricción cerebral y de la alcalosis respiratoria:
con un tumor cerebral). debilidad y vértigo, visión borrosa, opresión torácica, disnea
–síntoma más frecuente–, ansiedad, parestesias periorales y en
Tratamiento extremidades, confusión mental e incluso espasmos muscula-
res y tetania por la hipocalcemia, y síncope o incluso parada
Se basa en tratamiento etiológico de la IRA o IRC. Mientras en la
respiratoria.
IRA es frecuente recurrir a la ventilación mecánica endotraqueal,
en los cuadros de hipoventilación crónica el tratamiento es
menos agresivo: conservador (EPOC), ventilación no invasiva por Diagnóstico
BiPAP (enfermedades neuromusculares o deformidades de la Es fundamental establecer un diagnóstico etiológico, pues la
caja torácica) o CPAP (en los trastornos ventilatorios del sueño). causa de una hiperventilación puede ser grave. Las situaciones
que con mayor frecuencia producen hiperventilación inexpli-
Síndrome de obesidad-hipoventilacion (SOH) cable son la ansiedad y el TEP recurrente. Para diferenciarlas,
podemos calcular la diferencia alveoloarterial de O2, que en
También denominado síndrome de Pickwick. Es una de las
el segundo caso se hallará elevada. Los pacientes que hiper-
causas más frecuentes de hipoventilación. La obesidad con-
ventilan por enfermedad vascular pulmonar presentan disnea
diciona un trastorno ventilatorio restrictivo, con insuficiencia
de esfuerzo y el gradiente alveoloarterial de O2 está elevado,
ventilatoria crónica. Suelen presentar insuficiencia respiratoria
mientras que los pacientes con hiperventilación psicógena
crónica hipercápnica, bien tolerada. Puede provocar sobrecar-
presentan disnea fundamentalmente en reposo con gradiente
ga del ventrículo derecho y favorece el desarrollo de insuficien-
alveoloarterial de O2 normal, y la hiperventilación suele desa-
cia cardiaca. Es frecuente la asociación con apnea obstructiva
parecer con el ejercicio.
del sueño (la obesidad favorece el colapso de la via aérea supe-
rior), y con hipoventilación alveolar primaria (favorecida por la
obesidad, y que puede causar apneas centrales del sueño). Su Tratamiento
tratamiento etiológico es la pérdida de peso, puede emplearse Es el etiológico. En la hiperventilación psicógena (lo más fre-
la progesterona como analéptico respiratorio, y como soporte cuente son crisis de ansiedad) es útil hacer respirar al enfermo
ventilatorio está indicada la VMNI con BiPAP en una bolsa cerrada de plástico (inhalación de una concentra-

62
Tema 7 · Trastornos de la ventilación

ción baja de CO2), porque al mejorar la alcalosis respiratoria Recuerda que...


remiten los síntomas y se consigue control de la crisis de
ansiedad. Los siguientes son factores de riesgo que deben orientarte a
indagar SAOS: IMC >30 kg/m2, cardiopatías, DM2, antecedente
de evento vascular cerebral, hipertensión pulmonar, choferes o
conductores de vehículos.
7.3. Síndrome de apnea del sueño

La apnea del sueño consiste en la presencia de episodios de


cese del flujo aéreo nasobucal, de duración igual o superior a
10 segundos, durante el sueño. La hipopnea consiste en una
reducción del flujo aéreo mayor del 30%, junto a una desatu-
ración de oxígeno cíclica igual o superior al 4%.
En sujetos normales se pueden producir hasta diez apneas-
hipopneas por cada hora de sueño (índice apnea-hipopnea:
IAH). Se considera patológico un IAH >10/hora, y diagnóstico
de SAOS >15/hora.
Las apneas pueden ser de tres tipos:
- Centrales.
La interrupción del flujo aéreo se produce debido a una pausa
en los movimientos respiratorios toracoabdominales por cese
del estímulo ventilatorio.
- Obstructivas.
Ocurren en las fases de sueño profundo (III-IV) y en el sueño
REM. Son debidas a la atonía muscular que ocurre durante
estas fases del sueño, con lo que las estructuras de la faringe
e hipofaringe se relajan y obstruyen la vía aérea superior. Este
tipo de apnea es la más frecuente.
- Mixtas.
Via aérea bloqueada
Consisten en episodios centrales seguidos de otro episodio
obstructivo. Suelen ocurrir en las fases de sueño REM (y al
principio de las fases no-REM). Figura 1. Patogenia de la apnea obstructiva del sueño.

Síndrome de apnea obstructiva del sueño (SAOS)


Mecanismo productor y factores de riesgo Clínica
Las apneas obstructivas se deben a oclusiones inspiratorias En México la prevalencia ha aumentado en los últimos años en
repetitivas de la vía aérea superior (VAS), durante el sueño. La pacientes con IMC >30 kg/m2.
aparición de apneas está relacionada con la atonía muscular Predomina en varones obesos de edad media y en mujeres
que se produce en las fases del sueño profundo (fases III/IV y posmenopáusicas. Presenta tanto síntomas nocturnos, como
REM), lo que aumenta la resistencia al paso del aire y colapsa las apneas, ronquidos, movimientos corporales bruscos y
las paredes de la faringe. Pueden contribuir factores anató- microdespertares o arousal, como diurnos, siendo la manifes-
micos: obstrucción nasal, macroglosia, hipertrofia amigdalar, tación más frecuente la somnolencia excesiva. La valoración de
micro/retrognatia, trastornos neuromusculares orofaríngeos, la somnolencia se realiza mediante la escala de Epworth, que
obesidad, hipotiroidismo, acromegalia. consiste en un test que evalúa la probabilidad de quedarse dor-
Las apneas, hipopneas o el aumento de resistencia en las VAS mido en diferentes situaciones. La escala es de 0 a 24, donde
dan lugar a un breve despertar o arousal (que no debe con- 0 es no tener ninguna probabilidad de quedarse dormido y
fundirse con el despertar normal del sueño, del que el paciente 24 es tener mucha somnolencia diurna. Se considera excesiva
no es consciente y no interrumpe el sueño). Este arousal man- somnolencia una puntuación de 10-15.
tiene al paciente en fases I y II del sueño en la mayor parte de Además, es común que estos pacientes presenten trastornos
la noche (sueño no profundo y no reparador) y restablece el neuropsiquiátricos (como depresión y agresividad), intelectua-
tono de los músculos dilatadores de la VAS. El paciente realiza les o reducción de la libido, mediados por la fragmentación del
varias respiraciones profundas, la PaO2 y la PaCO2 vuelven a sueño y la falta de sueño profundo. Pueden presentar cefalea
sus niveles basales y el paciente se vuelve a dormir, reinicián- matutina, debida a la vasodilatación cerebral ocasionada por la
dose el ciclo. hipercapnia nocturna.
Es frecuente la asociación de SAOS con hipoventilación alveo- Las posibles complicaciones del síndrome de apnea del sueño
lar primaria, dada la asociación de ambos con la obesidad, por incluyen: hipertensión arterial, hipertensión pulmonar, polici-
lo que pueden coexistir apneas centrales con las obstructivas. temia secundaria y arritmias cardiacas durante el sueño (posi-
Por ello, hoy día se tiende a hablar de Síndrome de Apnea- blemente con aumento del riesgo de muerte súbita nocturna).
Hipopnea del Sueño (SAHS). Aumenta el riesgo cardiovascular, favoreciendo el infarto y la
insuficiencia cardiaca.

63
Manual ENARM · Neumología y Cirugía Torácica

Diagnóstico Síndrome de apnea central del sueño


El diagnóstico definitivo del síndrome de apnea Representa un 10% de los casos con síndrome de apnea del
del sueño se realiza mediante polisomnografía sueño y se produce por un fallo transitorio del estímulo central
nocturna o poligrafía cardiorrespiratoria noctur- dirigido a los músculos respiratorios. Aparece en trastornos de
na, considerándose como diagnóstico de SAOS diversa índole (metabólicos, neurológicos, farmacológicos, etc.)
un IAH mayor o igual a 15. cuyo denominador común es una alteración en el control de la
respiración: situaciones que desencadenen una respiración de
Cheyne-Stockes, parálisis diafragmática e incluso presencia de
Tratamiento
alteraciones de las VAS. Ello explicaría por qué en ciertos casos
- Medidas generales. la CPAP nasal es útil en el tratamiento de las apneas centrales.
Reducir el sobrepeso y evitar el consumo de alcohol y sedantes
(sobre todo antes de dormir).
- Medidas específicas. Clínica
El tratamiento de elección del SAOS consiste en la aplicación Depende del trastorno subyacente. Cuando el origen son las
nocturna de una presión positiva continua en la vía aérea alteraciones metabólicas o neuromusculares, predominan los
(CPAP), mediante una mascarilla nasal u oronasal. El trata- síntomas propios de la hipoventilación alveolar y la hipercap-
miento nocturno con CPAP mejora la oxigenación y reduce el nia diurna. Por el contrario, los enfermos con inestabilidad en
número de eventos ventilatorios, con lo que mejora el sueño el control del impulso respiratorio mantienen un buen nivel
y las manifestaciones diurnas. de ventilación durante la vigilia (no tienen hipercapnia) y sus
La CPAP está indicada en casos de IAH >30 apneas/hora, y síntomas son fundamentalmente consecuencia de las apneas
también en casos de IAH >15 apneas/hora que asocien mucha nocturnas y alteraciones del sueño (y por tanto los síntomas
somnolencia o comorbilidades (HTA mal controlada, antece- son similares a los del SAOS).
dentes de ictus, IAM, etc.). En los pacientes con IAH <15 o
entre 15-30 sin síntomas se recomiendan sólo las medidas
higiénico-dietéticas generales. Diagnóstico
- Otras medidas, indicadas en casos refractarios, son la uvulo- Se basa en la polisomnografía.
palatofaringoplastia y, excepcionalmente, la traqueostomía,
pero por lo general no dan muy buenos resultados. Tratamiento
- Apnea central del sueño hipercápnica (por alteraciones
metabólicas o neuromusculares).
Pueden emplearse los estimulantes respiratorios (medroxipro-
gesterona) y la oxigenoterapia nocturna (con precaución, ya
que puede prolongar las apneas y empeorar la hipercapnia).
Si fracasan estas medidas médicas o la causa es enfermedad
neuromuscular, está indicada la VMNI con BiPAP.
- Apnea central del sueño no hipercápnica.
En los pacientes con apnea central por inestabilidad en el
impulso, el tratamiento incluye oxígeno nocturno y también
pueden emplearse estimulantes respiratorios del tipo de la
acetazolamida e incluso la CPAP, si con el oxígeno no es sufi-
ciente.

Figura 2. CPAP, presión continua en la vía aérea.

64
Tema 8
Síndrome de dificultad respiratoria aguda
Concepto 1. Instauración dentro de 1 semana desde el inicio de la etiolo-
El síndrome de dificultad respiratoria aguda (SDRA) o distrés gía del SDRA o de síntomas respiratorios nuevos o peores.
respiratorio agudo es un síndrome caracterizado por la presen- 2. Insuficiencia respiratoria grave con cociente PaO2/FiO2
cia de insuficiencia respiratoria aguda grave, disminución <300 (en ocasiones lo han preguntado como PaO2 <55
de la distensibilidad pulmonar e infiltrados pulmonares mmHg con FiO2 >50%, o bien PaO2 <50 mmHg con FiO2
difusos en la radiografía de tórax. >60%), con descenso de distensibilidad.
3. Una causa desencadenante.
Si no hay desencadenante conocido, no puede diagnosti-
Etiología carse SDRA, debiendo valorarse el diagnóstico de neumonitis
El SDRA se debe a una enfermedad inflamatoria pulmonar o interstical aguda o pulmón de Hamman-Rich (se estudia en
sistémica que genera daño alveolar. Las causas más frecuentes el tema 9.2).
de SDRA son: 4. Y la existencia de infiltrados pulmonares bilaterales, no
completamente explicados por derrame pleural, colapso pul-
- Inflamación pulmonar.
monar/lobar o nódulos.
Neumonía grave (la causa más frecuente), embolia grasa, as-
piración de contenido gástrico, casi ahogamiento...
- SRIS. La insuficiencia respiratoria no es completamente explicada por
Sepsis de origen extrapulmonar, politraumatismo, politransfu- insuficiencia cardiaca o sobrecarga de volumen. Actualmente,
sión, gran quemado, pancreatitis aguda... no es imprescindible descartar la presencia de edema pulmo-
- Neumopatía grave. nar cardiogénico. El diagnóstico diferencial incluye otras causas
- Inflamación pulmonar. de infiltrados alveolares bilaterales (como brotes inflamatorios
Neumonía grave, embolia grasa, aspiración de contenido gás- de FPI, neumonitis organizativa criptogenética, hemorragia
trico, ahogamiento... alveolar difusa o neumonía eosinófila aguda).
- Fármacos que aumentan permeabilidad (opiáceos) o inducen El SDRA se diagnostica con cociente PaO2/FiO2 previo a venti-
inflamación pulmonar (nitrofurantoína). lación 300, pero se clasifica según el nivel del cociente PaO2/
FiO2 tras iniciar la ventilación mecánica en SDRA leve (200 a
Fisiopatología 300, con PEEP o CPAP 5 cm2 H2O), moderado (100-199) o
grave (<100).
El SDRA se produce como consecuencia de una respuesta
inflamatoria anormal, desencadenada por diversos estímulos
exógenos, que llegan al pulmón por vía inhalada o sistémica. Clínica
En consecuencia, se produce una lesión de la membrana alveo- El cuadro clínico se caracteriza primeramente por la aparición
locapilar, produciéndose un aumento de su permeabilidad que de taquipnea, junto con taquicardia, sudoración, cianosis,
provoca la extravasación de plasma rico en proteínas hacia el disnea e insuficiencia respiratoria grave. En un principio, la
intersticio y los alvéolos (se lesionan los neumocitos tipo II y se hipoxemia mejora con la oxigenoterapia, pero es típico que,
altera la síntesis del surfactante). Todo ello causa un edema según avanza la enfermedad, se instaure una hipoxemia
intersticial y un colapso alveolar, con las siguientes conse- refractaria al tratamiento con oxígeno suplementario (por el
cuencias: efecto shunt).
- Alteración de la relación ventilación/perfusión y shunt.
- Disminución de la distensibilidad pulmonar (es la alteración más
precoz y constante en la fase aguda), con el consiguiente au- Estudios complementarios
mento del trabajo de los músculos respiratorios, aparición de - Radiografía de tórax.
fatiga muscular y disminución de los volúmenes ventilatorios. Pueden apreciarse en la fase inicial infiltrados intersticiales
- La hipoxia alveolar determina una vasoconstricción arteriolar bilaterales de predominio basal y, posteriormente, infiltrados
pulmonar reactiva que produce un incremento de las resisten- alveolares bilaterales y difusos (forma parte del diagnóstico
cias vasculares y, por tanto, hipertensión pulmonar. diferencial del pulmón blanco bilateral junto con el edema
agudo de pulmón).
Finalmente, si la lesión del epitelio alveolar es grave, o hay un - Gasometría arterial.
reparo epitelial insuficiente, se produce fibrosis, pudiéndose En la gasometría inicialmente sólo se observan hipocapnia y
demostrar histológicamente incluso a partir del 5-7 día. La alte- aumento del gradiente alveoloarterial de oxígeno, pero en
ración funcional más frecuente en los pacientes que sobreviven fases más avanzadas hay hipoxemia grave.
a un SDRA es el descenso de la DLCO. - Estudio hemodinámico.
Es característico el aumento de la presión en la arteria pulmo-
nar, pero con la presión capilar enclavada normal (aspecto que
Diagnóstico lo diferencia del edema pulmonar cardiogénico).
El diagnóstico del SDRA es clínico pero, en casos de duda,
puede utilizarse un catéter de Swan-Ganz para medir la pre- Pronóstico
sión capilar pulmonar enclavada, que será normal o estará
Si no se corrige a tiempo puede aparecer fibrosis a partir de la
disminuida.
segunda o tercera semana de su instauración.
De acuerdo con los criterios de Berlín, el diagnóstico definitivo
requiere la presencia de:

65
Manual ENARM · Neumología y Cirugía Torácica

Tratamiento
El de la enfermedad de base, junto con soporte respiratorio:
oxigenoterapia y, si es necesario, ventilación mecánica. La
ventilación mecánica debe utilizar presión teleespiratoria posi-
tiva (PEEP) para evitar el colapso de los alvéolos, y volúmenes
corrientes bajos para evitar el volutrauma (lesión de los alvéolos
por distensión excesiva durante la inspiración).

Figura 1. Características radiológicas del SDRA.

66
Tema 9
Enfermedades intersticiales 1:
neumonitis intersticiales idiopáticas
Clasificación de las enfermedades intersticiales
Linfangioleiomiomatosis
Neumonitis intersticiales idiopáticas (se estudia en el PATRÓN OBSTRUCTIVO
Histiocitosis X
tema 9. Enfermedades intersticiales 1: neumonitis inters-
ticiales idiopáticas) Sarcoidosis
- Fibrosis pulmonar idiopática. PATRÓN MIXTO Silicosis
- Otras neumonitis intersticiales idiopáticas. (PUEDE SER OBSTRUCTIVO Neumonitis eosinofílica crónica
O RESTRICTIVO) Neumonitis por
Neumonitis de causa conocida (se estudia en el tema hipersensibilidad crónica
10. Enfermedades intersticiales 2: neumonitis de causa
conocida) Tabla 1. Enfermedades intersticiales con patrón espirométrico obstructivo.
- Por inhalación de polvos inorgánicos: neumoconiosis.
- Por inhalación de polvos orgánicos.
- Asociadas a enfermedades autoinmunes sistémicas.
- Por fármacos.
PATRÓN RETICULAR
Neumonitis asociadas a otros procesos no bien conocidos Los más frecuentes
PATRÓN
(se estudia en el tema 11. Enfermedades intersticiales 3: RETICULONODULILLAR
neumonitis asociadas a procesos no bien conocidos)
- Sarcoidosis. PATRÓN NODULILLAR Silicosis, antracosis…
- Linfangioleiomiomatosis e histiocitosis X.
- Eosinofilias pulmonares. PATRÓN Típico de la
- Proteinosis alveolar. NODULOQUÍSTICO histiocitosis X
Derrame
Típico de la pleural y
Aspectos comunes de las enfermedades intersticiales neumotórax
PATRÓN QUÍSTICO linfangioleio-
Las enfermedades pulmonares intersticiales son un grupo miomatosis
heterogéneo de procesos que afectan a estructuras alveoloin-
tersticiales y que tienen manifestaciones clínicas, radiológicas y
Tabla 2. Patrones radiológicos comunes de las enfermedades intersticiales.
fisiopatológicas similares.
La clínica suele consistir en disnea, tos seca y crepitantes
teleinspiratorios.
Fisiopatológicamente, el daño pulmonar produce inicialmen-
te una alteración de la difusión (que se manifiesta como Sarcoidosis
hipoxemia con el esfuerzo), y en estadios más avanzados ADENOPATÍAS
Silicosis
una alteración del patrón ventilación/perfusión (↑ V/Q) PERIHILIARES
Beriliosis
(que se manifiesta como hipoxemia en reposo). Así, la gaso-
metría en reposo es útil en su seguimiento ya que si estamos INFILTRADOS
en estadios avanzados aparecerá hipoxemia. PERIHILIARES Proteinosis alveolar
El patrón espirométrico típico es el restrictivo. (EN ALAS DE MARIPOSA)
Radiológicamente se manifiestan por presentar patrón inters-
ticial, que engloba la presencia de líneas en retícula, nodulillos INFILTRADOS
y quistes, existiendo numerosos patrones según la presencia PERIFÉRICOS Neumonitis eosinofílica crónica
(NEGATIVO DE
o no de estos elementos (ver tabla 2 y tabla 3). El patrón ALAS DE MARIPOSA)
intersticial suele tener un predominio en las bases pulmo-
nares, con algunas excepciones (ver tabla 4 en la página
Neumonía Organizativa
siguiente).
Criptogénica (NOC)
La radiografía de tórax es la prueba de imagen inicial, pero INFILTRADOS Churg-Strauss
tanto el diagnóstico como el seguimiento se basan en la TC PARCHEADOS Neumonías eosinofílicas:
torácica de alta resolución. MIGRATORIOS - Neumonitis eosinofílica aguda
Para llegar al diagnóstico de cada enfermedad intersticial - Síndrome de Löeffler
concreta, en ocasiones es necesaria la realización de lavado - ABPA
broncoalveolar y/o de biopsia pulmonar (ver tabla 5 en la
página siguiente). Tabla 3. Patrones radiológicos especiales de las enfermedades intersticiales.

67
Manual ENARM · Neumología y Cirugía Torácica

taquipnea, cianosis, acropaquias (dato frecuente pero tardío)


REGLA MNEMOTÉCNICA
y, en casos avanzados, cor pulmonale y fallo cardiaco derecho.
FITNESS
Fibrosis quística
Diagnóstico
Istiocitosis X
Tuberculosis Radiografía de tórax
Neumonitis por hipersensibilidad crónica Tradicionalmente se creía que en la FPI existía una progresión
Espondilitis anquilosante radiológica, presentando un patrón de vidrio deslustrado o
Sarcoidosis esmerilado en las fases iniciales de la enfermedad (alveolitis
Silicosis aguda), que traduce un aumento difuso del tejido intersticial.
Después se formarían infiltrados nodulares y reticulonodulares
y, finalmente, en la fase de fibrosis, aparece un patrón reticu-
Tabla 4. Enfermedades intersticiales con predominio radiológico en campos
superiores. lar difuso, en panal de abeja o queso suizo, que constituye el
estadio final y está caracterizado por la presencia de espacios
quísticos de paredes gruesas. Esta progresión actualmente
Patrón típico de las está en duda, pues se considera que la FPI es una enferme-
enfermedades intersticiales dad directamente fibrosante (imágenes reticulonodulares y
- ↑ CD4/CD8: sarcoidosis panal de abeja). Estas alteraciones radiológicas predominan en
PREDOMINIO - ↓ CD4/CD8: neumonitis por lóbulos inferiores y son de predominio periférico, subpleu-
LINFOCÍTICO hipersensibilidad crónica, ral. No son exclusivas de esta enfermedad, sino que pueden
Neumonía Organizativa aparecer en otras enfermedades pulmonares intersticiales,
Criptogénica (NOC) donde sí que se puede definir mejor la progresión radiológica
antes mencionada.
Fibrosis pulmonar idiopática
PREDOMINIO Neumonitis por
PMN hipersensibilidad aguda TC de Alta Resolución (TCAR)
Detecta alteraciones más precoces que la radiografía y permite
Eosinofilias pulmonares diferenciar las zonas de alveolitis de las más evolucionadas,
PREDOMINIO Neumonitis por fármacos siendo la técnica más empleada para valorar la actividad de
EOSINÓFILO (excepto amiodarona) la enfermedad. Para esto último, también pueden realizarse
estudios isotópicos con galio-67.
PREDOMINIO Proteinosis alveolar (PAS+)
MACRÓFAGOS Neumonitis por amiodarona Pruebas funcionales respiratorias
C. LANGERHANS >5% Histiocitosis X En la espirometría presenta un patrón restrictivo.

Tabla 5. Lavado broncoalveolar en las enfermedades intersticiales. Lavado broncoalveolar


Muestra aumento de PMN y de eosinófilos; los macrófagos
alveolares están aumentados y activados. El aumento de linfo-
9.1. Fibrosis pulmonar idiopática citos es raro, pero si aparece implica mejor pronóstico.

También llamada neumonitis intersticial usual o alveolitis fibro-


sante criptogenética.

Patogenia
Consiste en una respuesta inflamatoria de etiología desconoci-
da que desemboca en una fibrosis del tejido pulmonar. Aunque
se desconoce la causa, se sabe que se produce fijación de
inmunocomplejos a los macrófagos alveolares. Los macrófagos
así activados producen citocinas (LT B4 e IL-8), que atraen poli-
morfonucleares (PMN) y eosinófilos (cuyos productos provocan
lesiones locales y aumentan la permeabilidad de los neumocitos
tipo I), y sustancias estimulantes de los fibroblastos del inters-
ticio y paredes alveolares. Finalmente, la fibrosis afecta a los
vasos, oblitera los alveolos y retrae las vías aéreas terminales,
formándose áreas quísticas recubiertas de epitelio bronquial.

Clínica
Suele tratarse de pacientes alrededor de los 50 años. Además
de disnea de esfuerzo y tos seca, aparecen síntomas genera-
les (astenia, anorexia, pérdida de peso, artralgias).
Figura 1. Fibrosis pulmonar idiopática en fase avanzada. Patrón en panal de
La exploración puede evidenciar crepitantes secos (“en abeja, constituido por bronquiectasias arrosariadas de predominio periférico
velcro”) bibasales al final de la inspiración (teleinspiratorios), (subpleural) en campos inferiores.

68
Tema 9 · Enfermedades intersticiales 1: neumonitis intersticiales idiopáticas

Toma de biopsias asociado a distintos cuadros clínicos y etiológicos). El patrón


El diagnóstico definitivo de la FPI es histopatológico, aunque radiológico detectado en la TC torácica de alta resolución
no siempre es necesaria la realización de una biopsia pulmo- (TCAR) resulta fundamental para caracterizar las distintas enti-
nar. En el caso de clínica compatible, exploraciones funcionales dades, por lo que se ha propuesto la siguiente clasificación,
compatibles y una TC torácica de alta resolución (TCAR) con basada en los hallazgos radiológicos e histológicos:
alta probabilidad de FPI (ver tabla 6) podemos llegar al diag-
nóstico. En el caso de no cumplir estos criterios, será necesaria
la realización de biopsia pulmonar. NEUMONÍA - Patrón más frecuente, se suele correspon-
INTERSTICIAL der con FPI.
USUAL
CRITERIOS DE ALTA SOSPECHA DE FPI EN TCAR
(SE DEBEN CUMPLIR TODOS) - Varones jóvenes fumadores.
TCAR: vidrio deslustrado
NEUMONÍA - LBA: macrófagos hiperpigmentados.
Afectación subpleural y basal
INTERSTICIAL - Diagnóstico: biopsia.
Patrón reticular
DESCAMATIVA - Tratamiento: no fumar + corticoides.
Panal de abeja
Ausencia de: afectación peribroncovascular, vidrio esmerilado, - Supervivencia 70% a 5 años.
micronódulos, quistes, mosaico o consolidación
- 40% secundarias a colagenosis.
Tabla 6. Criterios de alta sospecha de FPI en la TCAR. NEUMONÍA - TCAR: vidrio deslustrado, fibrosis,
INTERSTICIAL escaso panal (mixto).
NO ESPECÍFICA - Tratamiento: corticoides.
Tratamiento y pronóstico
Se trata de una enfermedad pulmonar fibrosante crónica de
características heterogéneas y de curso variable con mal pro- NEUMONÍA - Clínica y Rx de SDRA, pero sin causa
nóstico sin tratamiento (supervivencia del 50% a los 5 años). INTERSTICIAL aparente.
Existen múltiples comorbilidades que pueden influir en el pro- AGUDA - Mal pronóstico: mortalidad 50% a 2 meses.
nóstico (reflujo gastroesofágico, infecciones, TEP, enfermedad (= PULMÓN - Posible curación completa o evolución a
HAMMAN-RICHS) neumonitis intersticial crónica.
coronaria, SAHS, depresión, enfisema, hipertensión pulmonar,
cáncer de pulmón). Hasta hace pocos años se empleaban com-
binaciones de corticoides, azatioprina y acetilcisteína. Dichos - Asociada a imunodeficiencias o procesos
regímenes se han abandonado por su toxicidad y ausencia de linfoproliferativos.
NEUMONÍA
beneficios clínicos. Hoy en día, el manejo terapéutico se basa - Clínica insidiosa disnea + mialgias +
INTERSTICIAL
en: cuadro constitucional
LINFOIDE
- BAS y biopsia: infiltrado linfocítico.
- Tratamiento de soporte:
- Tratamiento: corticoides, buena respuesta.
Oxígeno suplementario, vacunación de gripe estacional y
neumococo, rehabilitación pulmonar e identificación y tra-
tamiento activo de comorbilidades. Podría ser beneficioso el - Clínica subaguda constitucional.
tratamiento antirreflujo esofágico. NEUMONÍA - Rx: infiltrados parcheados migratorios.
- Tratamiento antifibrótico directo: ORGANIZATIVA - Espirometría restrictiva.
Con pirfenidona o con nintedanib, nuevos agentes orales CRIPTOGENÉTICA - Biopsia: fibrosis intralveolar.
que han demostrado retrasar el declive de la CVF en la FPI (ANTIGUA BONO) - Tratamiento: corticoides, buena
leve y moderada. En México está aprobado el uso de pirfeni- respuesta, frecuentes recaídas.
dona en pacientes con CVF 50-90% y DLCO entre 30-90%.
- Considerar la inclusión en ensayos clínicos en centros de Tabla 7. Diagnóstico diferencial de las otras neumonitis intersticiales idiopáticas.
referencia.
- Considerar la inclusión en lista de trasplante pulmonar:
En pacientes con enfermedad progresiva sin otras contra- - NIU (neumonitis intersticial usual).
indicaciones para el trasplante. La tendencia es incluir pre- Es el patrón más frecuente. Su hallazgo no implica necesaria-
cozmente en la lista a los pacientes candidatos, pues parece mente un diagnóstico de fibrosis pulmonar idiopática, pues en
mejor estrategia que esperar a que se deteriore su situación algunas ocasiones es secundaria a asbestosis, colagenopatías,
clínica. sarcoidosis, alveolitis extrínseca alérgica, etc. Sin embargo, en
la gran mayoría de las veces no se encuentran causas secun-
darias, correspondiéndose entonces con la fibrosis pulmonar
9.2. Otras neumonitis intersticiales idiopática.
- NID (neumonitis intersticial descamativa).
La clínica es similar a la de la FPI, pero aparece en varones
La patología intersticial pulmonar supone un extenso grupo
jóvenes fumadores; tiene mejor pronóstico que ésta (espe-
de patologías no bien conocidas, de entre las que destacan las
cialmente si se abandona el consumo de tabaco). Responde
neumonitis idiopáticas. Su estudio diagnóstico, clasificación y
a corticoides. Es característico el hallazgo de macrófagos hi-
tratamiento son complejos. Además de la gran heterogenei-
perpigmentados en el lavado bronquioalveolar. En la TCAR
dad de manifestaciones clínico-radiológicas, se suma una gran
predominan las opacidades en vidrio deslustrado, que pue-
variedad de correlaciones etiológicas (es decir, una misma
den acompañarse de zonas de patrón reticular, pero la pana-
causa -p. ej., una enfermedad reumatológica- puede producir
lización es muy infrecuente, y cuando aparece, escasa.
distintos patrones radiológicos de afectación pulmonar inters-
- NINE (neumonitis intersticial no específica).
ticial; por otro lado, un mismo patrón radiológico puede verse
Tiene características radiológicas mixtas entre la NIU y la NID,

69
Manual ENARM · Neumología y Cirugía Torácica

sin corresponderse claramente con ninguna de ellas (tiene - NIL (neumonitis intersticial linfoide).
opacidades en vidrio deslustrado, pequeñas áreas de pana- Cuadro muy raro, caracterizado por la proliferación linfoide en
lización, bronquiolectasias…). Hasta en un 40% de los casos el territorio intersticial (se asocia a cuadros linfoproliferativos
es secundaria a una enfermedad sistémica subyacente, princi- y situaciones de inmunosupresión). En el lavado broncoalveo-
palmente colagenosis. El tratamiento consiste en corticoides, lar se aprecia una marcada linfocitosis, precisándose para el
añadiendo inmunosupresores cuando hay poca respuesta. diagnóstico una biopsia pulmonar quirúrgica, que muestra
- NIA (neumonitis intersticial aguda). infiltración del intersticio pulmonar y espacios alveolares por
También denominada síndrome del distrés respiratorio agudo linfocitos, células plasmáticas e histiocitos. El tratamiento se
idiopático o pulmón de Hamman Rich. La lesión patológica co- realiza con corticoides, con respuesta en el 70% de los casos.
rresponde a daño alveolar difuso. Se suele presentar de forma Cursa con áreas de vidrio deslustrado, nódulos centrolobulilla-
aguda, con disnea rápidamente progresiva y mal pronóstico, res, y engrosamiento perilinfático, peribroncovascular y septal
con una supervivencia del 50% a los 2 meses del diagnóstico. en la TCAR.
El tratamiento se basa en corticoides, con respuesta variable. - NO (neumonía organizada).
(Se estudia en el tema 15. Bronquiolitis)

Pregunta ENARM
Hombre de 58 años de edad, jubilado en condición
de abandono. Refiere disnea de esfuerzo de al menos
un año de evolución, la cual ha sido progresiva y
que se ha hecho acompañar en los últimos 6 meses
de tos seca. Combe negativo. A la exploración física
se auscultan estertores inspiratorios tardíos difusos,
se observan dedos en palillo de tambor. Se solicita
radiografía de tórax encontrando un patrón reticular y
panal de abeja subpleural bilateral.

16. Con base en los antecedentes y los hallazgos


radiológicos se integra el diagnóstico de:

A. Fibrosis pulmonar idiopática.


B. Neumonía intersticial aguda.
C. Esclerosis generalizada progresiva.
D. Proteinosis alveolar pulmonar.

17. La lesión pulmonar es secundaria a:

A. Acumulación de material lipoproteináceo amorfo


alveolar.
B. Reacción autoinmunitaria mediada por igG.
C. Inflamación crónica con aparición de cicatrices de
fibrosis.
D. Sustitución progresiva de la estructura hística nor-
mal por tejido conjuntivo denso.

· Encuentra las respuestas y comentarios de las preguntas ENARM al final del manual ·

70
Tema 10
Enfermedades intersticiales 2:
neumonitis de causa conocida

10.1. Neumonitis intersticiales por inhalación de - Silicosis aguda.


Se caracteriza por un patrón miliar. Se asocia a exposiciones a
polvos inorgánicos: neumoconiosis
polvos de sílice muy intensas (p. ej., chorros de arena) en cor-
tos espacios de tiempo (semanas a meses). Puede ser mortal
Comprende un conjunto de enfermedades pulmonares provo- en menos de dos años. Radiográficamente presentan un pa-
cadas por la inhalación y penetración de polvo inorgánico, que trón alveolar difuso fundamentalmente en lóbulos inferiores
ocasiona una lesión histológica. con o sin broncograma aéreo.

Silicosis Complicaciones
Concepto La exposición al sílice potencia los efectos nocivos del tabaco,
incrementando en esta población el riesgo de EPOC. Asimismo,
Neumoconiosis producida por la exposición a SiO2 (sílice o
la silicosis predispone a la tuberculosis y a la infección por
cuarzo cristalino), por lo que tienen riesgo los trabajadores de
micobacterias atípicas. Por último, la prevalencia de neumo-
minas de carbón, fundiciones, cerámica y canteras de granito.
tórax también está aumentada entre los pacientes silicóticos.

Formas clínicas
Recuerda que...
- Silicosis simple, crónica o clásica.
Las lesiones aparecen al cabo de 15-20 años de exposición. Se ha identificado la silicosis como un factor de riesgo para tuber-
Consiste en fibrosis pulmonar que, aunque diseminada, está culosis, aún después de finalizada la exposición. La GPC recomien-
focalizada en nódulos silicóticos menores de 1 cm, sobre todo da en estos pacientes la monitorización periódica con PPD.
en lóbulos superiores y adenopatías hiliares, a veces con cal-
cificación en cáscara de huevo.
Puede ser asintomática o manifestarse con tos, expectoración
Diagnóstico
crónica y disnea de esfuerzo, debido a la patología asociada,
generalmente bronquitis crónica, producida por el frecuente Se basa en la presencia de antecedentes de exposición y de
hábito tabáquico en esta población. radiografías compatibles. Las pruebas funcionales permiten
valorar el grado de incapacidad. No suele ser necesario realizar
biopsia pulmonar para llegar al diagnóstico.

Tratamiento
No existe. Se debe insistir en la prevención (disminuir los niveles
de polvo y SiO2 en el ambiente laboral) y diagnóstico precoz
(chequeos periódicos).
Dada la asociación entre silicosis y tuberculosis, está indica-
do iniciar tratamiento específico ante la sospecha clínica de
tuberculosis, y quimioprofilaxis secundaria en caso de Mantoux
positivo.

Antracosis
Concepto
Es la neumoconiosis de los mineros del carbón.

Figura 1. Patrón radiológico de la silicosis clásica. Formas clínicas


- Antracosis simple.
Inicialmente hay un patrón reticular, al que se añaden nódulos
- Silicosis acelerada. de 1-5 mm de diámetro. Cursa como bronquitis crónica (tam-
Exposición más corta e intensa que la simple, pero clínica y bién contribuye la asociación con tabaquismo).
radiográficamente es muy similar a la anterior. Es frecuente - Antracosis complicada (fibrosis masiva progresiva).
su asociación a esclerodermia. Nódulos mayores de 1 cm confluentes, sobre todo en campos
- Silicosis complicada o fibrosis masiva progresiva. superiores.
Ocurre incluso cuando la exposición al sílice ha cesado, los
nódulos confluyen y forman conglomerados mayores de 1 cm
En la antracosis, al igual que en cualquier otra enfermedad
en lóbulos superiores.
pulmanor intersticial, puede asociarse la presencia de nódulos
Clínicamente hay marcadas alteraciones funcionales obstruc-
pulmonares reumatoideos, formando lo que se denomina sín-
tivas y restrictivas y, en casos avanzados, insuficiencia respira-
drome de Caplan.
toria. Pueden presentar infecciones bacterianas recurrentes.

71
Manual ENARM · Neumología y Cirugía Torácica

aparece tras 25-30 años de exposición. No hay tratamiento


eficaz. La supervivencia mediana es de 7 meses tras el diagnós-
tico. El tratamiento con cirugía + QT (en enfermedad localiza-
da) o QT combinada (cisplatino + pemetrexed, en enfermedad
avanzada) eleva la supervivencia mediana a 12-18 meses.

Figura 2. Antracosis.

Beriliosis
La exposición al berilio (cerámica, electrónica de alta tecnología
y tubos fluorescentes antiguos) puede producir una neumonitis
intersticial crónica que se manifiesta radiológicamente con un
patrón reticulonodular a veces con adenopatías, similar al de
la sarcoidosis. En la anatomía patológica se encuentran granu-
lomas idénticos a los de la sarcoidosis, pero centrados en un
cristal de berilio.

Figura 3. Imagen radiográfica de placas pleurales en un paciente con asbestosis.


Asbestosis
Concepto
La exposición laboral a asbesto o amianto ocurre en la cons- 10.2. Neumonitis intersticiales por inhalación de
trucción (aislantes térmicos y eléctricos, tuberías, calderas, polvos orgánicos
uralita) y también se usa este compuesto en la elaboración de
los trajes de neopreno, frenos.
Neumonitis por hipersensibilidad o alveolitis alérgica
extrínseca
Clínica
Concepto
La disnea es el síntoma más frecuente y que más precozmente
Es una inflamación de las vías respiratorias distales mediada
presentan las personas expuestas.
inmunológicamente, secundaria a la inhalación repetida de
Las manifestaciones radiológicas más frecuentes de la exposi- sustancias de origen orgánico, como antígenos derivados de
ción a asbesto son las lesiones pleurales benignas. Dentro de bacterias, hongos, animales o plantas, o también por la inha-
ellas, la más frecuente es la presencia de placas pleurales lación de productos químicos. En la mayoría de los casos, los
en la radiografía de tórax, sobre todo en campos inferiores y antígenos son inhalados en el curso de una exposición laboral:
diafragma. Sin embargo, las placas pleurales sólo indican expo- granjeros, cosechadores de caña de azúcar (begazosis), criado-
sición a asbesto. Además, aunque son típicas de la asbestosis, res de aves, trabajadores de productos de la madera, etc.
no son exclusivas de esta neumoconiosis. Funcionalmente, se
Los dos tipos más frecuentes son el pulmón del granjero (por
caracteriza por un trastorno restrictivo con alteración de la
actinomicetos termofílicos del heno y otros vegetales) y el pul-
transferencia gaseosa e hipoxemia. La disminución de la DLCO
món del cuidador de aves (por proteínas de los excrementos,
es un dato de enfermedad grave. Otras lesiones pleurales
plumas y escamas de determinadas aves).
benignas causadas por la exposición al asbesto son el derrame
pleural benigno, las atelectasias redondas y la fibrosis pleural El tabaco parece ejercer un efecto protector en esta enfermedad.
difusa.
La exposición a asbesto puede dar lugar también a la aparición Clínica
de cáncer de pulmón. El cáncer de pulmón es el tumor aso- Podemos distinguir varias formas en función del tiempo de
ciado más frecuentemente al asbesto (más que el mesotelio- exposición:
ma). Aparece al cabo de 15-20 años de la exposición. Los tipos
histológicos no difieren de los encontrados en la población - Forma aguda-subaguda.
general (el epidermoide es el más frecuente) y está demostrado Clínicamente cursa con tos seca, disnea, fiebre y malestar ge-
el efecto sinérgico entre el asbesto y el tabaco. neral después de la exposición (4-8 horas en la aguda y varias
semanas en la subaguda).
El mesotelioma difuso maligno es el cáncer que más - Forma crónica.
característicamente se asocia al asbesto. Puede ser pleural o Es una enfermedad pulmonar intersticial de instauración gra-
peritoneal. Produce derrame pleural que ocupa más del 50% dual, asociada a la exposición prolongada a bajas dosis de
del hemitórax afecto, lo que dificulta el diagnóstico porque antígeno. La clínica es similar a la de un bronquítico crónico,
enmascara el tumor subyacente. Es independiente del tabaco, pero en un paciente no fumador.

72
Tema 10 · Enfermedades intersticiales 2: neumonitis de causa conocida

Diagnóstico Los corticoides parecen útiles en formas agudas y subagudas,


Se basa fundamentalmente en la historia clínica, incidiendo mientras que en las formas crónicas probablemente no modi-
sobre la exposición a ciertos polvos orgánicos. fiquen la evolución.
Hay neutrofilia y linfopenia tras la exposición aguda pero no
hay eosinofilia ni elevación de IgE.
Recuerda que...
La radiografía de tórax, en la fase aguda-subaguda, es
normal o aparecen infiltrados alveolares mal definidos, foca- En nuestro medio la exposición a proteínas de plumaje de ave es
les o difusos, de predominio en lóbulos inferiores. En la fase una causa común de neumonitis intersticial.
crónica se observa un infiltrado reticulonodular difuso de pre-
dominio en lóbulos superiores que puede producir un pulmón
en panal. Bisinosis
El estudio de la función pulmonar indica la existencia de un Provocada por la inhalación de polvo de algodón, que ocurre
patrón restrictivo con disminución de la DLCO, en la forma sobre todo durante el proceso de cardado.
aguda. En la forma crónica también predomina un patrón Se manifiesta por un cuadro clínico de disnea y opresión torá-
restrictivo, pero frecuentemente asocia un defecto obstructivo cica junto con reducción del FEV1 el primer día de la semana
debido a bronquiolitis asociada. laboral (opresión torácica de los lunes), aunque luego se
Los estudios serológicos pueden mostrar precipitinas séricas puede hacer persistente. Se trata evitando el contacto con el
frente al antígeno responsable. polvo de algodón y con broncodilatadores y antihistamínicos.
El lavado broncoalveolar demuestra en las fases aguda y
crónica un incremento de linfocitos T, con predominio de los
CD8 (cociente CD4/CD8 disminuido) y macrófagos espumosos. - Cuarzo, canteras de piedra, fundiciones, cerámica
En la fase aguda, puede aparecer neutrofilia acompañada de SILICOSIS - Adenopatías calcificadas en cáscara de huevo
una elevación de linfocitos T CD4. - Aumento de riesgo de TBC
La biopsia pulmonar (transbronquial o abierta) está indicada - “Neumoconiosis por carbón”
si el resto de los datos son insuficientes para el diagnóstico. ANTRACOSIS - Síndrome de Caplan

- Cerámicas, tubos fluorescentes


Anatomía patológica BERILIOSIS - Radiología e histología similares a sarcoidosis
Anatomopatológicamente, se pueden observar macrófagos de
citoplasma espumoso, infiltrados intersticiales de linfocitos y - Amianto, frenos, aislantes
células plasmáticas en las formas agudas; granulomas aislados - Placas pleurales
ASBESTOSIS - Cáncer de pulmón ↑ ↑
no necrotizantes en las subagudas y crónicas, y fibrosis pulmo-
nar en las crónicas. - Mesotelioma maligno ↑

- Granjero, criador de aves


NEUMONITIS - No hay eosinofilia ni aumento de IgE
AGUDA CRÓNICA POR HIPER- - Característica la neutrofilia con linfopenia
SENSIBILIDAD - Aumento de CD8 (↓ CD4/CD8)
Exposición breve a Exposición prolongada a
EXPOSICIÓN grandes dosis de Ag dosis de Ag menores - Algodón
BISINOSIS - “Opresión torácica del lunes”
Disnea, tos seca, Clínica de EPOC
CLÍNICA fiebre, MEG en un no fumador
Tabla 2. Rasgos característicos de las enfermedades por inhalación de polvos.
RADIO- Infiltrados alveolares en
Fibrosis en LLSS
GRAFÍA LLII

Patrón restrictivo pero 10.3. Neumonitis intersticiales asociadas a


FUNCIÓN Patrón restrictivo con
también asocia enfermedades autoinmunes sistémicas
PULMONAR disminución de DLCO
patrón obstructivo
La presencia síntomas sistémicos en un paciente en estudio de
LBA Neutrófilos Aumento de CD8
un patrón intersticial en la radiología del tórax debe hacernos
Inflamación alveolar e intersticial pensar en una enfermedad intersticial asociada a enfermeda-
BIOPSIA Granulomas des del colágeno.
Fibrosis intensa - Esclerodermia.
Es la conectivopatía que con mayor frecuencia produce enfer-
TRATA- Evitar la exposición al agente y corticoides medad intersticial. Predomina en lóbulos inferiores.
MIENTO - Lupus eritematoso sistémico.
La afectación pulmonar más frecuente es la pleuritis con o sin
Tabla 1. Características principales de la neumonitis por hipersensibilidad. derrame. A su vez, la conectivopatía que más frecuentemente
produce derrame pleural es el LES.
- Artritis reumatoide.
Tratamiento Las manifestaciones pleuropulmonares en la AR son más fre-
Debe evitarse el contacto con el antígeno responsable. cuentes en varones. La afectación pulmonar más frecuente es

73
Manual ENARM · Neumología y Cirugía Torácica

también la pleural, pero también puede haber afectación pul- - Neumonitis organizada.
monar intersticial por los fármacos usados como tratamiento Menos frecuente, 25-30%. Clínica de tos, disnea, expectora-
de esta enfermedad. La presencia de nódulos pulmonares en ción y fiebre aguda, o cuadro constitucional subagudo, con
pacientes afectos de artritis reumatoide y neumoconiosis se infiltrados alveolares únicos o parcheados. Se confunde habi-
denomina síndrome de Caplan. tualmente con una neumonía.
- Síndrome de Sjögren. - Neumonitis fibrosante.
La afectación pulmonar es más frecuente en el Sjögren prima- Primaria o como evolución de neumonitis intersticial, 5-7%.
rio. Se caracteriza por una infiltración linfocítica intersticial, que Clínica de fibrosis, con escasa tos y mucha disnea e hipoxemia.
puede llegar a comportarse como un linfoma de bajo grado. Radiológicamente infiltrados reticulonodulillares y bronquiec-
- Dermatomiositis. tasias por tracción.
La enfermedad intersticial aparece fundamentalmente en pa- - Neumonitis rápidamente progresiva con SDRA.
cientes con anticuerpos anti-JO1 positivos. Rara, aguda, más frecuente post-neumonectomía o tras infu-
- Espondilitis anquilosante. sión de contraste yodado.
El paciente suele estar asintomático y, en fases avanzadas, - Nódulos pulmonares.
puede aparecer fibrosis bilateral de lóbulos superiores, que Habitualmente periféricos o subpleurales en campos superiores.
adopta un patrón quístico. - Hemorragia pulmonar.
Lo más infrecuente.

10.4. Neumonitis intersticiales por fármacos


Bleomicina
El antineoplásico con mayor toxicidad pulmonar (20%), dosis-
Los fármacos que más frecuentemente pueden ocasionar dependiente y edad-dependiente. Se precede de descenso de
enfermedad pulmonar intersticial son la amiodarona, el meto- la DLCO. Revierte o mejora con suspensión y corticoides.
trexate, la nitrofurantoína, el busulfán y la bleomicina.

Metotrexate
Amiodarona La clínica es idéntica a la producida por la amiodarona pero
La toxicidad pulmonar intersticial por amiodarona es más además, es característica la presencia de granulomas y eosino-
frecuente a dosis altas y puede aparecer tras años de trata- filia. No es dosisdependiente. El proceso es casi siempre reversi-
miento o tras pocas semanas. Dosis >400 mg/día provocan ble, con o sin corticoides. No hay disminución de la DLCO hasta
neumotoxicidad en el 5-15% de pacientes, mientras que dosis que no aparece la clínica.
200 mg/día (la más habitual) en torno al 1.5%. Afecta más a
lóbulos superiores, especialmente a pulmón derecho. Es muy
típica la presencia, tanto en LBA como en AP, de macrófagos Nitrofurantoína
espumosos y neumocitos con inclusiones lamelares. Mejora Característicamente puede producir neumonitis eosinófila
con la suspensión del fármaco y añadiendo corticoides. aguda, como consecuencia de la toxicidad aguda, que se
Puede dar distintos cuadros anatomoclínicos: manifiesta con fiebre, tos y disnea. La toxicidad crónica por
nitrofurantoína produce una neumonitis intersticial por fibrosis
- Neumonitis intersticial.
que se manifiesta con tos seca y disnea.
Lo más frecuente, 2/3 de los casos. Clínica típica de EPID,
con tos seca, disnea progresiva, y en ocasiones febrícula o
astenia. Radiológicamente infiltrados intersticiales periféricos
de predominio superior, sobre todo lóbulo superior derecho.

Pregunta ENARM
18. Hombre de 65 años de edad, el cual trabajó
durante al menos 30 años en una marmolería
cortando piedra. Acude a consulta refiriendo tos
y expectoración crónica. La radiografía de tórax
muestra fibrosis pulmonar, nódulos de algunos
mm en lóbulos superiores y adenopatías parahi-
liares bilaterales. El diagnóstico pulmonar más
probable es:

A. Silicosis.
B. Enfisema pulmonar.
C. Tuberculosis miliar.
D. Criptococosis.

· Encuentra las respuestas y comentarios de las preguntas ENARM al final del manual ·

74
Tema 11
Enfermedades intersticiales 3:
neumonitis asociadas a procesos no bien conocidos

11.1. Sarcoidosis

Epidemiología
Suele afectar a individuos entre los 20 y los 40 años, con pre-
dominio en el sexo femenino y raza negra. Parece ser que el
tabaco ejerce un papel protector.

Anatomía patológica
El granuloma sarcoideo, aunque es típico, no es patognomóni-
co. Se trata de un granuloma no caseificante formado por un
acúmulo de macrófagos, células epitelioides y células gigantes
multinucleadas tipo Langhans rodeadas por linfocitos T CD4+
y, en menor medida, linfocitos B. El conjunto de linfocitos T,
monocitos y granulomas representa la enfermedad activa.
Las células gigantes pueden contener inclusiones inespecí-
ficas como corpúsculos asteroides, cuerpos concoides o de
Schaumann y cuerpos de Hamasaki-Weisenberg. En el centro
hay, a veces, necrosis fibrinoide (no caseosa).

Figura 1. Adenopatías hiliares bilaterales en la sarcoidosis.


Clínica
Manifestaciones torácicas
Aparecen en más del 90% de los casos y constituyen la mayor
causa de morbimortalidad. Se caracterizan por una afectación
intersticial que produce como síntomas típicos disnea de esfuer-
zo y tos seca (aunque los pacientes pueden estar asintomáticos).
Las manifestaciones torácicas se suelen clasificar en función
de la afectación ganglionar y/o pulmonar según su forma de
presentación en la radiografía de tórax del siguiente modo:

ESTADIOS

I Adenopatías hiliares bilaterales sin afección parenquimatosa

Adenopatías hiliares bilaterales con afección parenquimato-


II sa reticulonodular

III Afección parenquimatosa sin adenopatías hiliares


Figura 2. Afectación parenquimatosa en la sarcoidosis.
IV Fibrosis pulmonar
Manifestaciones extratorácicas de la sarcoidosis
Tabla 1. Clasificación radiológica de la sarcoidosis.
- Cutáneas (35%).
La más frecuente es el eritema nodoso, sobre todo en las
La afectación ganglionar se localiza, por orden de frecuen- formas agudas, pero la lesión más característica es el lupus
cia, en ganglios hiliares bilaterales (80-90% de los casos), para- pernio.
traqueales derechos, ventana aortopulmonar y subcarinales. - Oculares (25%).
Suele ser asintomática (a no ser que compriman estructuras Lo más frecuente es la uveítis anterior y también, aunque
vecinas). También es común la afectación de ganglios linfáticos menos frecuente, puede presentar coroiditis.
supraclaviculares, axilares, epitrocleares e inguinales. - Hígado.
Las adenopatías son indoloras y móviles. La afectación del parén- Pese a que la expresión clínica es rara, la afectación histológica
quima pulmonar se suele tolerar bastante bien hasta que se es frecuente, apareciendo en un 60-90% de los pacientes,
alcanzan formas muy evolucionadas. Además pueden aparecer lo que hace que la biopsia hepática sea muy rentable para el
granulomas en el tracto respiratorio superior, que pueden diagnóstico. El síntoma más común de la sarcoidosis hepática
producir rinorrea u obstrucción nasal junto a pólipos nasales, y en es la fiebre prolongada.
la tráquea y bronquios, provocando estenosis. También pueden - Médula ósea (20-40%).
aparecer granulomas laríngeos, que se asocian con lupus pernio. Leve anemia, neutropenia y/o trombopenia.

75
Manual ENARM · Neumología y Cirugía Torácica

- Bazo (5-10%). - Aparato reproductor.


Esplenomegalia e hiperesplenismo. En las mujeres, puede haber afectación del útero pero no
- Neurológicas (5%). suele afectar el curso del embarazo (pueden mejorar en el em-
Parálisis de pares craneales (especialmente del VII, produ- barazo y recaer tras el parto). En los varones lo más frecuente
ciendo una parálisis facial unilateral que suele ser brusca y es el agrandamiento testicular asintomático y la epididimitis.
transitoria). Meningitis linfocitaria.
- Cardiovasculares. Formas clínicas especiales
Cardiopatía (20-30% en necropsias) con poca repercusión clí- Cabe destacar dos síndromes en la forma aguda de la enfer-
nica, pero son frecuentes las arritmias. medad: el síndrome de Heerfordt-Waldenström o fiebre
- Renales. uveoparotídea (fiebre, uveítis, parotiditis y parálisis facial) y el
• Hipercalciuria (60%), hipercalcemia (10-20%) y nefropatía síndrome de Löfgren que reúne eritema nodoso, adenopa-
cálcica (nefrocalcinosis, litiasis renal, uropatía obstructiva). tías hiliares (generalmente bilateral y simétrica), fiebre y artral-
• Glomerulopatías (membranosa, proliferativa, membranopro- gias. Este último es muy frecuente en nuestro país (30- 40%
liferativa, focal). de todos los casos).
• Nefritis intersticiales agudas y crónicas.
- Endocrinológicas.
Las más frecuentes son las lesiones hipotalamohipofisarias
(diabetes insípida, hipopituitarismo).
REGLA MNEMOTÉCNICA
- Osteoarticulares. El síndrome de Löfgren es una FAENA:
• Osteopatía (5%). Fiebre
Falanges de manos y pies (osteítis cistoide de Jüngling), meta- Artralgias
carpianas, metatarsianas, cuerpos vertebrales, calota y pelvis. Eritema Nodoso
• Artritis aguda o crónica (artritis granulomatosa, de evolución Adenopatías hiliares
tórpida hacia la destrucción articular), ambas generalmente
de grandes articulaciones y migratorias. Autor: Víctor Manuel González Cosano
• Miopatía sarcoidea (la biopsia muscular es rentable).
- Glándulas exocrinas.
Es típico el aumento bilateral de las parótidas, pero sólo dan (Ver figura 3)
clínica en el 10% de los casos. Puede haber afectación de las
glándulas salivales, apareciendo xerostomía.

Diabetes insípida
Uveítis anterior

Parálisis facial
Hipertrofia parotídea
Congestión y pólipos nasales
Adenopatías no dolorosas Fibrosis pulmonar
Adenopatías hiliares
Cardiopatía subclínica

Afectación histológica Esplenomegalia

Artralgia/artritis migratoria de Hipercalciuria


grandes articulaciones

Citopenias

Eritema nodoso

Figura 3. Órganos afectados en la sarcoidosis.

76
Tema 11 · Enfermedades intersticiales 3: neumonitis asociadas a procesos no bien conocidos

Diagnóstico
Radiografía de tórax
El 90% de los pacientes presenta anomalías en la radiografía
de tórax en algún momento de la evolución de la enfermedad.
Lo más característico es la presencia de adenopatías hiliares
bilaterales y simétricas que, en ocasiones aparecen calcificadas
en “cáscara de huevo”. Cuando hay afectación pulmonar
aparecen infiltrados intersticiales bilaterales, especialmente en
campos superiores, y, en formas terminales, fibrosis pulmo-
nar. La clasificación de la sarcoidosis, según su forma de pre-
sentación en la radiografía de tórax, está muy extendida, pero
no tiene valor predictivo de la actividad ni valor pronóstico.

Pruebas de función respiratoria


Demuestran las alteraciones típicas de las enfermedades inters-
ticiales: disminución de la DCLO con patrón restrictivo (dismi-
nución de la CPT y la CV). La CV es el indicador más sensible Figura 4. Biopsia de mucosa bronquial que muestra un granuloma de tipo
en el curso de la enfermedad. En ocasiones puede haber una sarcoide.
limitación al flujo aéreo en fases avanzadas, dando un patrón
obstructivo. Recuerda que...
El LBA en la sarcoidosis se caracteriza por
Estudios de laboratorio un cociente CD4/CD8 aumentado.
Muestran linfopenia, a veces eosinofilia, hipergammaglobu-
linemia y aumento de la VSG. Hay también elevación de los El diagnóstico de sarcoidosis requiere siempre:
niveles séricos del enzima de conversión de la angiotensina 1. Manifestaciones clinicorradiológicas compatibles
(ECA). No obstante, la elevación de la ECA no es tan fiable para 2. Demostración de granulomas sarcoideos en muestras histológicas
valorar la actividad de la sarcoidosis como se pensaba hace
unos años. Hoy en día se utilizan la progresión clínica, radio-
lógica y funcional para valorar la actividad de la enfermedad.
Tratamiento
El tratamiento de la sarcoidosis es controvertido, debido a que
Gammagrafía con galio-67 la enfermedad puede remitir espontáneamente, y a la varia-
Suele estar alterada, mostrando un patrón de captación difusa. bilidad de las manifestaciones clínicas iniciales, su gravedad y
evolución.
Prueba de Kwein El tratamiento inicial consiste en la administración de gluco-
corticoides. No existe consenso respecto al inicio, duración,
Consiste en la inyección intradérmica de un extracto de bazo
dosis e indicaciones. No obstante, como resultado de estudios
con sarcoidosis y posterior biopsia. Ha perdido valor por la
controlados y de otros estudios abiertos, se deducen pautas
dificultad para obtener antígeno homologado, por una estan-
más o menos uniformes. Su indicación está bien definida en
darización deficiente y por ser potencial transmisor de enfer-
la sarcoidosis extrapulmonar grave, principalmente en las
medades. La respuesta a las pruebas cutáneas de hiper-
afecciones cardiaca, neurológica, ocular, hepática, muscular y
sensibilidad retardada puede estar disminuida durante los
cutánea, y en la hipercalcemia.
periodos de actividad de la enfermedad (anergia cutánea).
En la sarcoidosis pulmonar, los glucocorticoides son eficaces
a corto y a medio plazo, pero no está demostrado que modifi-
LBA quen el curso de la enfermedad. En el estadio I no está indica-
Muestra un aumento de linfocitos (fundamentalmente lin- do el tratamiento debido a la elevada frecuencia de resolución
focitos T helper, CD4), por lo que el cociente CD4/CD8 está espontánea. En los estadios II y III, se instaurará tratamiento
aumentado (los valores normales del índice CD4/CD8 son de si existe sintomatología y/o alteraciones funcionales respirato-
0,4-1,8, y se considera muy sugestivo de sarcoidosis los niveles rias; cuando no hay sintomatología o alteraciones funcionales,
superiores a 3,5). el tratamiento debe iniciarse a los 6 meses del diagnóstico si
persisten los infiltrados intersticiales o cuando existan signos de
progresión de la enfermedad. En el estadio IV, deben tratarse
Confirmación histopatológica
todos los pacientes, aunque en general responden mal a los
Para el diagnóstico de sarcoidosis resulta imprescindible la glucocorticoides.
demostración de granulomas sarcoideos en muestras histoló-
La dosis inicial es de 40 mg/día de prednisona, o dosis equiva-
gicas dentro de un contexto clínico y radiológico compatible.
lente de otro glucocorticoide por vía oral durante un mes, que
La biopsia transbronquial es la más útil para la demostración
se disminuye de forma paulatina.
de los granulomas, por su alta sensibilidad. La elección de
otros órganos para la práctica de biopsias dependerá de su El tiempo de tratamiento debe ser como mínimo de 1 año para
afectación. la forma pulmonar y de 2 años para la extratorácica.

77
Manual ENARM · Neumología y Cirugía Torácica

liferación anormal de células musculares lisas en localización


Tratamiento sarcoidosis peribronquiolar, perivascular, perilinfática y en el intersticio
pulmonar.

Indicaciones absolutas tratamiento No indicado Clínica


- Estadio IV - stadio o asintom ticos
- Estadio III con indicios actividad y con pruebas funcionales Aparte de los síntomas propios de las EPI, la linfangioleiomio-
- Estadio II cuando: respiratorias normales o matosis se caracteriza por presentar neumotórax recidivante
• mpeora cl nica pulmonar le emente alteradas (69%), quilotórax (23%), hemoptisis (20%). La presencia de
• Deterioro función pulmonar
• ro resión lesiones en o TAC quilotórax, aunque es muy infrecuente, es muy característica
- Estadio I: Corticoides (8-12 meses) de esta enfermedad. Existe una frecuente asociación (60% de
• Adenopat as persisten meses -( m d a reduciendo los casos) con angiomiolipomas renales.
• ro resión enfermedad pro resi amente
- m da En las fases más evolucionadas de la enfermedad, la radio-
• Afectación cardiaca grafía de tórax y la TACAR muestran imágenes quísticas
Indicaciones extratorácicas • Afectación C
- Cardiacas de pared fina, sin predominio zonal. En un contexto clínico
- enales Corticoides tópicos apropiado, las imágenes de la TACAR son muy indicativas del
- C - arcoidosis cut nea diagnóstico. El patrón espirométrico muestra generalmente
- esiones deformantes piel - ritis un trastorno obstructivo, junto con un aumento de volúmenes
- a aérea superior - e tis
- nsuficiencia ep tica - Afectación a aérea pulmonares. Es característica la disminución de la DLCO.
- Debilidad artral ias fati a
- ipercalcemia ecurrencias postrasplante

Figura 5. Manejo de la sarcoidosis.

Recuerda que...
En la mayoría de los casos, la evolución de la sarcoidosis es
benigna autolimitada, por lo que generalmente no se trata. Sólo
se instaura tratamiento a partir del estadio II en la afectación
pulmonar y cuando existe afectación extrapulmonar grave.

Pronóstico
Es una enfermedad granulomatosa sistémica de causa desco-
Figura 6. Múltiples imágenes quísticas y neumotórax derecho en paciente con
nocida, con tendencia a la resolución (ya sea espontánea o con
linfangioleiomiomatosis.
tratamiento esteroideo).
En un tercio de los casos se produce remisión espontánea en
1-2 años; otro tercio muestra progresión y el resto permanecen Diagnóstico
estables. El diagnóstico definitivo se establece mediante estudio histo-
La mortalidad atribuible a la sarcoidosis es de un 10%. lógico por biopsia transbronquial o biopsia pulmonar abierta.
El anticuerpo monoclonal HMB-45 tiñe de forma selectiva la
proliferación muscular de la linfagioleiomiomatosis.
11.2. Linfangioleiomiomatosis e histocitosis X
Tratamiento
Dada la posible intervención de factores hormonales en la
Recuerda que...
patogenia, se han empleado diversas terapias hormonales, con
La linfangioleiomiomatosis y la histiocitosis X son enfermedades resultados variables. Los mejores resultados se han obtenido
“raras” con características propias, distintas del resto de con la administración de acetato de medroxiprogesterona.
enfermedades intersticiales: Si no se produce respuesta, se recomienda la realización de
- Debutan como neumotórax o derrame pleural en lugar de con ooforectomía.
disnea de esfuerzo.
- El patrón radiológico es quístico o noduloquístico.
- El patrón espirométrico puede ser restrictivo u obstructivo. Pronóstico
- No responden a los corticoides. Se trata de una enfermedad de mal pronóstico. La mayoría de
- No se hace trasplante pulmonar, porque recidivan. los casos evoluciona hacia la destrucción microquística difusa
de los pulmones, que conduce a la insuficiencia respiratoria
grave, muriendo a los 7-8 años del diagnóstico.

Linfangioleiomiomatosis
La linfangioleiomiomatosis es una rara enfermedad multisisté- Histiocitosis X o granulomatosis de células de Langerhans
mica de causa desconocida, que afecta casi exclusivamente a o granuloma eosinófilo
mujeres en edad fértil, por lo que es probable que intervengan La histiocitosis X se caracteriza por una infiltración bronco-
factores hormonales en su patogenia. Se caracteriza por la pro- vascular por células del sistema monocítico-macrofágico que

78
Tema 11 · Enfermedades intersticiales 3: neumonitis asociadas a procesos no bien conocidos

ocasiona progresivos cambios estructurales en el pulmón, y Diagnóstico


termina produciendo una fibrosis, que determina una insufi- Se establece por los hallazgos de la TACAR, la biopsia trans-
ciencia respiratoria. bronquial y el LBA, en el que las células de Langerhans
En la patogenia de la histiocitosis X se ha implicado el taba- suponen más del 5% de las células. En caso de duda, debe
quismo, debido a que la enfermedad es rara en no fumadores recurrirse a la biopsia transbronquial o pulmonar abierta. Las
y a que el pulmón de los fumadores contiene más células de células de Langerhans de la histiocitosis X tienen antígenos en
Langerhans que el de los no fumadores. su superficie que pueden ser identificados mediante anticuer-
pos monoclonales, como el CD1 o la proteína S-100 y por la
presencia de inclusiones citoplasmáticas, denominadas gránu-
Clínica
los de Birbeck, visibles con microscopia electrónica.
La enfermedad afecta a adultos jóvenes, fumadores, y se
manifiesta con tos y disnea de esfuerzo progresiva. Un 25% de
los casos debuta en forma de neumotórax espontáneo. En Tratamiento
ocasiones se asocia a quistes óseos solitarios en el cráneo, El pronóstico es variable, ya que la enfermedad puede resol-
huesos largos, costillas y pelvis. También puede asociarse a verse espontáneamente, permanecer estable o evolucionar
diabetes insípida en un 15% de los casos, teniendo en estos a la fibrosis pulmonar. La sintomatología y las alteraciones
casos un peor pronóstico. funcionales y radiográficas pueden mejorar con el abandono
La radiografía de tórax muestra infiltrados intersticiales con del hábito tabáquico. Se han empleado la penicilamina y los
pequeños espacios quísticos de predominio en campos supe- corticoides, con resultados variables.
riores. En fases avanzadas aparecen zonas con patrón en panal
de abeja. En la TACAR se observan con claridad los quistes
aéreos, con paredes bien definidas, y sus hallazgos orientan LINFANGIOLEIO-
HISTIOCITOSIS X
al diagnóstico. El patrón ventilatorio también muestra aumen- MIOMATOSIS
to de los volúmenes pulmonares y disminución de la DLCO.
Aunque puede mostrar un patrón obstructivo, según progresa
EPIDEMIO- Varón joven fumador Mujer en edad fértil
la enfermedad predomina la alteración ventilatoria restrictiva.
LOGÍA

FACTORES Tabaco Estrógenos


IMPLICADOS

Neumotórax recurrentes
CLÍNICA Neumotórax recurrentes Quilotórax (derrame pleural)
Hemoptisis

ASOCIACIÓN Quiste óseo solitario


EXTRA- Diabetes insípida
Angiomiolipomas renales
PULMONAR
Rx:
- Patrón reticulonodular TACAR:
PRUEBA DE y quistes en LLSS - Quistes de pared fina
IMAGEN - Patrón en panal de sin predominio zonal
abeja

LBA: >5% de células Biopsia:


ESTUDIO de Langerhans, CD1+ Células de músculo liso
CITOLÓGICO ME: gránulos de Birbeck HMB-45+

Abadono del tabaco Progesterona


TRATA- Penicilamina Ovariectomía
MIENTO Trasplante. Recidiva Trasplante. Recidiva

Tabla 2. Características diferenciales de la histiocitosis X y la linfangioleiomio-


matosis.

Recuerda que...
Penicilamina en el examen
“La penicilamina H A C E 3 efectos adversos:
gastrointestinales (más frecuentes), leucopenia (más grave) y
síndrome nefrótico (más típico)”
Enfermedades que se tratan con penicilamina:
Histiocitosis X
Artritis reumatoide
Cistina (litiasis por cistina)
Esclerodermia
→ Wilson
W

Figura 6. Quistes óseos en fémur en paciente con histiocitosis X (flechas).

79
Manual ENARM · Neumología y Cirugía Torácica

11.3. Eosinofilias pulmonares nofilia en sangre periférica. Además, habitualmente se realiza


una fibrobroncoscopia, donde un lavado broncoalveolar con
abundantes eosinófilos (entre 30-50%) termina de confirmar el
Las eosinofilias pulmonares son un grupo heterogéneo de
diagnóstico. Tiene una respuesta espectacular a los corticoides,
neumonitis intersticiales caracterizadas por la presencia de
con rápida mejoría de la clínica, radiología y disminución de la
infiltrados pulmonares eosinofílicos y de eosinofilia en
eosinofilia sanguínea. Sin embargo, son típicas las recaídas tras
sangre periférica.
la suspensión del tratamiento, lo que obliga a la administración
de una dosis de mantenimiento de corticoides a largo plazo.
Síndrome de Löeffler
Puede ser tanto idiopática como debida a fármacos o a hiper-
sensibilidad al Ascaris lumbricoides, en cuyo caso el tratamien- NEA NEC
to será el mebendazol. Tiene un curso benigno y cursa con EPIDEMIO- Cualquier edad,
infiltrados pulmonares intersticiales y/o alveolares migratorios Mujeres adultas
LOGÍA cualquier sexo
y mínimas manifestaciones clínicas.
Similar a una tbc o tumor
Similar a una neumonía
(fiebre y sudoración
Neumonía eosinófila aguda infecciosa (fiebre, tos,
CLÍNICA disnea, crepitantes,
nocturnas, astenia y
Debuta con una clínica similar a la de una neumonía infecciosa, pérdida de peso, tos,
con fiebre alta, tos, disnea, mialgias, crepitantes auscultatorios hipoxemia)
disnea) con asma
e hipoxemia grave. La radiografía de tórax muestra infiltrados
pulmonares alveolointersticiales bilaterales (similares al síndro- Infiltrados bilaterales
RADIO- Infiltrados bilaterales,
periféricos
me de dificultad respiratoria del adulto) y ausencia de historia LOGÍA perihiliares, migratorios
de asma. Existe eosinofilia objetivable en el LBA, pero no es (negativo del EAP)
constante en sangre periférica. Suele evolucionar bien, con Eosinófilos en LBA pero
buena respuesta a los corticoides sistémicos y no recidiva. LABO- Eosinófilos en LBA y
no siempre en sangre
RATORIO sangre periférica
periférica
Neumonía eosinófila crónica
RECIDIVA No Sí
Suele afectar a mujeres de edad avanzada y tiene una evolu-
ción subaguda, con importantes síntomas generales (fiebre, TRATA-
Corticoides
sudoración nocturna, tos, anorexia, pérdida de peso) de MIENTO
semanas o meses de duración. Suele asociarse a una historia
de asma en la mitad de los casos. Típicamente la radiografía Tabla 3. Principales características de la neumonía eosinófila aguda (NEA) y
de tórax muestra infiltrados bilaterales periféricos (imagen en neumonía eosinófila crónica (NEC).
negativo del edema agudo de pulmón). El diagnóstico de neu-
monía eosinófila crónica se establece mediante una situación
clínico-radiológica compatible junto con el hallazgo de eosi- Síndrome hipereosinofílico
Es más frecuente en varones de edad media. Se caracteriza por
eosinofilia periférica (>1.500/µl) durante 6 o más meses (en
ausencia de una causa objetivable) y disfunción de múltiples
órganos y sistemas por infiltración difusa por eosinófilos (cora-
zón, pulmón, hígado, piel, sistema nervioso). El corazón es el
órgano más afectado y la principal causa de morbimortalidad.
Se trata con corticoides y/o hidroxiurea.
Las eosinofilias pulmonares pueden ser también debidas a
infestaciones parasitarias o reacciones a fármacos como
sulfamidas, hidralacina, nitrofurantoína o clorpropamida. La
eosinofilia pulmonar por fármacos suele cursar con tos, disnea,
fiebre y escalofríos. Radiográficamente presentan un patrón reti-
culonodular en las bases y su tratamiento consiste en la retirada
del fármaco y si no responde se puede probar con corticoides.

Aspergilosis broncopulmonar alérgica (ABPA)


La aspergilosis broncopulmonar alérgica (ABPA) es debida a
una reacción inmunológica de hipersensibilidad, fundamental-
mente de tipo I o III, frente a la colonización crónica de la vía
aérea por Aspergillus fumigatus.
La ABPA suele afectar a pacientes de cualquier edad, con
historia previa de asma bronquial, sobre todo si han sido
tratados con corticoides, pues éstos favorecen el crecimiento
intrabronquial del hongo. Se manifiesta típicamente por clínica
de asma (tos, disnea y sibilancias), junto con febrícula, malestar
y expectoración de auténticos tapones mucosos marronáceos.
Figura 8. Imagen radiológica característica de la neumonía eosinófila crónica, La radiografía de tórax muestra infiltrados pulmonares recu-
con infiltrados bilaterales periféricos. rrentes junto a bronquiectasias centrales en dedo de guante.

80
Tema 11 · Enfermedades intersticiales 3: neumonitis asociadas a procesos no bien conocidos

Existen unos criterios mayores y otros menores para el diagnós- Clínica


tico de la enfermedad. Se corresponde con la clínica general de cualquier enfermedad
El tratamiento de la ABPA son los corticoides por vía sistémica. intersticial, con tos seca y disnea. Estos pacientes tienen un
riesgo aumentado de padecer infecciones oportunistas por P.
jiroveci, M. avium y Nocardia.

Figura 9. Bronquiectasias centrales en el TC de un paciente con ABPA. Figura 10. Infiltrados radiológicos en la proteinosis alveolar.

- Asma bronquial crónica


Diagnóstico
- Infiltrados pulmonares transitorios Radiografía de tórax
- Prueba cutánea de hipersensibiliadad inmediata La radiografía de tórax muestra infiltrados alveolares bilaterales
positiva a Aspergillus y simétricos más marcados en las regiones perihiliares (en alas
- Precipitinas séricas IgG frente a Aspergillus de mariposa), que recuerdan al edema pulmonar.
MAYORES - Aumento de IgE total sérica
- Bronquiectasias centrales o proximales
- Eosinofilia sanguínea (>1.000/µl) LBA
- Aumento de IgG e IgE específicas positiva a El diagnóstico suele establecerse por los hallazgos del LBA. El
Aspergillus líquido obtenido con el LBA tiene un aspecto lechoso y está
formado por macrófagos cargados de lípidos y material PAS
- Cultivo de Aspergillus en esputo positivo.
MENORES - Tapones mucosos en esputo
(O SECUNDARIOS)
- Reacción cutánea retardada a Aspergillus
Anatomía patológica
El diagnóstico de ABPA requiere cinco criterios mayores En ocasiones es necesario recurrir a la biopsia transbronquial
o cuatro mayores y dos menores o incluso abierta, que muestra macrófagos alveolares cargados
de inclusiones (cuerpos laminares), estando las paredes alveo-
Tabla 4. Criterios diagnósticos de la ABPA. lares típicamente respetadas.

Tratamiento
11.4. Proteinosis alveolar Se trata con LBA repetidos.

Se trata de una rara enfermedad caracterizada por la acumu-


lación anormal en los alvéolos de los constituyentes normales
del surfactante.

81
Tema 12
Enfermedades del mediastino

12.1. Masas mediastínicas Teratoma


Puede asociar tirotoxicosis, ginecomastia e hipoglucemia.
Los tumores mediastínicos más frecuentes, considerando su Radiográficamente se caracteriza por la presencia de calcifi-
frecuencia global (adultos y niños) son los neurogénicos. Para caciones.
el diagnóstico suele realizarse una radiografía de tórax, que
aporta información muy valiosa, pero la prueba de imagen de Hernia diafragmática de Morgagni
elección es la TC de tórax. El diagnóstico definitivo es histoló-
gico y para ello suelen emplearse técnicas invasivas, puesto que
la obtención de muestras por punción-aspiración con aguja Masas del mediastino medio
fina sólo da una aproximación citológica. Así, para obtener Adenopatías
tejido para estudio histológico, suele realizarse una mediastino-
Causa más frecuente de masa en el mediastino medio.
tomía anterior, mediastinoscopia o videotoracoscopia, según
Reactivas, tumorales o por linfoma.
la masa se sitúe en mediastino anterior, medio o posterior,
respectivamente.
Otros
Masas del mediastino anterior En el mediastino medio se localizaría la patología del peri-
cardio, grandes vasos, esófago y bronquios. Entre las masas
Timoma destacan los quistes congénitos (broncogénicos los más
Masa más frecuente en este compartimento. Puede asociarse frecuentes, entéricos en 2.º lugar, y pericárdicos en 3.er lugar).
a miastenia gravis (10-50%), aplasia pura de la serie roja, hipo/
agammaglobulinemia, síndrome de Cushing, megaesófago y
colagenosis. Masas del mediastino posterior
Tumores neurogénicos
(Schwannomas, neurofibroma, neuroblastoma, paraganglio-
ma, feocromocitoma). Son los más frecuentes de este com-
partimento.

Otros
Hematopoyesis extramedular, hernia de Bochdaleck y pseudo-
quiste pancreático.

Recuerda que...
Masas en madistino anterior: “4 T”
Timo
Tiroides
Teratoma
Terrible linfoma

12.2. Mediastinitis
Figura 1. Timoma. Masa bien delimitada y de bordes nítidos en mediastino
anterior (la flecha señala su límite posterior). Mediastinitis aguda
Las causas más frecuentes son la perforación esofágica (por
Masas tiroideas traumatismos penetrantes, sobre todo iatrógenos tras la rea-
lización de una endoscopia digestiva alta; o bien espontánea
-síndrome de Boerhaave- tras esfuerzos esofágicos violentos,
Linfoma como vómitos repetitivos), y la infección postoperatoria precoz
Son radiosensibles y pueden asociar hipercalcemia. Como el tras una esternotomía media.
resto de masas del mediastino anterior, generalmente se pre-
sentan como ensanchamiento mediastínico en la radiografía
de tórax.

82
Tema 12 · Enfermedades del mediastino

Diagnóstico Clínica
El paciente con mediastinitis aguda presenta un dolor torácico Puede ser desde asintomática hasta manifestarse con pulso
retroesternal, intenso y transfixiante, clínica infecciosa (fiebre, paradójico y edema en esclavina.
escalofríos) y disnea. La auscultación puede evidenciar el signo
de Hamman, aunque éste no es específico, que consiste en la
Tratamiento
percepción de un crujido retroesternal sincrónico con el latido
cardiaco. Las radiografías en bipedestación pueden mostrar El tratamiento incluye toracotomía y exéresis de la fibrosis
aire en el mediastino (neumomediastino), neumotórax o hidro- existente.
neumotórax.

Tratamiento
Se trata mediante desbridamiento, drenaje quirúrgico y anti-
bióticos de forma precoz.

Pronóstico
En el caso de la rotura de esófago, el pronóstico depende de la
precocidad en el tratamiento quirúrgico.

Mediastinitis crónica
Diagnóstico
El cuadro clínico es subagudo y se sospecha ante la presencia
de antecedentes compatibles como son inflamaciones crónicas
tuberculosas pleurales o pericárdicas, histoplasmosis, silicosis,
ciertos fármacos (como la metisergida), etc. Figura 2. Mediastinitis necrotizante con neumomediastino (flecha).

83
Tema 13
Hipertensión pulmonar (HTP)
Concepto 13.1. Hipertensión arterial pulmonar idiopática
Se define como la existencia de una presión media en la arteria
pulmonar (PAPm) >20 mmHg en reposo. Puede deberse a un Se trata de un cuadro raro (5-15 casos por millón), de etiología
aumento aislado de la presión en el sistema arterial pulmonar desconocida, que suele afectar a mujeres entre 30-40 años.
(HTP precapilar, o hipertensión arterial pulmonar); o a una
elevación de la presión en los sistemas venoso y capilar (HP
postcapilar, o hipertensión venosa pulmonar). Puede tener Clínica
un curso progresivo y letal sin tratamiento, sin bien la tasa de Su presentación clínica es relativamente inespecífica, por lo
progresión es muy variable. que el retraso diagnóstico (entre 2-3 años de media) es lo
habitual. El síntoma más común es la disnea progresiva de
esfuerzo, añadiéndose episodios de dolor torácico atípico y
Clasificación síncope. En la exploración destaca el incremento de la presión
- Grupo 1: hipertensión arterial pulmonar (HAP). venosa yugular y el refuerzo del 2.º tono.
Antiguamente llamada hipertensión pulmonar primaria. Exis-
ten varios subgrupos:
• HAP idiopática. Anatomía patológica
• HAP familiar. Es inespecífica y heterogénea; la alteración más precoz es la hiper-
• HAP inducida por drogas (típicamente anorexígenos) y toxinas. trofia de la media, apareciendo posteriormente cambios plexogé-
• HAP asociada a conectivopatías, VIH, hipertensión portal, nicos (proliferación y fibrosis de la íntima) y trombos recanalizados.
cardiopatías congénitas o esquistosomiasis. Variante: enfermedad venooclusiva/hemangiomatosis
• Enfermedad venooclusiva pulmonar/hemangiomatosis capi- capilar pulmonar: se trata de un raro subtipo de HAP carac-
lar pulmonar. terizada por la oclusión difusa de las venas pulmonares, pro-
• Hipertensión pulmonar persistente del recién nacido. vocando una dilatación tortuosa de los capilares pulmonares.
- Grupo 2: hipertensión pulmonar secundaria a cardiopatías
izquierdas.
Diagnóstico
- Grupo 3: hipertensión pulmonar secundaria a neumopatía
crónica/hipoxia. El diagnóstico es de exclusión, tras descartar posibles causas
de HTP secundaria, fundamentalmente cardiopatías y neumo-
- Grupo 4: enfermedad tromboembólica crónica. patías. Por lo general, requiere de muchas exploraciones para
- Grupo 5: hipertensión pulmonar de mecanismo no acla- su diagnóstico:
rado/multifactorial. - Gasometría arterial basal.
Asociada a cuadros hematológicos (anemias hemolíticas cró- Casi siempre existe hipoxemia e hipocapnia.
nicas), sistémicos (p. ej., sarcoidosis) y desórdenes metabólicos - Radiografía de tórax.
(p.ej., enfermedad de Gaucher). Arterias pulmonares centrales prominentes con disminución
de la vascularización periférica, junto con crecimiento de aurí-
Epidemiología cula y ventrículo derechos.
Se desconoce con exactitud la prevalencia de la HTP en la - Electrocardiograma.
población general. La mayoría de los casos es secundaria a Signos de hipertrofia del ventrículo derecho, con eje desviado
cardiopatía izquierda (Grupo 2, por aumento de presión pos- a la derecha.
capilar) y a neumopatías crónicas graves (Grupo 3, por vaso- - Ecocardiograma.
constricción hipóxica sostenida; que puede llegar a producir Aumento del tamaño del ventrículo derecho, anormalidad del
sobrecarga de cavidades derechas -situación conocida como tabique por sobrecarga derecha y llenado del ventrículo iz-
cor pulmonale). quierdo anormal.
- Pruebas funcionales respiratorias.
Una vez se ha descartado la existencia de una cardiopatía o
Patrón restrictivo o normal, y disminución de la capacidad de
neumopatía que justifique la HTP, cabría considerar otras cau-
difusión del CO (DLCO).
sas menos frecuentes, como la enfermedad tromboembólica
- Gammagrafía de perfusión.
crónica (Grupo 4), y factores predisponentes al desarrollo de
En la HAP la gammagrafía es normal o muestra defectos de
una HAP (Grupo 1, por afectación primaria de las arteriolas
perfusión subsegmentarios, a diferencia de la enfermedad
pulmonares): fármacos anorexígenos, aceite tóxico, inhalación
tromboembólica crónica, donde aparecen múltiples y grandes
de cocaína, VIH, conectivopatías (sobre todo esclerosis sitémi-
defectos segmentarios. Ante una gammagrafía de perfusión
ca), hipertensión portal, cardiopatías congénitas o esquistoso-
patológica que oriente a enfermedad tromboembólica cró-
miasis. Sólo en aquellos casos en los que no se detecta ninguna
nica, se debe realizar una arteriografía pulmonar para confir-
de las anteriores se habla de HAP idiopática.
mar el diagnóstico.
- Cateterismo cardiaco.
El diagnóstico definitivo de HAP es hemodinámico, me-
diante la realización de un cateterismo derecho que demues-
tra un aumento en la PAPm con presión de enclavamiento
capilar pulmonar normal ( 15 mmHg), a diferencia de lo que
ocurre en la HTP secundaria a enfermedad cardiaca.

84
Tema 13 · Hipertensión pulmonar (HTP)

Recuerda que... 2 y 3 por su potencial efecto deletéreo. En los grupos 4 y 5 se


debe individualizar.
El cateterismo cardiaco derecho con prueba de vasorreactividad
Para elegir el tratamiento a implementar, se realiza previamen-
(VR) debe realizarse en el momento del diagnóstico para identifi-
te un test de vasorreactividad pulmonar: se monitoriza de
car a los pacientes que puedan beneficiarse de una terapia a
forma invasiva (mediante cateterismo) la PAP, y se administran
largo plazo con bloqueadores de calcio.
vasodilatadores potentes de corta acción (el más utilizado es el
óxido nítrico). El test es positivo si hay un descenso significativo
de la PAP con mantenimiento (o mejoría) de la TA y del gasto
Tratamiento cardiaco.
(Se estudia en el tema 13.2)
Test positivo
Son candidatos a recibir calcioantagonistas (nifedipino,
13.2. Tratamiento general de la HTP
diltiazem), parecen mejorar la sintomatología y quizás la
supervivencia. Hay pacientes con poca o nula respuesta clínica,
Tratamiento etiológico y entre los respondedores el efecto suele ir disminuyendo con
el tiempo. No están indicados si el test es negativo.
- Grupo 1 (HAP).
Abandonar las exposiciones de riesgo (drogas/tóxicos), con-
trolar las enfermedades de base (conectivopatías, VIH) si es Test negativo (o pérdida de respuesta a calcioantagonis-
que existen. En general este grupo suele requerir tratamiento tas con el tiempo)
avanzado para la HTP (ver más adelante). El tratamiento se elige según la clase funcional (OMS) del
- Grupo 2 (cardiopatía izquierda). paciente:
Optimizar el control cardiológico (p. ej., revascularización co-
ronaria, reparación valvular, tratamiento de la insuficiencia - Asintomáticos (clase I).
cardiaca). Se realiza observación clínica y tratamiento general.
- Grupo 3 (neumopatía crónica). - Clase II-III.
Tratamiento neumológico (p. ej., abandonar el tabaco, bron- Se utilizan combinaciones de vasodilatadores pulmonares. La
codilatadores, antifibróticos, CPAP, etc.) y oxigenoterapia combinación de primera línea suele ser ambrisentan + tada-
contínua domiciliaria (para frenar la vasoconstricción hipó- lafilo v.o.
xica). • Antagonistas de la endotelina, v.o. (ambrisentan, bosentan,
- Grupo 4 (tromboembolismo crónico). macitentan).
Anticoagulación y considerar endarterectomía quirúrgica (ver • Inhibidores de la fosfodiesterasa 5, v.o. (sildenafilo, tadala-
más adelante). filo).
• Estimulantes de la guanilato ciclasa, v.o. (riociguat).
• Agonistas del receptor de prostaciclina, v.o. (selexipag).
Tratamiento general
• Agonistas de prostaciclinas (iloprost inhalado, treprostinil s.c.
- Diuréticos. o inhalado, beraprost v.o.).
Pueden ayudar a controlar la sobrecarga de volumen, tanto Sólo se usan en clase funcional III.
de forma crónica como en las descompensaciones agudas,
- Clase funcional IV.
pero deben emplearse con prudencia (riesgo de depleción de
Se recomienda perfusión continua de análogos de prostaci-
volumen y fracaso hemodinámico, pues los pacientes con HTP
clina (epoprostenol i.v.), como terapia de mantenimiento o
son muy dependientes de mantener una buena precarga).
como puente al trasplante.
- Oxígeno.
Indicado en los pacientes con HTP e hipoxemia.
- Anticoagulación.
Indicada en el grupo 4, y por lo general no indicada en los
grupos 2, 3 y 5. En el grupo 1 es controvertida (hasta hace
poco se recomendaba anticoagulación sistemática; hoy en día
el balance beneficio/riesgo es discutido). CLASE
- Inotrópicos. SÍNTOMAS
FUNCIONAL
La digoxina puede ser útil para controlar la frecuencia car-
diaca cuando la HTP se asocia a arritmias supraventriculares, Síntomas que no limitan la actividad física. La
Clase I
mejorando la contractilidad del VD. En situaciones de fracaso actividad física cotidiana no produce disconfort.
hemodinámico agudo, son útiles la dobutamina, milrinona y
el levosimendan. Limitación leve de la actividad física.
Clase II
- Otros. La actividad física cotidiana produce síntomas.
Programas de rehabilitación cardiaca, vacunaciones (gripe,
neumococo). Limitación marcada de la actividad física.
Clase III
Aparecen síntomas con mínima actividad física.
Tratamiento avanzado
Incapacidad de realizar actividad física.
Tratamientos dirigidos a combatir la propia HTP con indepen- En ocasiones aparecen síntomas incluso en
dencia de su causa. Se debe considerar para aquellos pacien- Clase IV
reposo, y se exacerban con cualquier actividad.
tes con deterioro de clase funcional a pesar del tratamiento Hay signos de insuficiencia cardiaca derecha.
etiológico, y por su complejidad debe controlarse en centros
experimentados. Por lo común, se indica para la mayoría de los
pacientes del Grupo 1, y no suele recomendarse en los grupos Tabla 1. Clasificación funcional de la HTP (OMS).

85
Manual ENARM · Neumología y Cirugía Torácica

Tratamiento general:
diuréticos +/- 02 +/- anticoagulantes

Test vasorreactividad

+ –

Calcioantagonistas Clase I Clase funcional II-III Clase IV

Observación y Epoprostenol IV
¿Respuesta? Tratamiento vasodilatador combinado
tratamiento general
1.º línea, vía oral: ambrisentan + tadalafilo No mejoría /
Alternativas: otras combinaciones, terapia única progresión clínica
+
Si mala respuesta:
- Teprostinil sc, iv o inh - Septostomía atrial
Continuar tratamiento - Epoprostenol iv - Trasplante cardiobipulmonar

Figura 1. Manejo avanzado de la hipertensión pulmonar.

Terapias adicionales La clínica suele ser disnea progresiva e intolerancia al ejercicio;


- Septostomía auricular. en casos avanzados pueden encontrarse edemas periféricos,
La creación de un shunt derecha-izquierda es una alternativa dolor torácico y síncopes.
paliativa en pacientes con HTP extrema refractaria a trata- El diagnóstico por lo común ocurre al realizar el estudio de
miento médico. Busca mejorar el gasto cardiaco, pero empo- una HTP de causa no aclarada, al encontrar defectos de per-
brece la oxigenación de la sangre por efecto shunt. fusión con ventilación normal en la gammagrafía V/Q (prueba
- Trasplante. más sensible), que se confirman mediante angiografía venosa
Generalmente cardio-bipulmonar (en algunos casos bipulmo- pulmonar (gold standard). En la mayoría se logra identificar
nar); no hay evidencia clara de recidiva en el injerto, pero la retrospectivamente al menos un evento tromboembólico pre-
mortalidad periquirúrgica es muy elevada. vio (pero para casi el 40% pasa desapercibido).
El tratamiento se basa en la anticoagulación indefinida y
(Ver figura 1) en la posibilidad de realizar una tromboendarterectomía
quirúrgica (sólo si los trombos son suficientemente proxima-
les), que se considera curativa. En casos no quirúrgicos, se ha
13.3. Enfermedad tromboembólica crónica empleado con discreta utilidad el riociguat.

Se trata del desarrollo de HTP crónica inducida por eventos


tromboembólicos venosos pulmonares. Ocurre en una minoría
de pacientes tras un tromboembolismo agudo, desconocién-
dose los factores predisponentes.

Pregunta ENARM
19. El crecimiento cardiaco que ocurre en los pacien-
tes que padecen cor-pulmonale crónico se inicia
en:

A. Aurícula izquierda.
B. Ventrículo derecho.
C. Aurícula derecha.
D. Ventrículo izquierdo.

· Encuentra las respuestas y comentarios de las preguntas ENARM al final del manual ·

86
Tema 14
Bronquiectasias
Concepto (Recuerda que cuando se asocien bronquiectasias + esterilidad
Las bronquiectasias son dilataciones anormales e irreversibles deberemos pensar en Kartagener, Young o fibrosis quística).
de los bronquios grandes (mayores a 2 mm) debidas a una
destrucción del tejido elástico, muscular y cartilaginoso de la Bronquiectasias localizadas
pared bronquial.
- Infecciones.
Generalmente virus (influenza y adenovirus) y bacterias (S. au-
reus, Klebsiella y anaerobios, y micobacterias).
- Obstrucción endobronquial.
Bronquios normales Tumores endobronquiales, cuerpo extraño, compresión ex-
trínseca, tuberculosis y otras enfermedades granulomatosas,
enfisema, etc.
- Otras causas.
Síndrome de Swyers-James-Macleod o síndrome del pulmón
hipertransparente unilateral, que asocia bronquiectasias loca-
lizadas e hipoplasia de la arteria pulmonar ipsilateral.

Clínica
Bronquioectasias La clínica es similar a la de un paciente EPOC pero más grave.
Se caracteriza por la presencia de tos crónica productiva con
expectoración purulenta de más de 150 ml/día (la secreción
bronquial de un sujeto sano es de unos 100 ml/día). Las bron-
Figura 1. Bronquiectasias. quiectasias son la principal causa de hemoptisis leve-modera-
da, normalmente en el contexto de una infección bronquial. Si
la enfermedad es grave pueden aparecer acropaquias.
Etiología
Bronquiectasias diseminadas
Diagnóstico
- Causa infecciosa.
La radiografía de tórax es poco sensible y puede mostrar infil-
Los adenovirus y el virus de la gripe son los principales virus
trado y pérdida de volumen (lo más frecuente), lesiones quís-
que producen bronquiectasias. Las infecciones por bacterias
ticas agrupadas, a veces con niveles hidroaéreos, imágenes en
necrotizantes, como Staphylococcus aureus, Klebsiella, anae-
“nido de golondrina”, “en raíl de vía” o “en anillo de sello”.
robios y tuberculosis son también causa de bronquiectasias
difusas. Asimismo, éstas pueden encontrarse en la infección Aunque la broncografía ofrece una excelente visualización
por VIH. de las vías respiratorias bronquiectásicas, hoy en día ha sido
La alteración en los mecanismos de defensa del hospedador sustituida por la TC de alta resolución, que además permite
predispone a las infecciones recurrentes y, por tanto, a la apa- planificar la intervención quirúrgica. Las pruebas funcionales
rición de bronquiectasias difusas. Es el caso de las siguientes suelen demostrar un patrón obstructivo.
enfermedades:
• Fibrosis quística.
• Discinesia ciliar primaria.
El subgrupo más importante es el síndrome de Kartage-
ner, en el que se asocian bronquiectasias, esterilidad, sinu-
sitis y situs inversus.
• Inmunodeficiencias congénitas o adquiridas.
Déficit selectivo de IgG, panhipogammaglobulinemia, agam-
maglobulinemia de Bruton.
- Causa no infecciosa.
• Aspergilosis broncopulmonar alérgica (ABPA).
• Exposición a sustancias tóxicas, como inhalación de amo-
níaco o aspiración del contenido ácido gástrico.
• Déficit de α1-antitripsina.
• Síndrome de Williams-Campbell.
Consiste en una deficiencia del cartílago bronquial.
• Síndrome de Young.
Reúne bronquiectasias, azoospermia obstructiva y sinusitis.
• Síndrome de uñas amarillas.
Cursa con bronquiectasias, sinusitis crónica, neumonía recu-
rrente, linfedema, derrame pleural y uñas amarillas.
- Síndrome de Mounier-Kuhn.
Junto con las bronquiectasias produce traqueobronquiomegalia. Figura 2. Bronquiectasias bilaterales en TC de alta resolución.

87
Manual ENARM · Neumología y Cirugía Torácica

Tratamiento Quirúrgico
Médico Ante clínica severa en un paciente con bronquiectasias loca-
Hidratación, fisioterapia respiratoria, drenaje postural, bron- lizadas y que lleva más de un año con tratamiento médico
codilatadores y antibioterapia de amplio espectro de forma sin obtener resultado se debe practicar segmentectomía o
precoz ante infecciones respiratorias. lobectomía. Ante un paciente con bronquiectasias y hemopti-
sis masiva resistente al taponamiento broncoscópico, se debe
realizar cirugía o embolización arterial bronquial.

88
Tema 15
Bronquiolitis
Concepto mas postrasplante, en cambio, son la base del tratamiento). Se
Inflamación de los bronquiolos respiratorios. han propuesto ciclos largos de macrólidos a dosis bajas, tera-
pias antirreflujo e inmunosupresores, con resultados dispares.

Variantes histológicas
Neumonía organizada (NO)
Bronquiolitis simple
Cursa de forma subaguda con disnea, tos, expectoración,
Inflamación de la pared del bronquiolo. febrícula/fiebre, pérdida de peso, astenia y malestar general;
encontrando en la Rx de tórax áreas de consolidación bilate-
Bronquiolitis obliterante rales periféricas, que con frecuencia son migratorias. Se ha
de sospechar ante bronconeumonías atípicas de evolución
Inflamación de la pared del bronquiolo, a la que se añaden
tórpida a pesar de repetidos ciclos de antimicrobianos. Suelen
tapones fibrosos en la luz.
presentar patrón restrictivo (se consideran un tipo de neu-
- Constrictiva. monía intersticial (se estudia en el tema 9)), deterioro del
Hay tejido fibroso alrededor del bronquiolo, comprimiéndolo. intercambio gaseoso y en el LBA predominio de linfocitos CD8
Se corresponde clínicamente con la bronquiolitis obliterante (cociente CD4/CD8 <1). El patrón de neumonía organizada
(idiopática y secundarias). (con bronquiolitis obliterante proliferativa en la biopsia) se
- Proliferativa. puede encontrar asociada a distintos trastornos (NO secunda-
El tejido fibroso se extiende desde los bronquiolos a los al- ria) o de forma primaria (NO criptogénica):
veolos adyacentes, ocupándolos. Se corresponde clínicamente
- NO secundaria:
con las neumonías organizadas (NO), siendo la variante cripto-
A drogas (amiodarona, cocaína, ciclofosfamida, betablo-
génica (NOC) la más frecuente.
queantes, bleomicina), conectivopatías, leucemias y linfomas,
radioterapia torácica, postrasplante. También pueden apare-
Variantes clínicas cer focos de neumonía organizada alrededor de otras lesiones
Bronquiolitis aguda infecciosa pulmonares primarias (otras neumonías intersticiales, abscesos
(Se estudia en Pediatría) pulmonares, daño alveolar difuso, etc.).
- NO criptogenética (NOC), antiguamente llamada bronquiolitis
obliterante con neumonía organizada (BONO).
Bronquiolitis obliterante Representa la mayoría de los casos de NO; para su diag-
Cursa con clínica de tos escasamente productiva y disnea nóstico se requiere un cuadro clínico y radiológico compati-
progresiva. La radiografía es poco expresiva. Hay obstrucción ble, junto con histología congruente (bronquiolitis obliterante
espirométrica con gran aumento del VR, desproporcionado al proliferativa) y exclusión de causas secundarias.
consumo de tabaco. En el LBA puede haber aumento de PMN.
Puede ser idopática o secundaria a distintas causas: El tratamiento de las causas secundarias se basa en evitar las
- Asociada a conectivopatías. exposiciones de riesgo y el control de las enfermedades subya-
Sobre todo artritis reumatoide, Sjögren, LES... centes. En la NOC, la base del tratamiento son los corticoides,
- Postrasplante. por lo general con buena respuesta.
Puede aparecer meses o años tras un trasplante pulmonar o
cardiopulmonar. También puede verse tras un trasplante he-
terólogo de médula ósea en forra de EICH crónico. NOC BO IDIOPÁTICA
- Otras.
Áreas de consolida-
Inhalación de tóxicos (cloro, gas mostaza, polvos inorgánicos…), Normal o
RX TÓRAX ción bilaterales
hiperinsuflación
drogas (cocaína base, busulfán), o postinfecciosa (VRS, gripe,
periféricas
sarampión, CMV, VVZ, tos ferina, Nocardia, Mycoplasma…).
- Idiopática. Alteración obstructiva
Es un diagnóstico de exclusión, al descartar causas secundarias. FUNCIÓN Restrictivo desproporcionada al
PULMONAR consumo de tabaco
El diagnóstico se basa en la clínica y los hallazgos de la TC
de alta resolución (patrón en vidrio deslustrado “en mosaico”, DLCO Disminuido Normal
que se magnifica en la TC realizada en espiración, lo que
traduce importante atrapamiento aéreo). En casos dudosos LBA ↑ CD8 ↑ PMN
se suele requerir una biopsia pulmonar abierta (aunque en las
RESPUESTA Buena (pero cortico-
Mala
bronquiolitis postrasplante sí son útiles las biopsias transbron-
CORTICOIDES dependiente)
quiales), que demuestre el patrón de bronquiolitis obliterante
constrictiva.
Tabla 1. Tabla comparativa de la NOC y BO idiopática.
El tratamiento se basa en retirar las exposiciones de riesgo
y en controlar la patología subyacente (conectivopatías, etc.)
si es que ésta existe. Además, se usan broncodilatadores y Recuerda que...
antitusígenos para control sintomático. Las formas idiopáticas La bronquiolitis obliteRAnte se asocia a Artritis Reumatoide
muestran escasa respuesta a corticoides sistémicos (en las for-

89
Tema 16
Síndrome de hemorragia alveolar difusa
Concepto Diagnóstico
La hemorragia alveolar difusa es una extravasación sanguínea Es un diagnóstico de exclusión. Es fundamental descartar un
que tiene su origen en el territorio alveolar (generalmente por Goodpasture. La presencia de coagulopatía, trombopenia, dis-
afectación de los pequeños vasos). Suele cursar con hemoptisis función hepática o daño renal/glomerulonefritis van en contra
(aunque puede faltar), disnea y anemia. del diagnóstico.

Recuerda que... Tratamiento


En la fase aguda se utilizan corticoides. Aunque no han
En el paciente con hemoptisis, de acuerdo a la GPC, el estudio que
demostrado alterar el curso de la enfermedad a largo plazo,
debe ser empleado de primera intención es la radiografía de tórax
parecen disminuir la mortalidad de los brotes y la evolución
PA y lateral. La fibrobroncoscopia está indicada si no se ha locali-
hacia fibrosis.
zado la causa de una hemoptisis no masiva. Si esto no es posible,
se recomienda la fibrobroncoscopia virtual. Si hay enfermedad celiaca asociada, pueden mejorar al intro-
ducir dieta sin gluten.

Etiología
Incluye entidades que afectan al pulmón y al riñón (como la
enfermedad de Goodpasture) y otras que son exclusivamente
pulmonares (característicamente la hemosiderosis pulmonar
idiopática). Pueden cursar con hemorragia alveolar, además
de los citados anteriormente, las vasculitis, conectivopatías,
estenosis mitral, tromboembolismo pulmonar (sobre todo,
embolia grasa), secundaria a fármacos como la D-penicilamina,
linfografía, linfangioleiomiomatosis, etc.

16.1. Hemosiderosis pulmonar idiopática (HPI)

Patogenia
La patogenia es desconocida, aunque se sospecha la participa-
ción de fenómenos autoinmunes (un 50% tiene elevación de
IgA y se asocia a la enfermedad celíaca y la intolerancia a las
proteínas de leche de vaca).

Figura 1. Imagen radiológica de la hemorragia alveolar difusa.


Clínica
Se caracteriza por hemorragia pulmonar recurrente que afecta
a niños y a adultos jóvenes sin afectación de otros órganos. Recuerda que...
Cursa con hemoptisis y disnea. Puede haber síndrome ané-
Para diferenciar la hemosiderosis pulmonar idiopática (HPI) del
mico pero no hay afectación renal ni anticuerpos anti-MBG.
síndrome de Goodpasture, recuerda que la HPI:
En la radiografía de tórax, se observan infiltrados pul-
- Es más frecuente en niños
monares alveolares bilaterales difusos. Generalmente el
- No afecta al riñón
patrón alveolar evoluciona a un patrón reticular a los 2-3 días
- No presenta anticuerpos anti MB
del episodio agudo y, en ausencia de sangrado repetido, la
radiología vuelve a la normalidad. Sin embargo, en caso de
hemorragia persistente o recidivante, el cuadro radiológico
evoluciona a un patrón reticulonodular persistente. Cuando
16.2. Enfermedad de Goodpasture
hay sangrado activo, es característico el aumento de la DLCO.
Se identifican macrófagos alveolares cargados de hemosiderina
(siderófagos) en el LBA, lo que constituye un dato diagnóstico (Se estudia en Nefrología)
relevante.

90
Tema 17
Enfermedad del diafragma

17.1. Parálisis del diafragma Parálisis bilateral


Se caracteriza por disnea de esfuerzo, ortopnea y respiración
Parálisis unilateral toracoabdominal paradójica. La causa más frecuente son los
traumatismos cervicales y torácicos (por lesión medular directa
Es frecuente, pero poco sintomática, por lo que
o lesión de ambos nervios frénicos), y la segunda causa son
suele ser un hallazgo casual al realizar una radio-
los trastornos neuromusculares (enfermedad de motoneurona,
grafía de tórax (elevación de un hemidiafragma).
esclerosis múltiple, Guillain Barré, miastenia gravis, etc.). El
El diagnóstico se confirma con radioscopia
diagnóstico radiológico es mucho más difícil que en el caso
dinámica, que muestra cómo durante la ins-
de las parálisis unilaterales, siendo muy útil la espirometría en
piración el diafragma no se mueve o asciende (“movimiento
bipedestación y decúbito (se observa una gran caída de la
paradójico”). La causa más frecuente es la infiltración del ner-
CVF en decúbito), así como la caída en la PIM.
vio frénico por cáncer de pulmón y el tratamiento es etiológico.
Otras causas posibles son la lesión del nervio frénico durante la El tratamiento es la ventilación positiva por vía nasal e incluso
cirugía cardiaca o post-radioterapia, la neuropatía por VVZ, y el marcapasos frénico (si el nervio frénico está intacto).
en una gran cantidad de casos, idiopática.
En general no requiere tratamiento, salvo cuando es muy
sintomática: se realiza plicatura del hemidiafragma afecto 17.2. Hernias diafragmáticas
(para disminuir el movimiento paradójico).
(Se estudia en Pediatría)

91
Tema 18
Trasplante pulmonar

18.1. Indicaciones del trasplante de pulmón Tardías


- Rechazo crónico.
De acuerdo con estadísticas del CENATRA en su reporte anual Es la causa fundamental de muerte pasado el primer año.
del 2019, únicamente 4 pacientes esperaban un trasplante Suele aparecer entre 8-12 meses. La forma más frecuente es
pulmonar. Es un trasplante poco habitual en nuestro país. bronquiolitis obliterante, pero también existe el síndrome res-
Fundamentalmente se realiza trasplante de pulmón en enfer- trictivo del aloinjerto (que afecta predominantemente a vér-
medades como el enfisema (es la indicación más frecuente), la tices). Debe sospecharse por síntomas insidiosos que simulan
fibrosis quística, la hipertensión pulmonar primaria y las enfer- una infección respiratoria. El tratamiento consiste en incre-
medades intersticiales. mentar la inmunosupresión con corticoides, y si no hay res-
Además de una repercusión funcional respiratoria, se deben puesta puede emplearse globulina antitimocítica o rituximab.
cumplir también otros requisitos como: - Complicaciones por la inmunodepresión.
Como infecciones por citomegalovirus (la neumonía por CMV
- Ausencia de coronariopatía o disfunción de ventrículo iz- debuta entre el segundo y sexto mes postrasplante), seguido
quierdo. por Aspergillus, y procesos linfoproliferativos.
- Buen estado nutricional.
- Integración social.
- Edad (<50-55 años para el trasplante de pulmón bipulmonar La incidencia de neumonía por Pneumocystis jirovecii es baja,
y <60-65 años para el trasplante de pulmón unipulmonar). debido al empleo sistemático de profilaxis.
- Ausencia de enfermedad sistémica que afecte a otro órgano
principal.

Las distintas modalidades de trasplante incluyen: unilateral,


bilateral y cardiopulmonar.
Regla mnemotécnica
18.2. Complicaciones Indicaciones de trasplante de pulmón
1 pulmon (unipulmonar) - 1 palabra:
Enfisema
La primera causa de mortalidad actualmente son las infecciones. 2 pulmones (bipulmonar) - 2 palabras:
Fibrosis quística
Precoces 3 órganos (cardiobipulmonar) - 3 palabras:
- Infecciones bacterianas nosocomiales (gramnegativos y esta- Hipertensión Pulmonar Primaria
filococos), la causa más frecuente global y en el primer año
postrasplante.
- Rechazo agudo o rechazo primario.
Es la causa más frecuente de muerte en el primer mes. Se con-
Recuerda que...
sidera un tipo de SDRA de aparición habitual en los primeros 3
días. Es difícil distinguirlo de una infección, por lo que suele re- Recuerda que las infecciones son actualmente
querirse una biopsia, generalmente transbronquial. Aumenta la primera causa de muerte por trasplante pulmonar:
el riesgo de padecer rechazo crónico. El tratamiento son cor- Primeros meses: gérmenes nosocomiales (bacterias)
ticoides i.v. a dosis alta. 2.º-6.º mes: CMV
- En relación con la cirugía (dehiscencia de anastomosis, etc.).

92
Tema 19
Malformaciones

19.1. Deformidades de la pared torácica

Las principales son el pectus excavatum (la más frecuente) y


el pectus carinatum (o tórax en quilla).
Otra anomalía es el síndrome de Poland: consiste en la
ausencia congénita del músculo pectoral mayor y puede acom-
pañarse de hipoplasia unilateral de la mama, tejido subcutáneo
e hipoplasia del pulmón ipsilateral.

19.2. Anomalías traqueobronquiales


Figura 1. Secuestro pulmonar intralobar en segmentos posteriores de lóbulo
inferior izquierdo (asterisco), con irrigación arterial procedente de aorta des-
Agenesia pulmonar cendente.
Consiste en la ausencia completa del pulmón, sin que exista
vestigio de irrigación vascular, árbol bronquial o parénquima Secuestros extralobares
pulmonar. La agenesia más frecuente es la agenesia unilateral 80% bajo lóbulo inferior izquierdo. A diferencia de los intralo-
del pulmón izquierdo. La bilateral es incompatible con la vida. bares, están separados del parénquima normal, rodeados por
una pleura visceral propia y tienen pedículo vascular propio,
con drenaje venoso hacia la circulación sistémica (vena cava,
Aplasia pulmonar aurícula derecha).
Consiste en la existencia de un bronquio rudimentario que
termina en fondo de saco, sin que existan vasos ni parénquima
pulmonar. INTRALOBAR EXTRALOBAR

PLEURA PROPIA No Sí
Hipoplasia pulmonar
El bronquio está totalmente formado pero su tamaño es redu- DRENAJE VENOSO Normal Anormal
cido y tiene escasas ramificaciones, generalmente de aspecto
quístico, y rodeadas de parénquima rudimentario con peque- APORTE
Arteria sistémica
ños vasos. SANGUÍNEO
El más frecuente es el complejo malformativo de Potter:
COMUNICACIÓN
hipoplasia pulmonar, agenesia renal, anomalías en los miem- No
VÍA AÉREA
bros y facies típica.

Tabla 1. Diferencias entre el secuestro pulmonar intra y extralobar.


Secuestro pulmonar
Consiste en una exclusión de un segmento del parénquima
pulmonar que presenta aporte sanguíneo propio procedente Quistes bronquiales congénitos
de la circulación sistémica (de la aorta torácica o abdominal, y Pueden ser pulmonares (20%) y mediastínicos (80%).
no de las arterias pulmonares) y que no presenta comunicación La complicación más frecuente es la infección recidivante.
con la vía aérea. Este tipo de malformación sucede, en la mayo-
ría de los casos, en el lado izquierdo, siendo su localización más El tratamiento, cuando son sintomáticos, es la resección qui-
frecuente el ángulo costofrénico posterior izquierdo. rúrgica.
Clínicamente, el secuestro pulmonar suele ser asintomático
pero puede manifestarse también como procesos infecciosos Malformación adenoide quística
frecuentes. Grandes quistes dando la imagen de un queso de gruyer que,
El método diagnóstico de elección es la arteriografía. Previo a en ocasiones, es una urgencia vital por atrapamiento aéreo en
la cirugía, debe realizarse también un esofagograma para des- alguno de los quistes.
cartar que exista una comunicación con el esófago.

Existen dos tipos de secuestros broncopulmonares:

Secuestros intralobares
Son los más frecuentes (generalmente en segmentos poste-
riores de lóbulos inferiores) y tienen mayor repercusión clínica
(neumonías de repetición).
El tratamiento de elección es la lobectomía.

93
Tema 20
Anatomía

20.1. Histología capa celular sobre tejido conectivo (membrana basal) formada
por neumocitos tipo I (aplanados, ocupan el 95% de la super-
ficie alveolar) y tipo II (cuboideos), siendo estos últimos los que
Zona de conducción sintetizan y liberan el surfactante pulmonar, una sustancia rica
Incluye la zona comprendida desde la tráquea hasta los bron- en fosfolípidos y proteínas que disminuye la tensión superficial
quiolos terminales, es decir, las 16 primeras generaciones. Se en el interior del alveolo pulmonar y con el ello el esfuerzo
denomina también espacio muerto anatómico porque no inter- necesario para distenderlos.
viene en el intercambio gaseoso. Está constituida por: El surfactante se puede detectar en líquido amniótico a partir
- Esqueleto fibrocartilaginoso. de la 34 semana de gestación, y su déficit está implicado en la
- Mucosa de epitelio respiratorio (pseudoestratificado). patogenia del síndrome de distrés respiratorio y en la aparición
Células ciliadas (mayoritarias), células mucosas y basales (entre de atelectasias. El mecanismo responsable de la disminución en
ellas las células neuroendocrinas, o células Kultschitsky). la síntesis de surfactante puede ser un defecto de perfusión del
parénquima pulmonar, una alteración de los neumocitos tipo II
A partir de los bronquiolos, la estructura de la vía respirato- o una hipoxemia mantenida.
ria cambia, siendo característica la desaparición de cartílago, El acino respiratorio es la unidad funcional pulmonar distal al
glándulas submucosas y células caliciformes. El epitelio de los bronquiolo terminal. Comprende bronquiolo respiratorio, con-
bronquiolos se aplana progresivamente hasta un epitelio pris- ductos alveolares, sacos alveolares y alveolos.
mático simple, constituido al 50% por células ciliadas y células
de Clara. Estas últimas producen una sustancia tensoactiva que
evita el colapso bronquiolar.

Cavidad nasal

Cilios
Tráquea
Laringe
Carina

Lóbulo superior Lóbulo superior


derecho izquierdo

Célula calciforme
Lóbulo medio Mediastino

Lóbulo inferior Lóbulo inferior


derecho izquierdo
Núcleo de célula
epitelial columnar
Figura 2. Anatomía del aparato respiratorio.

20.2. Radiografía de tórax

Las proyecciones básicas utilizadas en la radiología simple


Membrana basal de tórax son la postero-anterior (PA) y la lateral, ambas en
inspiración. Existen otras proyecciones como la lordótica o la
de parrilla costal. También pueden realizarse radiografías en
Figura 1. Mucosa del epitelio respiratorio. espiración (útiles para valorar neumotórax o cuerpos extraños).
Los patrones radiológicos básicos son el alveolar (edema pul-
monar, neumonía, distrés respiratorio o broncoaspiración) y el
Zona de transición intersticial (TBC miliar, EPID, etc.).
Está formada por los bronquiolos respiratorios (generaciones El signo de la silueta de Felson consiste en que cuando dos
17 a 19). Los bronquiolos respiratorios ya no presentan epitelio estructuras anatómicas (p. ej., masa) contactan, desaparece
pseudoestratificado, porque éste se transforma en cuboideo. el borde de ambas estructuras, mientras que si contacta una
No tiene células caliciformes pero sí células de Clara. estructura no anatómica (infiltrado) desdibuja el contorno
anatómico. Si la estructura no contacta, sino que se superpone
Zona respiratoria al hacer la proyección bidimensional, el contorno se respeta.
Constituida por los conductos y sacos alveolares y los alveolos. Cuando la silueta cardiomediastínica está borrada, permite
El epitelio alveolar es de tipo escamoso. Consiste en una única localizar una lesión:

94
Tema 20 · Anatomía

- Si borra el contorno cardiaco, es anterior y corresponde al - Si borra la aorta ascendente es LSD, segmento anterior.
segmento medial del lóbulo medio (en el hemitórax derecho) - Si borra aorta ascendente y aurícula derecha es el segmento
o a la língula (en el hemitórax izquierdo). apical del LID (segmento 6).
- Si no borra el contorno cardiaco, es posterior (segmentos me- - Si borra la aorta descendente posterior (segmento apical de LII
dial o posterior). o segmento posterior de LSI).
- Si borra el hemidiafragma, es lóbulo inferior.

95
Tema 21
Ventilación mecánica

21.1. Fundamentos 21.2. Indicaciones generales

La ventilación mecánica consiste en suplementar o suplir la - Insuficiencia respiratoria aguda.


mecánica ventilatoria mediante dispositivos que aportan pre- • Disminución del nivel de conciencia.
siones positivas en la vía aérea en diferentes momentos del • Inestabilidad hemodinámica.
ciclo respiratorio (durante la respiración espontanea se pro- • Agotamiento muscular u origen neuromuscular.
ducen presiones negativas en la vía aérea en la inspiración, y • Hipoxemia grave (PaO2 <40 mmHg), que no se corrige con
positivas en la espiración). oxígeno.
Puede aplicarse de forma no invasiva (VMNI, aplicando presio- • Insuficiencia respiratoria hipercápnica aguda.
nes positivas a través de una mascarilla nasal o nasobucal, con - Insuficiencia respiratoria crónica reagudizada.
el paciente consciente) o de forma invasiva (mediante intuba- • Deterioro del nivel de conciencia.
ción orotraqueal, generalmente con el paciente sedado y por • Inestabilidad hemodinámica.
tanto supliendo completamente la mecánica ventilatoria). En • Agotamiento muscular.
ambos casos, la presión positiva puede ser espiratoria (EPAP, • Acidosis mixta (respiratoria y metabólica).
expiratory positive airway pressure) o inspiratoria (IPAP, inspi- • Retención progresiva de CO2 que no se corrige con disminu-
ratory positive airway pressure). ción de la FiO2.
La aplicación de una EPAP mantiene abierta la pequeña vía
aérea impidiendo el colapso alveolar durante la espiración,
por lo que su efecto fundamental es mantener el intercambio
21.3. Indicación de las distintas modalidades
gaseoso durante todo el ciclo respiratorio, mejorando la oxi-
genación sanguínea. También permite la apertura de alveolos La VMNI se usa de forma domiciliaria para manejar patolo-
semicolapsados, aumenta el volumen residual pulmonar y gías crónicas:
disminuye el shunt intrapulmonar. Se puede aplicar EPAP
- La CPAP es el tratamiento de elección del síndrome de apneas
mediante la VMNI y mediante ventilación invasiva (entonces
del sueño.
se llama PEEP: positive end-espiratory pressure). La CPAP o la
PEEP no mejoran la hipercapnia, excepto en el SAOS, donde la - La Bi-PAP es el tratamiento de la insuficiencia respiratoria hi-
CPAP, al impedir el colapso de las VAS, impide la aparición de percápnica crónica.
hipoventilación. • Hipoventilación alveolar primaria refractaria a tratamiento
médico.
En caso de insuficiencia respiratoria hipercápnica la ventilación
• Síndrome de obesidad-hipoventilación.
mecánica debe incluir IPAP. La IPAP se aplica casi siempre
• Enfermedades neuromusculares y otros trastornos restrictivos.
sobre una EPAP basal, y proporciona una embolada de aire ins-
piratoria que ayuda a mejorar la ventilación alveolar, por tanto
lavando carbónico. Se puede aplicar IPAP en VMNI mediante La VMNI también se usa de forma aguda en urgencias ante
BiPAP (bilevel positive airway pressure), y en ventilación inva- enfermedades crónicas agudizadas:
siva mediante presión de soporte. La aplicación de IPAP trata - La CPAP puede ser parte del tratamiento de un EAP.
la hipercapnia, por lo que un SAOS con SOH asociada se debe - La Bi-PAP puede ser útil en ICC o EPOC agudizado con insufi-
tratar con BiPAP, no CPAP. En VMNI el paciente está conscien- ciencia respiratoria hipercápnica.
te y respira por sí mismo, ayudado por las presiones positivas
aplicadas. Algunas BiPAP disponen de la opción de sensar
La ventilación invasiva se usa ante patologías agudas o cró-
el volumen/minuto, y en caso de hipoventilación administrar
nicas agudizadas que requieran intubación orotraqueal.
emboladas extras de aire para asegurar una ventilación míni-
ma: son los llamados ventiladores volumétricos, especialmente - La presión positiva al final de la espiración (PEEP) se usa en
útiles en situaciones agudizadas. insuficiencias respiratorias hipoxémicas, principalmente en el
distrés respiratorio del adulto, insuficiencia cardiaca y en ate-
lectasias.
- La presión de soporte se usa para tratar insuficiencias respi-
ratorias hipercápnicas (EPOC, asma, neumonía grave, distrés,
ICC...).

21.4. Modalidades de soporte ventilatorio invasivo

El objetivo de la ventilación mecánica es asegurar la ventilación


alveolar evitando el daño pulmonar, el deterioro circulatorio y
la asincronía con el ventilador. Dentro de la ventilación invasiva
hay dos modalidades básicas: la controlada por el ventilador y
la espontánea. Se considera controlada si el ventilador deter-
mina el comienzo y el final de la fase inspiratoria, es decir, la
Figura 1. Ventilación mecánica no invasiva.
máquina dispara y cicla la ventilación. Pero se pueden definir

96
Tema 21 · Ventilación mecánica

otras modalidades de ventilación atendiendo a parámetros Incluyen:


como la variable de control (volumen o presión), o la secuencia - Ventilación con apoyo de presión (PSV: modo “presión
respiratoria (sustitución total o parcial de la ventilación). soporte”).
Modalidad de ventilación espontánea en la cual cada inspira-
Sustitución total de la ventilación: ventilación controlada ción es asistida por el ventilador hasta un límite de presión. Es
disparada por el paciente y limitada por presión. Puede utili-
o modo control (CMV: controlled mode ventilation)
zarse en periodos de estabilidad o como método de retirada, y
El ventilador aporta toda la energía necesaria para mantener tanto de forma invasiva como no invasiva. Requiere un centro
la ventilación alveolar, sin verdadera interacción paciente-ven- respiratorio intacto, estar libre de relajantes musculares y un
tilador. Está indicada en pacientes con fallo respiratorio grave patrón ventilatorio fiable. Es esencial que la alarma de apnea
o inestabilidad hemodinámica, y cuando reciben relajantes esté activada. El grado de soporte puede variar desde casi
musculares. Si el paciente es capaz de activar el trigger e incre- total hasta ventilación espontánea. Este modo mejora la sin-
mentar la frecuencia respiratoria, se denomina ventilación cronía con el ventilador, disminuye el riesgo de barotrauma y
asistida-controlada (A/C). La CMV-A/C es la modalidad más sirve para preparar al paciente para la desconexión (weaning).
utilizada, sobre todo al inicio del soporte ventilatorio. Puede Como desventajas, destaca la variabilidad del volumen circu-
aplicarse con: lante, por lo que no se recomienda en situaciones de compro-
- Control de volumen (VCV: volume-controlled ventilation). miso respiratorio grave.
La ventaja es que aporta un volumen constante que asegura - CPAP.
la ventilación alveolar. Sin embargo, la presión alveolar puede No se trata de un verdadero modo de ventilación (pues el
elevarse para poder aportar el volumen programado (p. ej. si ventilador no suministra ciclos ventilatorios). El ventilador
disminuye la distensibilidad), aumentando el riesgo de baro- mantiene de forma constante un nivel de presión positiva
trauma. Dado que el patrón de flujo es fijo (no se adapta a durante todo el ciclo. El paciente asume la mayor parte del
las demandas del paciente), hay mayor riesgo de asincronía y trabajo respiratorio, ya que genera su propio flujo inspiratorio,
desadaptación. su frecuencia respiratoria y su volumen circulante, simulando
- Control de presión (PCV: pressure-controlled ventilation). en gran medida el patrón de ventilación espontánea.
Tiene la ventaja de que ofrece una presión constante, redu-
ciendo el riesgo de barotrauma; y, dado que el flujo inspirato-
rio varía con las demandas del paciente, mejora la sincronía.
No obstante, dado que el volumen que ofrece es cambiante,
tiene el riesgo de ofrecer volúmenes insuficientes para una
ventilación óptima.

Sustitución parcial de la ventilación


Cualquier grado de ventilación en la cual el paciente participa
de manera activa y asume parte del trabajo respiratorio para
conseguir una ventilación eficaz. Sirven como procedimientos
de retirada del ventilador, aunque también pueden usarse
como modalidad primaria de ventilación. Las ventajas que ofre-
cen son: menor requerimiento de sedo-relajación, prevención
de la atrofia muscular por desuso, mayor sincronía paciente-
ventilador, mejor tolerancia hemodinámica y facilitar la retirada
del soporte. Desventajas: incremento del trabajo respiratorio
y dificultad para asegurar un intercambio gaseoso adecuado.

97
Tema 22
Semiología respiratoria

Murmullo vesicular disminuido


- Derrame pleural
- Neumotórax
- Enfermedad pulmonar obstructiva
- Condensación

Percusión disminuida o timpánica: Vibraciones disminuidas: Vibraciones aumentadas:


- Enfisema - Derrame pleural - Condensación pulmonar
- Neumotórax - Neumotórax

Condensación pulmonar:
- Aumento de vibraciones vocales
- Pectoriloquia áfona: ruidos de susurro se transmiten a pared torácica con gran claridad
- Egofonía o broncofonía: similar a la pectoriloquia pero cualidad nasal
- Soplo tubárico: condensación pulmonar con bronquio permeable (respiración bronquial)
- Soplo anfórico: condensación pulmonar cavitada con bronquio permeable

Sibilancias:
- Ruidos musicales o prolongados
- Luz traqueobronquial
- Aumento turbulencia
- Espiratorios e inspiratorios
- Ejemplos: asma, cuerpos extraños

Estridor:
- Sonido agudo y chillón. Generalmente inspiratorio.
- Por obstrucción de la vía aérea superior (estridor laríngeo o traqueal)

Roce pleural:
- Como el cuero. Al final de la inspiración e inicio de la espiración.
- Indica inflamación de la pleura

Estertores (crepitantes):
- Producidos por el paso del aire a través de bronquios o alvéolos ocupados por líquido
- Ruidos húmedos, cortos, burbujeantes e inspiratorios
- Ejemplos: neumonía, edema agudo de pulmón
- El tono depende del tamaño de la cavidad afectada:
• Baja tonalidad:
- Tercio inicial inspiración
- Exudado en bronquios de grande y mediano tamaño
• Tonalidad media:
- Tercio medio inspiración
- Afectación bronquios más periféricos
• Alta tonalidad o finos:
- Tercio final inspiración
- Afectación parénquima pulmonar

Roncus:
- Ruidos similares a ronquidos
- Producidos por el paso del aire a través de las grandes vías aéreas ocupadas por secreciones
- Ejemplos: infecciones respiratorias, broncoaspiración

Soplo anfórico:
- Ruido similar al producido al soplar por la boquilla de una botella vacía
- Producido por el paso del aire por cavernas o cavidades
- Ejemplos: cavernas tuberculosas grandes, algunos casos de neumotórax

Figura 1. Semiología respiratoria.

98
Valores normales en
Neumología y Cirugía Torácica

CONCEPTO VALORES NORMALES


PaO2 >80 mmHg

SpO2 97-100%

Gradiente A-a Hasta 15-20 mmHg

FEV1/FVC >80

FEV1 >80%

CV >80%

CVF >80%

CPT 80-120%

PEF >80%

PEF 25-75 >60%

DLCO 80-120%

Shunt V/Q = 0

Espacio muerto V/Q = infinito

Unidad silente V/Q = 1

Complianza 60 ml/cmH2O

PaFI (PaO2/FiO2) >300

Ca pulmón no microcítico
T1a <2 cm
T1b 2-3 cm
T2a 3-5 cm
T2b 5-7 cm
T3 >7 cm
T4 Cualquier tamaño
N0 No hay ganglios
N1 Ganglios pulmonares ipsilaterales
N2 Ganglios mediastínicos ipsilaterales
N3 Ganglios contralaterales

EPOC
GOLD I (FEV1/FVC <70) + → FEV1 >80%
GOLD II (FEV1/FVC <70) + → FEV1 50-80%
GOLD III (FEV1/FVC <70) + → FEV1 30-50%
GOLD IV (FEV1/FVC <70) + → FEV1 <30%

SAHS
Índice de Apnea-Hipopnea (IAH) <5 apneas/h
SAHS Grave >30 apneas/h

Hipertensión pulmonar
PAP media (Presión arteria Pulmonar) <25 mmHg
PAP ejercicio <30 mmHg
PCP (presión capliar pulmonar) <18 mmHg
HTP grave >50 mmHg

Tabla 1. Valores normales en Neumología y Cirugía Torácica.

99
BIBLIOGRAFÍA

- Harrison’s: Principles of Internal Medicine, 18.ª Edición. DL Longo, AS Fauci, DL Kasper, SL Hauser, JL Jameson, E Braunwald.
McGraw Hill, 2011.
- Farreras-Rozman: Medicina Interna, 16.ª Edición. C Rozman, F Cardellach, JM Ribera, A de la Sierra, S Serrano. Elsevier, 2009.
- OncoGuia de Pulmó. Actualització 2008. Pla Director d’Oncologia, Agència d’Avaluació de Tecnologia i Recerca Mèdiques, CatSalut.
Generalitat de Catalunya, 2008.
- Rivas de Andrés JJ, Jiménez López MF, Molins López-Rodó L, Pérez Trullén A, Torres Lanzas J. Normativa SEPAR: Normativa sobre
el diagnóstico y tratamiento del neumotórax espontáneo. Arch Bronconeumol 2008;44(8):437-48.
- Plaza V, Fernández M, Gómez F, López A, Molina J, Navarro D, et al. Guía Española para el manejo del asma 2009. Arch Bron-
coneumol 2009;45(supl.7): 2–35.
- Guías de práctica clínica de la American Cancer Society (www.cancer.org).
- Diagnóstico, tratamiento y prevención de la exacerbación del asma en adultos. Catálogo Maestro de Guías de Práctica Clí-
nica: IMSS-806-17.
- Prevención y detección temprana del cáncer de pulmón en el primer nivel de atención. Catálogo Maestro de Guías de Prác-
tica Clínica: GPC-SS-022-18.
- Diagnóstico y tratamiento de cáncer pulmonar de células no pequeñas. Catálogo Maestro de Guías de Práctica Clínica: GPC-
IMSS-030-19.
- Medina-Morales F, Salazar-Flores M. Frecuencia y patrón cambiante del cáncer pulmonar en México. Salud Publica Mex 2000;
42: 333-336.
- Diagnóstico y tratamiento del derrame pleural. Catálogo Maestro de Guías de Práctica Clínica: IMSS-243-09.
- Diagnóstico y tratamiento, enfermedad tromboembólica venosa. Catálogo Maestro de Guías de Práctica Clínica: GPC-
IMSS-425-18.
- Prevención, diagnóstico y tratamiento de la tromboembolia pulmonar aguda en el embarazo, parto y puerperio. Catálogo
Maestro de Guías de Práctica Clínica: GPC-SS-656-18.

100
RESPUESTAS ENARM

Tema 1. Fisiología y fisiopatología


1. RESPUESTA CORRECTA: C
Comentario: El VEF1 o FEV1 es el volumen de gas espirado en el primer segundo de una espiración forzada. La Capacidad Vital
Forzada (CVF) es el volumen total que el paciente espira mediante una espiración forzada máxima. La relación VEF1/CVF tiene un
valor normal es de 0.8. Si es menor de 0.7 es diagnóstico de obstrucción al flujo aéreo.

2. RESPUESTA CORRECTA: C
Comentario: Diversos factores desplazan la curva de disociación de la hemoglobina a la derecha o izquierda haciendo que dis-
minuya o aumente aún más la afinidad de la hemoglobina por el oxígeno. Los factores que desplazan la curva hacia la derecha
(disminuyen la afinidad de la hemoglobina por el oxígeno y por tanto, aumentan la cesión tisular de O2) son: la acidosis (↑ H+ y ↓
pH –efecto Bohr–), el aumento de la temperatura, el aumento del 2-3 difosfoglicerato (DPG) y el aumento de la PaCO2. Desplazan
la curva hacia la izquierda los contrarios: el aumento del pH, el descenso de la PaCO2, el descenso del DPG y el descenso de la
temperatura.

Tema 2. Neoplasias pulmonares


3. RESPUESTA CORRECTA: C
Comentario: Las personas en quienes se deben realizar estudios de tamizaje para cáncer pulmonar son:
1. Edad entre 55 y 74 años, fumadores activos o que dejaron de fumar en los últimos 15 años y que tengan índice tabáquico
igual o mayor a 30 paquetes/año;
2. Edad igual o mayor a 50 años, antecedente de índice tabáquico igual o mayor a 20 paquetes/año y con otros factores de riesgo
para cáncer pulmonar, y
3. Presencia de factores de riesgo diferentes del tabaquismo, con o sin síntomas respiratorios o pérdida de peso involuntaria,
independientemente de la edad.
En estos grupos anteriormente se sugería realizar una radiografía de tórax como paso inicial. No obstante, en la actualidad la
GPC recomienda solicitar de primera instancia una tomografía de tórax simple. Actualmente tampoco se recomienda realizar
citología de esputo como prueba de cribado.

4. RESPUESTA CORRECTA: D
Comentario: La máxima incidencia del cáncer pulmonar ronda los 60 años. La mayoría de tumores pulmonares son primarios
y malignos. El pulmón, no obstante, es también uno de los principales lugares de aparición de metástasis a distancia. El tipo
histológico más frecuente tanto en México como en el mundo hasta hace unos años era el epidermoide, pero actualmente es el
adenocarcinoma, En cuanto a los síntomas locales, los tumores que se presentan con localización central (carcinoma epidermoide
y microcítico) dan clínica por obstrucción bronquial: cursan con tos productiva (lo más frecuente) con/sin hemoptisis.

5. RESPUESTA CORRECTA: D
Comentario: Se ha demostrado que existe una relación entre el número de cigarrillos consumidos y todos los tipos de canceres
pulmonares. La relación es dosis dependiente; el riesgo relativo (RR) tras haber consumido 40 paquetes-año es de 20. El riesgo de
desarrollar un cáncer de pulmón en los exfumadores se aproxima al de los no fumadores (aunque no llega a igualarse); al cabo
de cada 10 años disminuye a RR = 0.6 (respecto a un fumador).

Tema 3. Enfermedades de la pleura


6. RESPUESTA CORRECTA: D
Comentario: El estudio bioquímico del líquido pleural permite la clasificación de los derrames pleurales en exudados y trasudados.
Si el derrame cumple al menos uno de los siguientes criterios de Light se clasificará como exudado: proteínas en líquido pleural/
proteínas séricas >0,5, LDH en liquido pleural/LDH sérica >0,6 y LDH en líquido pleural >2/3 del límite de la normalidad en suero.
El derrame neoplásico (por ejemplo, por mesotelioma) es la causa más frecuente de derrame sanguinolento y la segunda causa
más frecuente de exudado (la primera es el paraneumónico). El resto de causas mencionadas generarían un derrame de tipo
trasudado (la pericarditis constrictiva podría manifestarse como insuficiencia cardiaca).

7. RESPUESTA CORRECTA: A
Comentario: Un neumotórax es la presencia de aire en el espacio pleural. Puede ser espontáneo (sin traumatismo torácico previo)
o traumático. Debe sospecharse en pacientes con dolor y disnea brusca con abolición del murmullo vesicular. Si se sospecha
neumotórax y en los casos dudosos, se debe practicar una radiografía de tórax en inspiración y espiración máxima, que mostrará
despegamiento de las hojas pleurales, colapso pulmonar subyacente, hiperclaridad donde el aire sustituye al parénquima pulmo-
nar y, si es a tensión, desviación contralateral del mediastino.

Tema 4. Enfermedad Pulmonar Obstructiva Crónica (EPOC)


8. RESPUESTA CORRECTA: C
Comentario: Ante todo paciente con sospecha de EPOC realizaremos inicialmente una espirometría con prueba broncodilatadora
y una radiografía del tórax. El diagnóstico de EPOC requiere demostrar la presencia de una obstrucción bronquial crónica poco
reversible con tratamiento. Este hecho se confirma a través de la espirometría: disminución del FEV1, capacidad pulmonar total

101
conservada o aumentada y disminución del índice de VEF1/CVF. La relación VEF1/CVF menor a 0.7 es el criterio de patrón obs-
tructivo, aunque la clasificación de la gravedad se establece por el FEV1.

9. RESPUESTA CORRECTA: A
Comentario: Los broncodilatadores son el pilar fundamental del tratamiento de la enfermedad pulmonar obstructiva crónica,
preferiblemente inhalados: agonistas beta-2 adrenérgicos y anticolinérgicos (inhibidores competitivos del receptor muscarínico de
acetilcolina).

Tema 5. Asma
10. RESPUESTA CORRECTA: C
Comentario: La espirometría demuestra un típico patrón obstructivo, con disminución del FEV1 y de la relación FEV1/CVF. Así
mismo se objetiva un aumento del VR de hasta un 400% y una CVF menor al 50%. La reversibilidad se evidencia mediante un test
de broncodilatación: el aumento del FEV1 mayor o igual al 12% o 200 ml tras la administración de un broncodilatador inhalado
confirma el diagnóstico; pero la negatividad de la prueba broncodilatadora no lo descarta.

11. RESPUESTA CORRECTA: A


Comentario: La reacción asmática se debe a un mecanismo de hipersensibilidad de tipo inmediato (tipo I), con liberación de his-
tamina, bradiquinina y SRS-A (sustancia de reacción lenta: leucotrienos C4 y D4).

Tema 6. Tromboembolismo pulmonar


12. RESPUESTA CORRECTA: A
Comentario: Lo más frecuente es que el electrocardiograma de pacientes con tromboembolia pulmonar (TEP) sea inespecífica-
mente anormal; la alteración más común es la taquicardia sinusal. Si el TEP es extenso, aparecen signos de sobrecarga derecha:
desviación del eje a la derecha, patrón SI-QIII-TIII (es lo más específico), bloqueo de rama derecha y ondas “P pulmonale”.

13. RESPUESTA CORRECTA: D


Comentario: Es la prueba de certeza o gold standard, es decir, la más sensible y específica. Actualmente tiene poco uso al ser sus-
tituida por métodos de diagnóstico menos invasivos. Permite establecer el diagnóstico definitivo de TEP mediante la visualización
de defectos de llenado intraluminal en más de una proyección. Su mayor utilidad aparece cuando la probabilidad clínica de TEP
difiere del resultado de la gammagrafía pulmonar, o cuando esta tiene una probabilidad intermedia de TEP. Cuando la sospecha
clínica sigue siendo alta y las pruebas diagnósticas negativas, debe realizarse una angiografía.

14. RESPUESTA CORRECTA: B


Comentario: Es la primera prueba a realizar cuando la sospecha clínica de TEP es baja. Tiene un elevado valor predictivo negati-
vo (cercano al 100%), de forma que ayuda a excluir el diagnostico de TEP si resulta negativo. De acuerdo con la GPC, un valor
positivo se considera mayor de 1000 mcg/L en personas con baja probabilidad clínica, y mayor de 500 mcg/L en casos de riesgo
mayor. Si es positivo, los D-dímeros pueden estar elevados por otras causas que producen también fibrinolisis endógena como el
IAM, la cirugía, los carcinomas, o casi cualquier enfermedad sistémica.

15. RESPUESTA CORRECTA: A


Comentario: En TEP de riesgo bajo o intermedio, el tratamiento va dirigido a la prevención secundaria (evitar nuevos episodios de
TEP) mediante anticoagulación, inicialmente con heparina (heparina no fraccionada o de bajo peso molecular); la fibrinolisis no
está indicada. En el TEP de bajo riesgo puede darse de alta al paciente de manera precoz y hacer manejo ambulatorio, mientras
que en el TEP de riesgo intermedio debe realizarse vigilancia hospitalaria para poder detectar precozmente inestabilización hemo-
dinámica (que sí sería indicación de fibrinolisis).

Tema 9. Enfermedades intersticiales 1: neumonitis intersticiales idiopáticas


16. RESPUESTA CORRECTA: A
Comentario: En la fibrosis pulmonar idiopática (FPI), suele tratarse de pacientes de alrededor de 50 años. Además de disnea de
esfuerzo y tos seca, aparecen síntomas generales (astenia, anorexia, pérdida de peso, artralgias). La exploración puede evidenciar
crepitantes secos (“en velcro”) bibasales al final de la inspiración (teleinspiratorios), taquipnea, cianosis, acropaquias (dato fre-
cuente pero tardío) y, en casos avanzados, cor pulmonale y fallo cardiaco derecho. Tradicionalmente se creía que en la FPI existía
una progresión radiológica, presentando un patrón de vidrio deslustrado o esmerilado en las fases iniciales de la enfermedad
(alveolitis aguda), que traduce un aumento difuso del tejido intersticial. Después se formarían infiltrados nodulares y reticulono-
dulares y, finalmente, en la fase de fibrosis, aparece un patrón reticular difuso, en panal de abeja o queso suizo, que constituye
el estadio final.

17. RESPUESTA CORRECTA: C


Comentario: La patogenia de la FPI consiste en una respuesta inflamatoria de etiología desconocida que desemboca en una
fibrosis del tejido pulmonar. Aunque se desconoce la causa, se sabe que se produce fijación de inmunocomplejos a los macró-
fagos alveolares. Los macrófagos así activados producen citocinas (LT-B4 e IL-8), que atraen polimorfonucleares y eosinófilos

102
(cuyos productos provocan lesiones locales y aumentan la permeabilidad de los neumocitos tipo I), y sustancias estimulantes de
los fibroblastos del intersticio y paredes alveolares. Finalmente, la fibrosis afecta a los vasos, oblitera los alveolos y retrae las vías
aéreas terminales, formándose áreas quísticas recubiertas de epitelio bronquial.

Tema 10. Enfermedades intersticiales 2: neumonitis de causa conocida


18. RESPUESTA CORRECTA: A
Comentario: La silicosis es una neumoconiosis producida por la exposición a sílice o cuarzo cristalino, por lo que tienen riesgo los
trabajadores de minas de carbón, fundiciones, cerámica y canteras de granito. Las lesiones aparecen al cabo de 15-20 años de
exposición. Consiste en fibrosis pulmonar que, aunque diseminada, está focalizada en nódulos silicóticos menores de 1 cm, sobre
todo en lóbulos superiores y adenopatías hiliares, a veces con calcificación en cáscara de huevo. Puede ser asintomática o mani-
festarse con tos, expectoración crónica y disnea de esfuerzo, debido a la patología asociada, generalmente bronquitis crónica,
producida por el frecuente hábito tabáquico en esta población.

Tema 13. Hipertensión Pulmonar (HTP)


19. RESPUESTA CORRECTA: B
Comentario: Se desconoce con exactitud la prevalencia de hipertensión pulmonar en la población general. La mayoría de los casos
es secundaria a cardiopatía izquierda (Grupo 2, por aumento de presión poscapilar) y a neumopatías crónicas graves (Grupo 3,
por vasoconstricción hipóxica sostenida; que puede llegar a producir sobrecarga de cavidades derechas -situación conocida como
cor pulmonale).

103
NOTAS
NOTAS
NOTAS

También podría gustarte